You are on page 1of 291

‫ﻣﻌﻠﻮﻣﺎت ﻋﺎﻣﮫ ﻋﻦ اﻟﺘﻤﺮﯾﺾ‬

‫‪NURSING PRACTIC 2019‬‬

‫اﻟﻧﺳﺧﺔ اﻟﺳﺎدﺳﺔ ‪٢٠١٩‬‬

‫ھذا اﻟﻣﻠف ﻟﺟﻣﯾﻊ ﻣن ھو ﻣﮭﺗم ﺑﺎﻟﻌﻠوم اﻟﺗﻣرﯾﺿﯾﺔ ﻣﺟﻣوﻋﮫ ﺷروﺣﺎت ﻣﻊ اﻣﺛﻠﮫ ﺗﺳﺎﻋد ﻓﻲ ﻓﮭم‬
‫ اﺳﺗراﺗﯾﺟﯾﺎت اﺳﺗﺧراج اﻹﺟﺎﺑﺎت اﻟﺻﺣﯾﺣﺔ ﺑﺄذن ﷲ‬

‫ﻏﯾر ﻣﺻرح ﻷي ﺷﺧص ﺑﺎﻟﻣﺗﺎﺟرة او اﻟﻧﺳﺦ او اﻟﺑﯾﻊ ﻓﮭو ﻣﺟﺎﻧﺎ ﻟﻠﻛل وأرﺟو ا ﻋدم اﺳﺗﺧداﻣﮫ ﻓﻲ‬
‫ أﻏراض أﺧرى وﷲ وﻟﻲ اﻟﺗوﻓﯾق‬

‫أﺗﻣﻧﻰ ﻟﻛم اﻟﺗوﻓﯾق واﻟﻧﺟﺎح واﻟدﻋﺎء ﻟﻲ ﻓﻲ ظﮭر اﻟﻐﯾب‬

‫ﺷﻜﺮ ﺧﺎص ﻟﻼﺳﺘﺎذه‬


‫ﻏﺎده اﻟﺨﻠﯿﻔﮫ‬
A mother came to the clinic and afraid that her baby maybe will have
meningitis as his brother already have

What should the nurse tell the mother?


A- it’s not common for any infant
B-The meningitis has a vaccine now
C- The vaccine is reducing the risk for this decease

A pregnant in 9 weeks had bleeding for several timesnnnnn

The nurse should appropriate that pregnant has which of the following?

A- Ectopic pregnancy
B- Normal during the pregnancy for 9 month

C- Low potassium

D-Anemia

Pregnant woman had traffic accident and she came to the emergency
department during the nursing assessment Palpation from the nurse the
woman has acute pain and possible bleeding
Which of the following nursing diagnosis should the nurse expect?

A- Abruption placenta
B- Pain


What is the complication of child had aortic regurgitation?

A. Cyclones during crying


B. Cyclones during feeding
C. Clipping nail
D. Cyclones

A Woman Patient has molar pregnancy, what is the best advice from the
nurse?

A. Hysterectomy
B. Not get pregnant again
C. Wait a year then try to pregnant

Reason for spinal defect is folic acid


Spinal defect Prone

A new infant has born and his mother is confused if the infant may have
Gestational diabetes as she had it during her pregnancy

Which of the following the mother is the consider of her baby?

A- Hypoglycaemia

B- Anaemia

C- Hyperglycaemia

D- Infection
A child has an abdominal pain and Hepatomegaly, what should the nurse
expect?

A- Hepatitis

B- Cancer

C- Liver damage

D- Low level of hemoglobin

A vaccine for 12 month is Measles

Definition of delegation

Delegation is the assignment of any responsibility or authority to


another person (normally from a manager to a subordinate) to carry out
specific activities. It is one of the core concepts of management
leadership. However, the person who delegated the work remains
accountable for the outcome of the delegated work.

How is the Nurse Manger evaluation the quality of work?

A-Evaluate patient’s satisfaction at discharge time

B- Asking everyone inside the hospital

C- Trusting

D- Time management
A nurse manager is not satisfied of the staff work

The best action should be taking?

A- Arrange meeting with the staff

B- Investigation

C- Ignorance

D- Acceptance

Rational:
The nurse manager should investigate and identify the reasons for the staff’s low
performance and then arrange a meeting to discuss ways for improvement.

Best normal therapy for patient with Multiple sclerosis is Cold therapy

A woman complaining of mastitis during Brest feeding the best way to


avoid this symptom is Apply a Warm or Cold Compress.

Which of the following reason for exercise air after surgery?

A. Prevent lungs atelectasis.


B. Comfortable
C. Prevent anxiety
D. Prevent bleeding
A patient diagnosed with obsessive-compulsive disorder (OCD)
continually carries a toothbrush, and will brush and floss up to fifty
times each day. The healthcare provider understands that the patient's
behavior is an attempt to accomplish which of the following?

A- Experience pleasure
B- Relieve anxiety
C- Avoid social interaction
D- Promote oral health

Before administering a dose of furosemide (Lasix) to a 2-year old with a


congenital heart defect, the nurse should confirm the child's identify by
checking the hospital ID band and

A. Verifying the child's room number.


B. Verifying the identity with a second nurse.
C. Asking the parent the child's name.
D. Asking the child to tell you his name.

Before sending a client for a CT with contrast dye, what the nurse's most
important action?
A- Teach about the need for post-procedure hydration.
B- Verify that the informed consent is complete.
C- Place the side rails of the bed up before transport.
D- Check the client's health record for allergies.
Which is the priority for a child who has returned to the unit after
surgery to repair a cleft palate and lip?
A- Interacting with others
B- Managing pain
C- preventing infection
D- Ambulating every hour

Rational:
Managing pain after surgery is highly important. Infection prevention interventions
will be utilized; however, pain management has a higher priority immediately after
surgery.

What precautions are necessary when caring for a patient with Hepatitis
A?
A- Gowning before entering the room
B- Wearing gloves for direct care
C- Wearing a mask at all times
D- Placing the patient in a private room
Following surgery, the patient is returned to the unit and complaining of
pain on a level of 9 out of 10. The nurse reviews the chart and finds the
surgeon's order for "10 mg MSO4" written post-operatively. Before
administering the morphine, what should the nurse do first?
A- Set up a piggyback infusion system.
B- Contact the surgeon before proceeding.
C- Prepare the medication for administration.
D- Call the pharmacy to send up the morphine.

Rational:
The joint commission created a “Do Not Use” list of medical abbreviations. MSO4
can be confused with magnesium sulfate. Instead, morphine Sulfate should be
written as “morphine sulfate”

When a patient is admitted with acute influenza, what type of isolation is


MOST appropriate?

A- Reverse isolation
B- Contact isolation
C- strict isolation
D- Respiratory isolation
While caring for a patient in the post-anesthesia care unit (PACU), a
nurse plans to keep the patient warm. What is the MUST important
reason for this action?
A. To preservenutritionalstores
B. To prevent cutaneous vessel dilation
C. To decrease patient anxiety
D. To lower risk of infection resulting from chill

Rational:

Maintaining patient normothermia pre-, peri- and post-operatively is a


critical element of preventing surgical site infections and other
complications such as metabolic acidosis, cardiovascular effects,
increased respiratory distress and surgical bleeding. All tissues heal
most effectively in optimal conditions of oxygenation, perfusion and
body temperature.

When using a fire extinguisher, the hose is aimed at the


A- Area around the flame.
B- Base of the flame.
C- Middle of the flame.
D- Top of the flame.
A staff nurse working on the gynecological surgery unit assigns the
unlicensed assistive personnel (UAP) to ambulate a new post-
hysterectomy patient. Two hours later, the nurse checks on the patient
and finds that the patient has not seen the UAP. When the nurse
discovers the UAP talking on her phone in the lunch room, what should
the nurse do?
A- Report the UAP to the nursing supervisor.
B- Reprimand the UAP for her neglect.
C- Begin an incident report.
D- Remind the UAP that phones may only be used during breaks.

A 16-year old male calls the clinic because he's worried that he might
have gonorrhea. However, he doesn't want his parents to know. What
should the nurse tell him?
A- "I'm sorry, but you are a minor. We will need a parent's consent
before we can do anything."
B- "We can test and treat you. All results will be kept confidential."
C- "We can test and treat you, but your parents will see it on your chart."
D- "We can test you without parental consent, but if there are
positive results, we must notify the health department."
Which of the Following Is the First Step for Inhaler Technique
Education in Patients with Asthma?

A- Breathe out gently.

B- Place mouthpiece between lips.

C- Shake the inhaler.

D- Wait a few seconds then repeat the above process

Which of the following is the best describe of hospital Vision?

A- To provide a quality of care we would want for ourselves,

B- Treat every individual in a non-judgmental manner, ensuring privacy,

fairness and confidentiality

C-Improve the responsiveness of our services for the benefit of our

patients and their families

D-Helping local people live longer, healthier lives.


Which of the following uses for abdominal examination?

A- Percussion, Inspection, Auscultation, Palpation

B-Auscultation, Percussion, Palpation, Inspection

C- Inspection, Auscultation, Percussion, Palpation

D- Inspection, Palpation, auscultation, percussion.

A nurse manger apply decision making with the staff this is can be
described as which of the following

B- Decision-making

C- Judgement

D- Decentralized

A Nurse Manger is Sharing Decision with the Staff this can be described

As which of the following?

A- Democratic

B- Decision-making

C- Judgement

D- Not all of above


Prior to transport to surgery, the operating room calls the unit nurse to
administer the preoperative medication as ordered. After giving the
medication, the unit nurse discovers that the surgical consent has not
been signed. What should the nurse do first?

A- Ask the patient to sign the consent before transport.


B- Notify the surgeon.
C- Notify the nursing supervisor.
D- Call the OR to cancel the procedure.

A patient is transferred to the post-anesthesia care unit (PACU) after


surgery with epidural anesthesia. After taking the patient's vital signs,
what should the nurse assess next?
A- Spinal headache
B- Postoperative pain
C- Bladder distention
D- Ability to move legs

When making shift assignments, which patient is most appropriate for a


nursing assistant?

A- A newly admitted patient with a seizure disorder.


B- A post-hysterectomy patient who needs to ambulate.
C- A dehydrated patient with an electrolyte imbalance.
D- A post-op laparotomy patient who needs education.
Your home health client has been throwing up for several days. You
now find her lying in her bed. She is pale, lethargic, and her eyes are
dull. She is also anxious. Which of the following is most likely?

A- Congestive heart failure


B- Multiple sclerosis
C- fecal impaction
D- Hypovolemic shock

A patient is admitted to the unit with an order for seizure precautions.


Which action is most appropriate?

A- Move the client to a room closer to the nurses' station.


B- Serve the client's food in paper and plastic containers.
C- Maintain the client's bed in the lowest position.
D- Ensure that soft limb restraints are applied to upper extremities.

A home nurse visits a patient for routine what is the first thing should
the nurse to do first

A- Collect information and schedule visit

B- Referral the patient to nearest hospital

C- Speak with the family

D-Provide advices
During a vaginal delivery of woman 38 years old the nurse should
consider the risk of which of the following?

A- Acute bleeding and coma

B- Low potassium

C- Brain Injury

D- Feta heart rate

Lochia red sometimes happens during postpartum to bleed heavily with


smell for the first three to ten postpartum days, the nurse should expect
that may to?

A- Lochia pink

B- Sign of abnormal hemoglobin

C- Infection

D- Bleeding

A head nurse of a Coronary Care Unit delegated the staff a senior nurse
in that unit what initial step must the head nurse implement before?

A. Check the hospital policies for delegating tasks

B. Explain the task to the senior nurse


B. Negotiate with the senior nurse
C. Take the signature of the senior nurse
The most common cause of hypercalcemia in Paget's disease patients is
hyperparathyroidism and adenectomy (surgical removal of all or part of
the gland) may improve the biochemical and sometimes also the clinical
symptoms of Paget's disease.

Vaccination HIB

The CDC and World Health Organization recommend these vaccines for
Hajj: hepatitis A, typhoid, meningitis, hepatitis B and rabies. Proof of
meningitis vaccination is required to take part in Hajj. Routine
immunizations against diseases like measles or pertussis are also
recommended if you have not received them.

A disability patient already on the wheel chair has visited by her


nephew, her nephew response to the patient that she has done something
wrong and that god punished her and become in this condition

Which of the following describing her nephew behaviour?

A- Moral

B- Anger

C- Abuse
A health female her period. Every 28 days and doing now diet the
period. Or menstruate has stopped since five month

The nurse should expect which of the following

a- Secondary amenorrhea

b- Primary amenorrhea

c- Cancer

d- HIV

The American Academy of paediatrics suggests that removal of the


tonsils under certain conditions.

Which of the following meets these conditions?

A. times viral tonsillitis per year

B. Infrequent snoring and nasal quality

C. Three times bacterial tonsillitis per year

D. Tonsillitis accompanied by adenoid inflammation


Multiparous patient on day 1 postpartum is asking the nurse to send her
baby to the nursery so she can sleep.

What is the most likely phase of psychological adaptation?

A. Taking-in

B. Letting-go

C. Taking-go

D. Letting-in

Rational:

Taking in: Mom is preoccupied with her own needs.

Taking hold: Mom is more ready to resume control of her body, baby and taking
on mothering role. May need reassurance if inexperienced.

Letting go: Mom adjusts to her new role.

Which goal is priority after surgical repair of cleft lip?

A.Pain Management

B. Prevent infection

C. Increase mobility

D. Develop parenting skills


A pregnant client with organ dysfunction presents with visual changes.
3+ deep tendon reflexes and a severe headache. At a subsequent follow-
up visit the client reports that the symptoms have resolved.

What is the most likely reason for the improvement?

A. Sleeping in a lateral position

B. Consumption of 8 ounces of water per day

C. Exposure to bright lighting

D. Ambulation every 30 minutes

Nurse receives a telephone call from the admission office of the hospital
and is told that a patient with streptococcal meningitis will be admitted
to the Medical Unit. The nurse is planning to apply infection control
measures for the patient.

Which type of isolation precaution the nurse must observe?

A. Droplet precautions

B. Contact precautions

C. Airborne precautions

D- All of above
What is the proper procedure of doing breast self examination?

A. Use the palm of the hand to feel the lumps

B.Apply three different levels of pressure to feel breast tissues

C. Stand when performing a breast self exam

D. Perform self exam annually

During Cardiopulmonary Resuscitation (CPR) for a 75-year-old man in


the Emergency Department, the doctor introduced himself as a leader for
the CPR.

What is the most appropriate leadership style for this situation?

A. Autocratic

B. Democratic

C. Bureaucratic

D. Laissez-faire

Mr. (A) admitted to CCU with the congestive heart disease his vital
Signs are HR 120 b/m. BP 110/60 mmhg . Stroke volume is 80. The
cardiac output for Mr (A) is

A. 182

B. 1700

C. 2400

D. 9600
19-year-old girl was scheduled for the extraction under general
anaesthesia. Her pre-operative was done and the consent was signed by
her, but her nail polish and trim her long nails, requirement.

What should be the nurse's action in response to?

A. Explain why nails need to be cleaned

B. Respect the patient's right to refuse

C. Record and inform physician

D. Remove as per protocol

During the home visit by the community health nurse she observes that
the infant baby during breastfeeding stop sucking and the milk is leaked
from the nose. What would the nurse suspect?

A.Pyloric stenosis

B. Tracheoesophageal fistula (TEF)

C. Cleft lip

D. Cleft palate
A woman is admitted to the hospital with ruptured ectopic pregnancy. A
laparotomy is scheduled preoperative. Which one of the following goals
is most important for the nurse to include on the client plan of care?

A. Fluid replacement

B.Pain relief

C. Emotional support

D. Respiratory therapy

A labouring woman desires to participate in her caesarean and have pain


control.

Which of the following methods would satisfy the women's needs?

A. Epidural block

B. Pudendal block

C. Meperidine injection

D. General anaesthesia
After a code blue, a doctor announced the death of a patient. A signature
other begins to show signs of decreased level of consciousness.

What is the priority intervention of the nurse at this time?

A. Offer a meal

B. Ensure safety

C. Set up an IV line

D. Offer a glass of water

The nurse was educating a postpartum woman during discharge about


importance of breast feeding.

Which of the following if said by the women, indicates the need for
further education?

A. Breast milk is nutritionally balanced

B. Breast milk reduces the risk of infection

C. Breast feeding promotes mother-child bonding

D. Breast feeding prevents pregnancy


A patient is being admitted in medical unit and has orders for fluid
restriction of 1400ml / 24 hours. He is receiving iV medications in 50ml
solutions QID.

How much fluid should the caring nurse allocate for cra intake in 24
hours?

A. 400 ml

B. 800 ml

C. 1000 ml

D. 1200 ml

Rational:

The client is to receive 1400ml/24 hr. To calculate how much the client
can take by mouth, we first have to calculate the medication intake (50 x
4 = 200), then we subtract 200 from 1400 (1400 – 200 = 1200).

Note* QID means 4 times per day

Nurse is assessing a child (an infant) with pyloric stenosis. Which of the
following is likely to note?

A. Diarrhoea

B. Projectile vomiting

C. Swallowing difficulties

D. Currant jelly like stool


A Rehabilitation nurse reviews post stroke patient immunisation history
which immunisation is a priority for a 72 year old patient

A. hepatitis A vaccine

B. hepatitis B vaccine

C. rotavirus vaccine

D. pneumococcal vaccine

Pregnant woman in the 3 months have a thrombus in the right leg what
do you expect the doctor will order

A. Heparin

B. Insulin

C. Warfarin

D. Aspirin

Pregnant woman 34 weeks has hypertension this problem is identified


as?

A. Hypertension

B. pregnancy induced DM

C. PIH (Pregnancy induced hypertension)

D. placenta Previa
A woman breastfeeds her infant one or two hours and her infant cries
most of the time and she feels pain in her breast. Which of the following
instructions are appropriate for the nurse to give the mother?

A. Regulate breast feeding every 3 hours

B. That’s normal feeding problem

C. Shift to bottle feeding

D. Start weaning your baby

Treating pregnant women diagnosed with syphilis is considered?

A. Primary level of prevention

B. Secondary level of prevention

C. Tertiary level of prevention

D. Primary and secondary

Neonate is admitted to the neonatal intensive care unit for observation


with a diagnosis of probable meconium aspiration syndrome (MAS) the
neonate weight 4.650 grams and is at 40 weeks gestation which of the
following would be the priority problem?

Impaired skin integrity

Hyperglycaemia

Risk for impaired parent - infant child attachment

Impaired gas exchange.


Patient arrived to emergency department with tracheal deviation and has
neck swelling, what is the first intervention should be done by the nurse?

A. Refer patient to do x-ray

B. Give patient bronchodilator

C. Drained blood sample for bag

D. Place patient in trendelenburg position

A client has atrial fibrillation the nurse should monitor the client for

A. cardiac arrests

B. cerebrovascular accident

C. heart block

D. ventricular fibrillation

During the home visit the community health nurse find a child hungry
and a appears with poor hygiene, this is considered as

A. Neglecting

B. avoidance

C. physical abuse

D. Assault
The drug of choice for anthrax is

A. Tetracycline

B. Ciprofloxacin

C. Quinolones

D. Streptomycin

The registered nurse should give the highest priority of care for?

A. Patient newly diagnosed with hypertension

B. Patient complaining of severe headache

C. Patient with chest pain

D. Patient with peptic ulcer

First time mother is concerned about her 6 months old infant is not
gaining enough weight what should the nurse tell the mother

A. Birth weight doubles by 6 months of age

B. Birth weight doubles by 3 months of age

B. The baby will eat what he needs

D. You need to make sure that the baby finishes each bottle
The drug of choice for patient with Parkinson's disease is

A. Phenytoin

B. Levodopa

C. Haloperidol

D. Lithium

Which one of the following signs and symptoms is associated with


increased intracranial pressure?

A. restlessness and confusion

B. bradycardia and hypertension

C. tachycardia and hypotension

D. respiratory depression and headache

Which of the following is considered one of the common complications


following abdominal total hysterectomy?

A. DVT

B. paralytic ileus

C. constipation

D. perforation
A client diagnosed with conductive hearing loss asks the nurse to
explain the cause of the hearing problem. The nurse plans to explain to
the client that this condition is caused by which problem?

A- A defect in the cochlea

B- A defect in cranial nerve viii

C- A physical obstruction to the transmission of sound waves

D. A defect in the sensory fibres that lead to the cerebral cortex

When planning the care of a client who has undergone an abdominal


hysterectomy which nursing measures is most helpful for preventing
post-operative complication and facilitating an early discharge?

A. Re-establishing oral fluids and the nutrition

B. Promoting ambulation and movement

C. Maintaining accurate intake and output

D. Exploring feelings about altered image


One of the following problems is considered as quinidine sulphate
toxicity:

A. Tachycardia

B. Hypertension

C. Ringing in the ear

D. Alopecia

A nurse discusses high risk complication with a group of women at a


prenatal clinic. Which client would the nurse identify as being highest
risk for developing complication during pregnancy?

A. A 25 years old gravida 1 client

B. a client with the placenta implanted on the fundus of the uterus

C. a client who has nausea and vomiting during the first trimester

D. A 30 years old client with DM

Which common uterus position cause of abortion

A. Longitudinal

B. Oblique

C. A retroverted uterus

D. Linear
During stressful situation the patient is complaining from chest pain,
sweating, tremors. And visual difficulties what is the expected cause?

A. Bipolar disorder

B. Panic disorder

C. Hyperglycaemia

D. Hypertension

A solder was brought to triage area after being exposed to chemical


weapons. Signs and symptoms of nerve gas exposure were noticed. A
nurse prepares for medical management. Which medication should the
nurse prepare for the patient?

A. Atropine

B. Adrenaline

C. Sodium nitrate

D. Sodium thiosulphate
42 year-old patient was to be prepared for the upper gastrointestinal
endoscopy. The patient anxiously asked the nurse about what will
happen in the procedure. The nurse explained that the endoscopy tube
will be inserted down his throat which will make him severely gag but
he will have to swallow it. Which type of communication the endoscopy
nurse has used here?

A. Assertive

B. Interactive

C. Therapeutic

D. Non therapeutic

When developing their plan of care for a client with suicidal ideation
Development Goals to address which issue is priority?

A. Self-esteem

B. Sleep

C. Stress

D. Safety
A client with depression and suicidal ideation voices feeling of self-
doubt and powerlessness and is very dependent on the nurse for most
aspects of hair care according to Erikson’s stage of growth and
development the nurse determines the client to be manifesting problem
in which stage?

A. Trust vs mistrust

B. Autonomy vs shame and doubt

C. Initiative vs guilt

D. Industry versus inferiority

A client with schizophrenia is withdrawn, is suspicious of others, and


project blames, the client's behaviour reflects problem in which stage of
development as identified by Erikson?

A. Trust vs mistrust

B. Autonomy vs shame and doubt

C. Initiative vs guilt

D. Intimacy versus isolation


A 22 year-old man presented to the mental health clinic with low mood
and general loss of interest in activities and inability to experience
pleasure he admitted suicidal thoughts and extreme lack of energy he
was prescribed a selective serotonin reuptake inhibitor to be taken daily
one month later he presented to the clinic and reports feeling more
energetic but still has a low mood what is the Patience level of risk
committing suicide at this time?

A. None

B. Low

C. Medium

D. High

Psychiatric patient appears violent for himself and others was put in the
room alone during the period of exacerbation, then patient calm down
and Informed the nurse I am OK now let me with others, but the nurse
refused that as a punishment way. At which underline label will the
nurse accused under the court?

A. Abandonment

B. False imprisonment

C. Negligence

D. Duty to act
A nurse is preparing to meet with an individual whose spouse recently
diagnosed with Alzheimer’s disease the nurse should know that the
primary goal of treatment is?

A. curing is a disease

B. Maximizing the functional ability and improve quality of life

C. Hearing the Alzheimer’s patient and safe control

D. Making all decisions for the patient and confirming to home


environment

Lithium therapy affects which of the following hormones?

A. Cortisone

B. TSH

C. FSH

D. Oestrogen
67 years old female has stayed at a geriatric home for 10 years. Last
night at 3 AM, the client become aggressive and agitated what should
the nurse do?

A. Enter her room and switch off the light

B. Enter her room and playing the music

C. Avoid anything to make her anger

D. Ask her to be quiet

A female patient diagnosed with cancer she intended to spend a part of


her money for the poor people in case of her cure. this action by the
patient is considered?

A. Denial

B. Acceptance

C. Bargaining

D. Anger

An 8 year old child is crying and said because of the death of his parent
what should the nurse do?

A. Ask him to stop crying

B. Ignore him

C. Allow him to cry and accept his feeling

D. Keep him in a quiet environment


Which food should the nurse eliminate from the diet of a client in
alcohol withdrawal?

A. Milk

B. Regular coffee

C. Orange juice

D. Eggs

The most common risk factors for Suicides is?

a. Dehydration

B. electrolyte imbalance

C. cardiogenic shock

D. Depression

Community is experiencing an outbreak of staphylococcal infections the


nurse and instructs residents that the most common mode of
transmission is by ?

A. Respiratory droplet

B. Contaminated food

C. Hands

D. Soil
Nurse is caring for a 58-year-old patient

(See lab result).

Test Result Normal Value

Magnesium 2.8 0.7-1.2mmoI/L

Which ECG change is the nurse expected to note?

A. Prolonged QRS

B. Multiple P waves

C. Prominent U waves

D. Depressed ST segment

During your colonoscopy, you'll lie on your left side on an exam table.
You'll get sedatives through an IV in your arm, and you'll go to sleep.
During the procedure, the doctor puts a tube-like instrument called a
colonoscopy into your rectum

WHY WE USE APGAR SCORE

This test was not designed to predict a baby's long-term health, behavior,
intelligence, personality, or outcome. It was designed to help health care
providers tell a newborn's overall physical condition so that they could
quickly decide whether the baby needed immediate medical care.
Baby born with hematoma on head

Although cephalohematomas are generally not a major threat, they can


increase the risk of health complications such as anemia and jaundice. If
a baby develops anemia, they may require a blood transfusion. Like
cephalohematomas, jaundice may only be a mild issue, and it is easily
treatable

Pyloric stenosis is a narrowing of the opening from the stomach to the


first part of the small intestine (thepylorus). Symptoms include
projectile vomiting without the presence of bile. This most often occurs
after the baby is fed.

Other names: Pylorostenosis, infantile hypertro...

Symptoms: Projectile vomiting after feeding

Complications: Dehydration, electrolyte proble...

Treatment: Surgery

The most common symptoms noted in a baby with pyloric stenosis is


forceful, projectile vomiting. This kind of vomiting is different from a
"wet burp" that a baby may have at the end of a feeding. Large amounts
of breast milk or formula are vomited, and may go several feet across
room.
How is pyloric stenosis diagnosed?

Your doctor might also recommend: Blood tests to check for dehydration or electrolyte
imbalance or both. Ultrasound to view the pylorus and confirm adiagnosis of pyloric
stenosis. X-rays of your baby's digestive system, if results of the ultrasound aren't clear

Is pyloric stenosis life threatening?

Pyloric stenosis is a fairly rare condition that affects newborns and


infants. ... Babies with pyloric stenosis need surgical treatment right
away to avoid life-threatening dehydration and electrolyte
imbalance. Pyloric stenosis happens in about 3 out of every 1,000
births. But surgery can fix the problem

Nurse goes to do education that’s called Primary Education


o The employer should be encouraged for the efforts that’s done by
staff -Encourage reward
Example:
Patient after discharge informed the head nurse about one nurse that she
was very kind with him and the head gave the nurse one day off. What is
the type of the action by the head nurse?

o When the charge nurse or the manger need to evaluate the staff, they
should use the strategy of patient Satisfaction

o When we give the medications for the patients must be on time, it's
not legal to give some patients and other later
Example A nurse has given the medication for some patient and other
will be giving after the break that’s would be Legal Accountability

o A nurse explaining the procedure for patient under endoscopy


Assertive
o The best Isolation for patient with TB Airborne

o Patient coming to gynecological and want to marriage the staff that's


called Ethical responsibility

o When we care for Patient has BM is higher, we should use Nutrition


management

o Stopping through chemical substances in rivers it is called water


pollution
o A Vaginal discharge green with foul odor, painful during urination its
Trichomonas
Parents have sickle disease100% the children will have. 100%

o When the Patient come to ER after RTA exhibiting dyspnea,


cyanosis, SOB, has tracheal deviation we should expect that he is in
Tension Pneumothorax

o New staff nurse is starting inserting vein access in patient median


cephalic vein when she heard another nurse calling for help, she
quickly secured the vein cannel and through the needle in regular box
rather than the sharp box; when she was asked who through the
needle in regular box she admitted doing that, this principal action
called Accountability

o Myocardial infarction (MI)ST- elevation


o Atropine using for Patient with bradycardia.
o The most common disease is Bacteria
o Best treatment for Rheumatic fever is Benzedrine penicillin
o When we see the site for IV infusion swollen and cold on touch and
We immediately stop the infusion

o Type of Isolation Category Is Indicated for Patient with Tuberculosis:


Airborne Isolation

o The antidote for morphine overdose is: Naloxone (Narcan)


o Surgery procedure that must be done within 24 hours is called:Urgent
Surgery

o If a patient develops a complication during a blood transfusion,


the nurse’s first action should do to is to Stop the transfusion

o The phase that include action of the nursing care plan is


called:Implementation

o Streptococcus transmutation is from person to another:droplet and


contact

o Type of precaution for smallpox Airborne




o Both parents have sickle cell anemia chances of having a baby
inheriting the disease 100%

o Antibiotic for Gonorrhea
A- Penicillin
B- Tetracycline
C- Cipro
D- Cephalosporin

The nurse teaches a client with chronic obstructive pulmonary disease


(COPD) to assess for signs and symptoms of right-sided heart failure.
Which of the following signs and symptoms should be included in the
teaching plan:

A- Clubbing of Nail Beds


B- Hypertension
C- Peripheral Edema
D- Increased Appetite

Patient with pulmonary emboli complains of pain, dyspnea, and a


A patient recently underwent coronary artery graft (CABG) surgery.
Which of the following nursing diagnose PRIORITY?

A- Anxiety
B- Impaired gas exchange
C- Acute pain
D- Sleep deprivation

A nurse is providing care to a patient with a new skin graft on left leg.
The patient is upset and the nurse notes copious red drainage oozing
around the dressing. The nurse should immediately:
A- Apply firm pressure for 10 to 15 minutes

A co-worker informs that the nurse about experiencing increased level
of stress associated with daily responsibilities to help cope with
professional stress, the nurse should encourage the co-worker to;

A. Make a list of unfinished tasks


B. Complete complex mental task before physical tasks
C. Acknowledge daily accomplishments
D. Spend time with colleague away from work

A nurse administers albuterol nebulizer to a child with asthma


exacerbation. The nurse measures pulse Oximetry and auscultates the
lungs to determine whether the goal of clear respiratory status has been
met. The step of nursing is called:
A- Assessment
B- Diagnosis
C - Implementation
D- Evaluation

The most effective procedure to prevent spared of infection is:


A- Wearing gloves
B- Using antiseptic Sterilization
C- Hand washing

Patient with gout is encouraged to increase fluid intake.


What is the benefit of the fluids?
Drinking plenty of water may reduce the risk of an attack and of urate
forming crystals in joint tissues.
o A home health care nurse visits a patient diagnosed with rheumatoid
arthritis. The nurse gathers information about the pain level after the
use of the prescribed pain medication to check on the effectiveness of
the intervention. What is this phase of the nursing process called?

A. Assessment
B. Diagnosis
C. implementation
D. Evaluation

o A public health Nurse is developing a care plan for a low-income


community in which there is a general low level of education and high
level of crime the nurse is using the nursing process to identify the
community needs.
Which step should be utilized?
A. Diagnosis
B. Evaluation
C. Assessment
D. Intervention

o The most important and first priority for new infant born is Avoiding
heat loss
o When is the best time to collect urine specimen for routine urinalysis
and culture and sensitivity?

A. Early morning
B. Later afternoon
C. Midnight
D. Before breakfast

o The physician has determined that the client with hepatitis has
contracted the infection from contaminated food. The nurse
understands that this client is most likely experiencing what type of
hepatitis?

A. Hepatitis A
B. Hepatitis B
C. Hepatitis C
D. Hepatitis D

o A client with bipolar disorder taking lithium tells the nurse that he has
ringing in his ears, blurred vision, and diarrhea. The nurse notices a
slight tremor in his left hand and a slurring pattern to his speech.
Which of the following actions by the nurse is appropriate?

A - Administer a stat dose of lithium as necessary.


B- Request an order for a stat blood lithium level.
C- Give an oral dose of lithium antidote.


If both parents have disease 100 % of children have disease

One parent has trait, other has disease 50 % of children have disease

Both parents trait 25 % of children have disease

o A primigravida woman is instructed to offer her breast to the baby for


the first time within 30 mint after delivery
What is the benefit of offering the breast for newborn early?

A- Initiate the occurrence of milk letdown


B- Stimulate milk production by the mammary acini
C- Make sure that the baby able to get colostrums
D- Allow the mom to practice breastfeeding in the presence of the health
worker

o A community health nurse is implementing an adult immunization


program in the neighborhood. Which of the following would MOST
likely be a universally recommended adult vaccination and dose
frequency general population?

A- Influenza vaccine every year


B- Tetanus_dephtheria toxic every 20 year
o The client has burns on both legs. These areas appear white and
leather-like. No blisters or bleeding are present, and there is just a
“small amount of pain.” How will the nurse categorize this injury?

A. Full-thickness
B. Partial-thickness superficial
C. Partial-thickness deep
D. Superficial

o Which of the following vaccines is not done by intramuscular (IM)


injection?

A. Measles vaccine Vaccine for child 9-12 month - Measles


B. DPT
C. Hepa-B vaccine
D. Tetanus toxoids

o In order to reduce the risk of disease transmission from a patient with


diphtheria, which of the following standard precautions would be the
nurse implemented?

A- Droplets
B- Airborne
C- Ventilatory
D- Contact
o A newly-admitted patient's medication orders include Donepezil
hydrochloride (Aricept). The nurse knows this medication is
prescribed for?

A- Bipolar disorder
B- Schizophrenia
C- Alzheimer's disease
D- Major depression

o Patient use potassium sparing diuretics


We should observe Hyperkalemia

Labor 6cm is called Active

he first stage of labor is the longest and involves three phases: Early Labor Phase –The
time of the onset of labor until the cervix is dilated to 3 cm.
Active Labor Phase – Continues from 3 cm. until the cervix is dilated to 7 cm.
Transition Phase – Continues from 7 cm. until the cervix is fully dilated to 10

The Rhogam should be given 72 H


RhoGAM, if given to you at the right time, will prevent your immune system from reacting to
your baby's blood. RhoGAM is made from human blood and may carry a risk of transmitting
disease-causing agents. RhoGAM prevents the Rh-negative mother from making antibodies
during her pregnancy

o The best time administer Oral contraceptive with patient has sexual
abuse from her family IS 12-24 Hours

o Patient with prostatectomy the nurse always should Assess urinary


function

o Planning to get pregnant and have the first baby without spine bifida
the patient should take folic acid dailyA- 400MCG
o A visiting family member informs the psychiatric nurse that another
patient is being sexually inappropriate with other patients. The nurse
tells the visitor to inform that patient's nurse. Which of the following
describes the nurse's actions?
A- Practical
B- Negligent
C- Organized
D- Delegation

o A nurse schedule patient for a surgical procedure to take place in one


week when would the nurse most likely implement surgical
education?
A- After admission to the hospital
B- Start during this visit
C- Immediately prior to anesthesia
D- After operation

Autonomy is the right to make your own decisions. The client is able to
make decisions regarding care after being informed of all available
options. It includes the client's ability to make decisions in advance
regarding end of life decisions and completing appropriate forms in
consultation with the health care provider.
Justice refers to the appropriate and fair distribution of resources. When
clients observe policies in place that they feel conflict with information
that has been provided to them, they feel they are being treated unjustly.
Beneficence refers to actions that promote the well-being of others. The
nurse who is exemplifying beneficence takes positive actions to help
clients.
Fidelity refers to the agreement to keep promises. The nurse is obligated
to be an advocate for her client and makes promises to do so.
o Alzheimer patient with dementia the doctor said to his son that they
will be responsible to take medical decisions because of his case the
patient loose two ethical aspects??
A- Autonomy & fidelity
B- Varsity & justice

o A nurse is providing care to a patient with a new skin graft on the leg.
Thepatient is upset and the nurse notes copious red drainage oozing
aroundthe dressing the nurse should immediately:

A. Lift the dressing to assess the area


B. Ask if the patient is having any pain
C. Apply firm pressure for 10 to 15 minutes
D. Assess the apical pulse

o The nurse is caring for a 4 year-old patient with a diagnosis of cystic


fibrosis and pneumonia. The child is feeling better on the 3rd day of
the hospitalization and "wants to play" What would be the BEST
choice ofentertainment?

A. Blowing bubbles
B. Looking at picture books
C. Watching videos
D. Riding in a wagon
o The mother of a chlid who has been recently diagnosed as having
hemophilia is pregnant with her secound chlid. She asks the nurse what
thechances are that this baby will also have hemophilia. The nurse`s
bestresponse would be :

A. There is no chance the baby will be affected.


B. Theres is a 25% chance the baby will be affected.
C. There is a 50% chance the baby will be affected.
D. There is a 75% chance the baby will be affected.

o A nurse is preparing the room for a patient who will be admitted the
post-anesthesia care unit after major abdominal surgery. The patient
received general anesthesia care. Which of the following should the
nurse place at the bedside in the room?

A. sterile suction catheter


B. oral way
C. incentive spirometer
D. face mask


The nurse is planning care for several children who were admitted
during the shift. Daily weights should be the plan of care for the
Child who is receiving:

A- Total parenteral nutrition(TPN)


B- Supplement oxygen
C- Intravenous anti-ineffective
D- Chest physiotherapy
A home care nurse instructing a mother of child diagnosed with cystic
fibrosis (CF) about the appropriate dietary measures. Which diet should
the nurse tell the mother that the child needs to consume?

A. Low calorie – low fat diet


B. High calorie – restricted fat
C. Low calorie – low protein diet
D. High calorie – high protein diet

Laboring client is in the first stage of labor and has progressed 6 cm in


cervical dilation. In which of the following phases of the first stage does
cervical dilation occur most rapidly?

A-Preparatory phase
B-Latent phase
C-Active phase
D-Transition phase

Nurse is caring for a client in labor. The nurse determines that the client
is beginning in the 2nd stage of labor when which of the following
assessments is noted?
A-The client begins to expel clear vaginal fluid
B-The contractions are regular
C-The membranes have ruptured
D-The cervix is dilated completely.
Which of the following is described as premature separation of a
normally implanted placenta during the second half of pregnancy,
Usually with severe hemorrhage?
A. Placenta Previa
B. Ectopic pregnancy
C. Incompetent cervix
D. Abruptio placentae

Parkinson’s disease patients have a hard time swallowing because they


lose control of their mouth and throat muscles As a nurse which of the
following is a Parkinson’s diet?
A- Solid
B- Semi solid
C- Liquid
D- Clear liquid

Which of the following is the correct order for putting on personal


protective equipment?
A- gown, mask, goggles, gloves
B- mask, gown, goggles, gloves
C- goggles, mask, gloves, gown
D- gloves, gown , goggles, mask.
A 6 year old male is diagnosed with nephrotic syndrome. In your
nursing care plan you will include which of the following as a nursing
diagnosis for this patient?
A-low NA,low fat
B-high NA,High fat
C-low NA,High fat
D-high NA,Low fat

Ebola virus is one of six known viruses within the genus Ebola virus.
Four of the six known ebolaviruses, including EBOV, because a severe
and often fatal haemorrhage fever in humans and other mammals they
using Contact to prevent infection.

Meningitis and septicaemia can kill in hours - know the symptoms. The
first symptoms are usually fever, vomiting, headache and feeling unwell,
the Meningitis vaccine called meningococcal vaccine

MODE OF TRANSMISSION: RSV (respiratory syncytial virus) is most


likely transmitted through direct contact with infectious secretions
Most side effect of oral contraceptive except is?

A. Fever

B. Nausea

C. Headache

D. Ovaries cancer

NOTE* Oral contraceptives lower the risk of ovarian cancer! However,


FEVER is NOT a side effect of oral contraceptives.

Medical complications often accompany Down syndrome?

A. hypospadias

B. renal failure

C. Congenital heart defects

Swine influenza (novel H1N1 and H3N2v) spreads from person to


person, either by inhaling the virus or by touching surfaces contaminated
with the virus, then touching the mouth or nose. Infected droplets are
expelled into the air through coughing or sneezing.

Which of the following as risk factors for otitis media?


A- Respiratory infection
B- Loss hearing
C- Vomiting
D - Cataract
Which of the following taking highest priority according to Maslow's
hierarchy of needs?

A. Enough breath

B. safety

C. love and belong

Cleft lip surgery repair between 10 weeks and 6 months, cleft palate
surgery repair 12-18 month

Position after surgery Side or Back. Restraint should be Elbow

Remember That the Priory before the Surgery to Prevent


Aspiration

After The Surgery Impaired Feeding

The Best Age to Fix a Cleft Lip or Palate

Although there are some differing schools of thought on the matter, most
plastic surgeons believe that the ideal patient age for undergoing cleft
palate repair surgery is between 6 to 18 months of age (though the
favored age for cleft lip repair is generally much earlier, at about 10 to
12 weeks old). This age appears to be advantageous partially because
healing times are fast, the patient's memory of the recovery process is
short, and the area around the cleft hasn't had much of a chance to
develop surrounding tissues in an abnormal manner
Nurse manager first days in new hospital will..?

A. introduce plan and strategy to top manager in hospital

B. introduce himself to staff

C. observation for 3 month before begin work

Nurse Manager Assign work according nursing competency this is?

A. Delegate

B. planning

C. controlling

Nurse community visit school students what the most important topic to
focus on.

A. Sport safety

B. Bicycle safety

MRSA is contact precaution, Smallpox Airborne isolation

TB precaution transmission airborne

TB patient nurse ALWAYS wearing N95 mask


Patient treating with TPN what is the first action the nurse should be
observed?

A- Hypoglycemia

B- Hyperglycemia
C- Hypokalemia

Delivery Post-Operative

Stopping swelling in its tracks, massage is particularly good at relieving


any type of pain the body may be experiencing after surgery. The
amount massage reduces pain is already the number one reason why
patients seek massage therapy, so this should be no different after
surgery

What Are the Signs & Symptoms of Hirschsprung Disease?

The symptoms of Hirschsprung disease can vary depending on how


severe it is. Children with severe cases usually will have symptoms
within the first few days of life.

Newborns with Hirschsprung disease may:

• Be unable to pass stool within the first or second day of life


• Have a swollen belly, bloating, or gas
• Have diarrhea
• Vomit (which may look green or brown)

A newborn who can't poop within the first 48 hours of life is often how
doctors find Hirschsprung disease. This red flag can be very valuable in
diagnosing the condition.
Less severe cases might not be spotted until a child is a little older, or
sometimes even later. Symptoms in these cases are usually milder but
can be long-lasting (or chronic chronic). They can include:

• A swollen belly
• Constipation
• Trouble gaining weight
• Vomiting
• Gas

Older kids with Hirschsprung disease might have a growth delay


because the condition can affect the body's ability to absorb nut
Weber test ‫ﻣﮭﻢ ﺟﺪا‬
PURPOSE: determination of a conductive vs. a sensorineural hearing loss

• strike tuning fork and place base in the centre of the forehead or

the top of the head

• ask if the tone is louder in the left ear, the right ear or equally loud

in both ears

• due to the sound localization process,

o in a patient with a unilateral conductive hearing loss, the

sound will be louder in the affected ear (airborne sounds

mask bone conduction in the normal ear; conductive loss

prevents masking in affected ear è sound is perceived to be

louder in affected ear)

o in a patient with unilateral sensorineural hearing loss, the

sound is louder in the normal ear (no signal is transduced

by the cochlea on the affected side, therefore the sound is

louder on the normal side and is perceived to be coming from

that side)

o in a normal person or a person with symmetrical hearing loss,

it is equally loud in both ears


‫اﻟﺮﺟﺎء ﻣﻨﻜﻢ اﻟﺘﺮﻛﯿﺰ ﻋﻠﻰ اﻟﺘﻄﻌﯿﻤﺎت‬

‫ﺟﺪوﻟﯿﻦ ﺧﺎﺻﮫ ﺑﺘﻄﻌﯿﻤﺎت اﻟﺮﺿﯿﻊ‪ -‬واﻟﻄﻔﻞ‬

‫ھﻨﺎك اﺧﺘﻼف ﺑﯿﻨﮫ‬

‫ ‬

‫‪For Infants‬‬
For Children


Allergy develops after many previous exposures symptoms may include
hives, itching, stuffy or runny nose. It can cause asthma symptoms of
wheezing, chest tightness and difficulty breathing. Symptoms begin
within minutes after exposure to which of the following ?

D. Latex

A. Peanuts

B. Strawberries

C. Eggs

A 12-year-old child has been diagnosed with Diabetes insipidus which


of the following is disorder?

A. Posterior pituitary

B. Adrenal medulla

C. Anterior pituitary

D. Adrenal cortex
While planning for discharge education for a mother or rickets, the nurse
knows to include the need

for an adequate

Which food should the mother choose for her child?

A. Potato and squash

B. Orange and tomatoes

C. Egg yolk and fish

D. Milk and yogurt

A 8- year-old child was admitted with suspected appendicitis. A nurse


was evaluating the child’s

Condition and the mother stated that the child did have his bowel
movement for the past two days and requested for laxatives. What is the
risk of giving laxative to patient with appendicitis?

A. Pain

B. Fever

C. Rupture

D. Diarrhea
A nurse manger is preparing and writing a plan for dealing disasters
(code red). Which procedure is the top priority for the nurse manager
the plan?

A. Infection control

B. Staff orientation

C. Patient education

D. Patient relocation

A 33 month-old infant is admitted to the Emergency Department (ED)


with fractured arm. The mother indicated that while the child was
crawling fell down the stairs and broke his arm. Which of the following
observation would lead the nurse to suspect that this is a victim of
abuse?

A. Age inappropriate injury

B. Pattern and shape of the injury

C. Child is appearing malnourished

D. Improper explanation of the cause of injury


A high school girl, who has fears of being obese, visited the primary
healthcare centre with her the mother reports that her daughter refuses to
eat with the family, and often pretends being a sleep

Which of the following disorders best describe girl?

A. Bulimia

B. Obesity

C. Substance Abuse

D. Anorexia Nervosa

A 40-year-old women patient with Parkinsonism Medical Ward. The


patient stated that she has the past two weeks. The nurse was planning to

Which type of diet is most suitable for Parkinson’s?

A. Solid

B. Liquid

C. Semi solid

D. Clear liquid
The nurse is assessing a child (an infant) with pyloric stenosis.

Which of the following is likely to note?

A. Diarrhea

B. Projectile vomiting

C. Swallowing difficulties

D. Currant jelly like stool

A nurse has been teaching a new mother how to feed her infant who
born with a cleft lip and palate before surgical repair of the defect.
Which of the following action from the mother indicate that the nurse
teaching has been successful?

A. Burp the baby frequently

B. Prevent the infant from crying

C. Placing the baby flat during feeding

D. Keep the infant prone following feedings


A nurse receives a report on a client transferred to the post-anaesthesia
care unit (PACU) after a cesarean delivery. Epidural pain management
was used for the procedure. Which is the safest position for this client
while in the PACU?

A- Knee-chest position

B- Semi-Fowler position

C- Modified Sim position

D- Dorsal recumbent position

Rationale

Postoperative patients who have received epidural anesthesia should be


positioned in the semi-Fowler position to prevent upward migration of
the opioid in the spinal cord. This decreases the risk for respiratory
depression associated with epidural anesthesia. In addition, the client
may also be tilted to one side or the other to further prevent the potential
for supine hypotension.
19 years old girl was underweight and malnourished. 163cm, her body
weight was 46kg has admitted to medical ward for observed that on the
second day she inducing dinner. On asking, she replied that inducing
helps clear the stomach bacteria. Which aspect of the nursing care area is
altered?

19 years old female has admitted to the hospital due to she is losing
weight

A. Self-concept

B. Health perception

C. Value-belief system

D. Nutrition management

Rational:

Health perception is defined as subjective ratings by the affected


individual of his or her health status. The client’s health perception is
altered as she thinks she can clear her stomach from bacteria by inducing
vomiting.
Many injured victims of road traffic accident Emergency Room for

treatment. Due to the high

Doctor asked a nurse to perform preliminary assess prioritise them.

Which of the following is the best way to assess?

A. Pain

B. Weight

C. Restlessness

D. Facial expression

Rational:

Patient prioritizing should be based on ABG. Restlessness indicate low

oxygen level (hypoxia), which will need immediate attention.


A nurse enters the room of a patient who is on ECG monitor, the patient

complains of not feeling well, the (see image).

What is the most likely interpretation?

A. Sinus tachycardia

B. Ventricular fibrillation

C. Normal sinus rhythm

D. Ventricular tachycardia

A progressive enlargement of a multi-nodular go tracheal compression


including pain at the site of the ear and jaw, difficulty swallowing,
change of pf breath by compressing the esophagus. There is in otherwise
the patient is at risk.
What is the preferred treatment?
A. Iodine treatment

B. Thyroid hormone treatment


C. Radioactive iodine treatment
D. Surgical resection of abnormal thyroid
A nurse diagnosis a patient with readiness for the..., the diagnosis is classified
within which of the nursing diagnoses?
A. Acute nursing diagnoses
B. Risk nursing diagnoses
C.Wellness nursing diagnoses
D. Possible nursing diagnoses

Rational:
Wellness nursing diagnosis is defined as the readiness and desire of the
individual, family or community to transition to a level of higher
wellness.

A 69-year-old patient was discharged from instructions to have follow-


up physiotherapy session paralysis as a result of the brain stroke. A few
physiotherapy sessions, he showed fast recovery. Sadly that despite his
progress he will remain
Which of the following should be the prioritise patient?
A. Support for health maintenance
B. Increase activity and exercise
C. Identify coping mechanism
D. Enhance self esteem

A patient complains of pain when standing upright hump on the upper


back. In the past year, slightly shorter. The doctor has suggested tests to
What is the best appropriate intervention?
A. Instruct the patient in the use of prescribed magnesium supplements
B. Have the patient sleep propped on two pain
C. Prepare the patient for a CT scan of both
D. Instruct the patient in the use of vitamins
What major factor should alert a nurse to discontinue the sponge bath for
an infant who is febrile?
A. Shivering
B. Tachycardia
C. Mottled skin
D. Increased respiratory rate

An 18-year-old man college student was rushed fainted at the school. He


complained of severe quadrant. Upon palpation, he jerks even with
sample was obtained.
What is the most likely diagnosis of this patient?
A. Appendicitis
B. Liver Cirrhosis
C. Kidney stones
D. Duodenal ulcer

A 59-year-old women is admitted in the Medical her arms and legs. The
muscular strength progressively decreased within one year. She patterns
and has difficulty in swallowing.

Which of the following nursing diagnosis requires?


A. Risk of choking due to disturbed swallowing
B. Weakness and fatigue due to lower muscle
C. Disturbed breathing due to chest muscle
D. Disturbed activities of daily living due to

A man is to be discharged from the General appendectomy. The


precautionary measures, plans are discussed with him.
What is the most important desired outcome after?
A. Remain free of post-surgical complications
B. Report fever, redness or drainage from
C. Use pain management techniques approp
D. Resume gradual activities and avoid weight
A 38-year-old women is in the Neurosurgery frontal lobe of the brain
was surgical remove dressing is changed every 24 hours and she is
order. Which essential outcome reflects the progress of?
A. Absence of swelling and discharge at
B. Ability to swallow and presence of gauge
C. Stable vital signs and 96% oxygen at room
D. Alert, follows commands and speaks clearly

A 46-year-old patient is admitted in the Female back pain, which is


graded 6 on the scale of10. With the slight elevation in her blood
pressure. The patient has refused to eat lunch which is a low sodium
diet. Documented that patient is uncooperative and lunch.

What can be assessed by the nursing document patient?

A. Subjective judgement of patient's statement


B. Misunderstanding of patient's attitude
C. Understatement of communication
D. Unethical evaluation of reality

A 30-year-old man was brought to the hospital by ambulance he felt


down of a height of 10 meters. He was mechanically ventilated a
Glasgow Coma Score showed a level of six. The nurse is observe for
any changes in perceptual, sensory or cognitive

Which of the following is an expected patient's response at this time?


A. Slowly obeys commands
B. Exhibits no motor response
C. Reacts towards painful stimuli
D. Uses incomprehensible words
Which of the following variables is considered as a major factor that
affects community health?
A. Personal behavioural choices include socio-cultural factors
B. Number of health care providers in hospitals within the community
C. Quality of the public safety officers includes police officers,
firefighters
D. The number of recreational services in the community

A male client has received a prescription for orlistat for weight and
nutrition management. In addition to the medication, client states plans
to take a multivitamin.
What teaching should a nurse provide?
A. Be sure to take the multivitamin and the medication at least two-
hours
B. As a nutritional supplement, orlistat contains all the recommended
daily vitamins and minerals
C. Multivitamins are contraindicated during treatment with weight
control medications such as orlistat
D. Following a well-balanced diet is a much healthier approach to good
nutrition than depending on a multivitamin

A nurse who works in the surgical unit at one of the hospitals was asked
by the home health care nurse to make a home visit to a patient with
colostomy, who had been discharged the previous day in order to give
him a follow-up care and education.
Which of the following nurses should do the assigned task?
A. Critical care nurse
B. Psychiatric nurse
C. Surgical nurse
D. Community nurse
A clinical picture of an antisocial personality disorder is a "pervasive
pattern of disregard for and violation of rights of others, deceit, and
manipulation". What will be the priority of care?
A. Safety
B. Set limitations
C. Behaviours therapy
D. Reduction of environmental stimuli

A 2-day-old newborn is admitted to the nursery. While the nurse is


administrating oral feeding, the milk returns through the child's nose and
mouth and the infant become cyanotic. Which of the following condition
the newborn should have?
A. Anorectal malformation
B. Tracheoesophageal fistula
C. Cleft lip and palate
D. Cardiac condition

Rational:


With cleft lip and palate, the milk will return through the nose as there is
an opening in the roof of the mouth.
The nurse was educating a postpartum woman during discharge about
importance of breast feeding. Which of the following if said by the
women, indicates the need for further education?
A. Breast milk is nutritionally balanced
B. Breast milk reduces the risk of infection
C. Breast feeding promotes mother-child bonding
D. Breast feeding prevents pregnancy

During Cardiopulmonary Resuscitation (CPR) for a 75-year-old man in


the Emergency Department, the doctor introduced himself as a leader for
the CPR.
What is the most appropriate leadership style for this situation?
A. Autocratic
B. Democratic
C. Bureaucratic
D. Laissez-faire

The head nurse meets with staff nurses to discuss ways to improve
communication among shifts.
Which of the following statement best exemplifies the final stage of
conflict management?
A. "We need to clearly define the nature of the conflict"
B. " I will evaluate the outcomes of the strategies used monthly"
C. " Let us create a time line for the implementation of our strategies"
D. " I have to force you to follow the rules to resolve the issue"
In the Emergency Unit a nurse made an error that lead to an admission
order for the client to be on a venous thromboembolic protocol is not
processed. Two days after, a nurse notices the omitted order for heparin
5000 units subcutaneous every eight hours.
Which of the following statement best describes the appropriate follow-
up?
A. " I will contact the supervisor immediately about this error"
B. " I need to contact a physician and complete a variance report"
C. " I am too busy to complete a variance report. I'll do it next week"
D. " I am so glad I didn't make that mistake, that other nurse is going to
be in trouble"

A nurse manager was not happy about low results of patient survey.
Which of the following is the first step for the nurse manager?

A. Schedule meeting with staff


B. Start changes to improve
C. Review evaluation details
D. Report to director

Rational:
It is important to review the details of patients’ evaluations to determine
which areas of nursing care need improvements, as well as to be able to
discuss these points with the staff.

A 28-year-old women is accompanied by her mother to the Ward. She is


admitted for her affective disorders with anger recent events and
stammering spells. She has panic followed by apathy and listlessness.
How should the nurse manage the patient?
A. Be supportive and use therapeutic communication
B. Improvise sign language to control forgetfulness
C. Maintain calm environment and avoid argument
D. Anger management and speech therapy
A nurse manger is preparing and writing a plan for dealing disasters
(code red).
Which procedure is the top priority for the nurse manager the plan?

A. Infection control
B. Staff orientation
C. Patient education
D. Patient relocation

An 8-month-old infant is admitted with Hirschsprung disease.


Which of the following would be a significant finding in this infant?

A. Depressed anterior fontanel


B. Polyuria, hematuria
C. Weight gain, edema
D. Failure to thrive, constipation

A 5-year-old child was admitted with Nephrotic Syndrome. a nurse


noticed that the child has slight facial puffiness with mild pitting edema
on his hands and feet. There was no distended abdomen.
Which type of diet the nurse should order for the child?
A. High protein, high salt diet
B. Low protein, low fibre diet
C. Low protein, normal salt diet
D. Normal protein, low salt diet
A Community Nursing nurse is working with a family in their home.
The parents complain that their eight year-old son is "wild" and that he
never listens to them. They become upset at his antics to gain attention
from the nurse and send him out of the room.

Which of the following responses from the nurse is the most


appropriate?

A. “That was the right thing, it teaches him who is in charge"


B. “Don’t worry, he is a boy and with time, he will grow out of this"
C. “Let’s talk about how we can teach your child the right
behaviour and your expectations of him"
D. “You are only reinforcing his behaviour when you do that. Try
putting him in time out instead"

During a CPR of an admitted patient in cardiac arrest, a family member


tasks the unit nurse to be at the bedside and say final words to their
loved one. The nurse explains to the family member that the scene is
very disturbing and the medical team is doing its job. \the family
member still insists in witnessing the resuscitation efforts.
What would be the most appropriate action by the unit nurse?
A. Allow family member to be at bedside
B. Wait and ask permission from team leader
C. Call security to escort family member out
D. Refuse because there is not enough space in the room

A women patient with a body mass index of 35 is admitted to ward.


Which of the following nursing diagnosis, related to Imbalance the most
appropriate?
A. Activity intolerance
B. Less than body requirement
C. More than body requirement
D. Deficient knowledge on normal nutrition
What must be known on legal points of delegation when process to a
new nurse?
A. Evaluation of performance of delegate by clients
B. Actual time it takes to complete the task by delegat
C. Institution definition of the job description of
D. Number of times that the delegate has previously task

A 34-weeks-pregnant mother experiences a sudden gush o from her


vagina and mild uterine contractions. She informs about her condition
and requests if she could wait until the delivery.
Which of the following is the best desired response for report to the
hospital?
A. Intravenous fluids and medicines need to be administer
B. Observation is necessary to identify premature labor
C. Pain and fluid flow both need to be controlled
D. Fetal heart sound monitoring is necessary

Mr. A attended in outpatient clinic with symptoms of shortness of


breath, diarrhoea and severe respiratory distress
Which of the following is the best diagnosis of Mrs. A case?
A. Corona virus
B. Swine Flue
C. Zika virus
D. Hepatitis

A 28-year-old pregnant women at 9 weeks presents to the ob with


vaginal bleeding. During assessment, the nurse found fundal height is
12cm.
Which of the following is the most likely diagnosis?
A. Placenta previa
B. Abruptio placenta
C. Ectopic pregnancy
D. Hydatidiform mole
A 5-month-old boy has been vomiting green colored vomit He has
intermittent abdominal pain during which he draws his chest, turns pale
and cries forcefully. On observation, the in the stool which has a jelly-
like consistency. Abdominal pal along. Tube-like mass. There is no
fever, rash nor diarrhoea. Are hyperactive in all quadrants.
Which is the most likely form of initial treatment?
A. Manual manipulation
B. Surgical resection
C. Normal saline enema
D. Laparoscopy

In the hospital digital dashboard, what types of data displayed?


A. Staffing
B. Financial
C. Performance
D. Knowledge-based

A nurse is transferring a patient with chest tube drainage on an X-ray


department for chest X-ray.
Which location should the nurse place chest tube drainage at?

A. Directly on the stretcher in an upright position


B. On the side of stretcher next to patient
C. Hanged on IV pole that is attached to the stretcher
D. Attached to the stretcher and hanged below a mattress
A35 year-old patient was admitted to the Medical Ward through
Emergency Department accompanied by her mother. Her chief
complaints include sever epigastria pain, abdominal tenderness and
distension for the last 24 hours. She was anorexic and had passed six
watery diarrhoea since the last few hours. She was feeling lethargic due
to frequent elimination and body fluid loss. Her medical order sheet has
a list of procedures to be done.
What should the nurse do to reduce patient's feeling of exertion?

A. Explain that she will feel better soon


B. Show support and understanding
C. Encourage her to drink fluids
D. Help her rest undisturbed

The nurse is assigned to care for several patients in her/his shift. Who
should be assessed first by the nurse?

A. A diabetic patient who will be discharged and needs die


B. A patient with rheumatoid arthritis who has swelling be
C. An asthmatic patient who is due for bronchodilator med
D. A patient with surgical incision who required dressing

A 40-year-old women patient with Parkinsonism admitted to medical


ward. The patient stated that she has lost 5 kg of weight the past two
weeks. The nurse was planning to provide detain
Which type of diet is most suitable for Parkinsonism patient?

A. Solid
B. Liquid
C. Semi solid
D. Clear liquid
The nurse cares for a client who has undergone a tonsillectomy. The
nurse is most concerned about which post-operative finding?

A- Lack of appetite
B- Throat pain
C- Frequent swallowing
D- Nausea

Rationale

Primary hemorrhage, oropharyngeal bleeding, is bleeding that occurs


less than 6 hours after surgery.Signs of bleeding after a tonsillectomy
include:

Bright red blood coming from the mouth or nose


Tasting blood —a metallic tasteFrequent swallowing Spitting out bright
red blood Vomiting bright red or old blood
A tonsillectomy is the surgical removal of one's tonsils. Indications for a
tonsillectomy include chronic or recurrent infection of the tonsils
(tonsillitis), obstructive sleep apnea (OSA), and recurrent throat
infections (pharyngitis).
Post-tonsillectomy complications include hemorrhage, infection
(tonsillar bed or pneumonia), and pulmonary complications (i.e., pain
and nausea/vomiting, adhering to an appropriate diet, and monitoring for
bleeding and vital signs.

The nurse educates a client with myocarditis. How does the nurse
describe this condition?

A- Inflammation of heart muscle


B- Affects the outermost heart layer
C- Inflamed inner layer of the heart
D- Diseased heart valves
A nurse cares for a labouring client who receives epidural anaesthesia
for pain management. Which assessment data 30 minutes after
placement prompts the nurse to notify the anaesthesia provider?

A- Bladder distention
B- Temperature of 100.1° F (37.8° C)
C- Client unable to demonstrate pedal strength
D- Client report of shortness of breath

Rationale

Epidural anesthesia is regional anesthesia which provides labor relief by


the administration of local anesthetic, opioid medication, or both into the
epidural space. It can cause serious and potentially life-threatening
complications. Safe and effective management requires a coordinated
multidisciplinary approach. Respiratory depression can result if opioids
are used and the level of anesthesia moves higher than needed for labor
management.Side effects of epidural opioids may include nausea and
vomiting, pruritus, and delayed maternal depression. The possibility of
late respiratory depression exists for up to 24 hours after the
administration of an epidural opioid, depending on the duration of action
of the drug used. Fetal complications are the result of maternal
complications and are generally related to maternal hypotension or
respiratory distress. Urinary elimination issues are addressed during
labor by the placement of an indwelling Foley catheter if labor is
anticipated to last greater than eight hours.
The nurse reads this prescribed medication order on a client: "16 u
insulin glargine to be given subcutaneously once daily at 0800 hours."
Which action does the nurse take?

A- Call the health care provider to clarify the administration time.

B- Ask another nurse to read the prescription and cosign.

C- Call the health care provider to rewrite the prescription.

D- Prepare to administer the insulin glargine as prescribed.

Rationale

The use of "u" for unit is on the Joint Commission's official list of "do
not use" abbreviations."U" could be mistaken for 4, O, or cc. Other
unapproved abbreviations include IU, QD and QOD or any version of it,
trailing zeros or lack of leading zeros, MS, which can mean magnesium
sulfate or morphine sulfate, and MSO4 and MgSO4 which are easily
confused for one another.
Although insulin glargine (Lantus) is often given at night, it has a 24
hour action time and may be given at any time of day as long as it is the
same time each day.
The nurse cares for a client in the post anesthesia care unit (PACU) after
general anesthesia was given for an open small-bowel surgery. The
nurse determines which nursing diagnosis is the most important when
caring for the client?

A- Impaired transfer ability related to sedation as evidenced by


drowsiness
B- Nausea related to manipulation of intestines as evidenced by
client report
C- Risk for infection related to recent open abdominal surgery
D- Acute pain related to surgery as evidenced by pain score of 4 out of
10

Rationale
The most important nursing diagnosis is one that presents an active
threat to the client's physiological status. Current nausea and the
associated immediate risks take priority over impaired transfer ability,
risk of infection, and mild acute pain. When treating multiple problems,
the nurse should prioritize and address the most urgent and active
problems before addressing other issues as a means of promoting safety
and physiological integrity

A nurse provides care for a client who is 5 days post gastrectomy. The
provider prescribes total parenteral nutrition (TPN). Which statement by
the nurse to an orienting nurse explains the client's need for TPN?

A- TPN is used for clients who are at increased risk for aspiration due to
swallowing inability.
B-TPN is for clients who are unable to receive nutrients by mouth
for an extended period of time.
C- TPN is the usual method of providing nutrition to clients after major
abdominal surgery.
D- TPN is a temporary solution and can be used for the first 24 hours
after surgery.
A nurse admits a client sent from the health care provider’s office. The
nurse calls the provider to obtain prescriptions. Which action does the
nurse take?

A- Hold treatments until the provider signs the prescriptions received.


B- Read each prescription back to the provider prior to ending the
call.
C- Have the provider provide the prescriptions to another nurse as well.
D- Verify the prescriptions with the charge nurse before beginning
treatment.

The nurse reviews a client’s ECG and notes P waves. With regard to the
cardiac cycle, what does the P wave indicate?

A- Ventricular depolarization

B- Atrial depolarization

C- Atrial repolarization

D- Ventricular repolarization

A client is orally intubated and mechanically ventilated due to


respiratory failure. Which item does the nurse ensure is kept next to the
client’s bed at all times?

A- Tracheostomy kit
B- Bag valve mask device
C- Arterial blood gas kit
D- An endotracheal tube
In the process of delegation, the nurse understands that the primary
person who maintains accountability for the accuracy, safety, and
completion of the task delegated remains with whom?
A- the delegator of the task
B- the delegatee the task
C- those who assisted with task completion
D- the person who documented task completion

Rationale:
The person who maintains accountability for the accuracy, safety, and completion of delegated
tasks is the delegator. Although all personnel are responsible for their own actions, the primary
person is the person who delegates the activity. .
•Test-Taking Strategy: Note the strategic word “primary.” Eliminate options 2, 3, and 4 because
they are comparable or alike in that they imply that those who take part in task completion and
documentation are primarily accountable. .

physician has written an order for a preoperative client to have “enemas


until clear.” The nurse has administered three enemas, and the client is
still passing brown liquid stool.
Which of the following actions should the nurse take next?

A- Wait 30 minutes, check the client’s electrolyte levels, and then administer
another
B- Continue to administer the enemas until the stool is clear
C- Encourage the client to drink clear liquids and administer another enema in 1
hour
D-Notify the registered nurse (RN)
Rationale:
Up to three enemas may be given when there is an order for enemas until
clear. If more than three are necessary, the nurse notifies the RN who will then call
the
physician (or act based on agency policy). Excessive enemas could cause fluid and
electrolyte depletion. Options 1, 2, and 3 are incorrect for these reasons.
Test-Taking Strategy: Use the process of elimination. Eliminate options 1, 2, and 3
because they are similar. Also, use knowledge of basic bowel elimination
procedures and consider the physiological effects that can occur with enema
administration. This will
assist in directing you to the correct option. Review the procedure for
administering enemas if you had difficulty with this question.
A nurse diagnosis a patient with readiness for the…
This diagnosis is classified within which of the nursing diagnoses?

A. Acute nursing diagnoses


B. Risk nursing diagnoses
C. Wellness nursing diagnoses
D. Possible nursing diagnoses

A 17 year-old arrived to the Emergency Room complaining abdominal


pain on right lower quadrant. Pain was rated as 9 numeric scale with
positive rebound tenderness over the pain
Blood pressure Heart rate Respiratory rate Temperature
120/70 mmHg 95 /min
20 /min 39.2
Which of the following interventions has the highest priority?

A Secure an IV access

B- Keep NPO

C- Prepare for ultrasound

D- Prepare for abdominal surgery


An 18-year-old man college student was rushed fainted at the school. He
complained of severe quadrant. Upon palpation, he jerks even with
sample was obtained.
What is the most likely diagnosis of this patient?
A. Appendicitis
B. Liver Cirrhosis
C. Kidney stones
D. Duodenal ulcer

A 2-day-old newborn is admitted to the nursery. While the nurse is


administrating oral feeding, the milk returns through the child's nose and
mouth and the infant become cyanotic.
Which of the following condition the newborn should have?

A. Anorectal malformation
B. Tracheoesophageal fistula
C. Cleft lip and palate

60-year-old women is admitted to the following a neck dissection for


laryngeal cancer nodes and the sternocleidomastoid muscle, Which of
the following is the best post-operative?
A- Risk for infection
B- Impaired nutrition
C- Ineffective airway clearance
D-Impaired verbal communication
patient is being admitted to the Recovery Room following a
thyroidectomy. The back of the neck wound is covered with dressing.
During the first 15 minutes, the patient started working and having
diarrhoea. A general assessment is performed with special attention
given for the high risk for hemorrhage.Where would bleeding most
likely occur?
A. Stool
B. Vomitus
C. Dressing
D. Back of neck

49-year-old women presented to the Emergency Department complaint


of severe chest pain. The ECG showed that the patient myocardial
infarction. The doctor ordered the nurse to give the 800 mg of aspirin.
What is the primary indication of aspirin in this case?
A. Breaks down the thrombus
B. Decreases the formation of platelet plugs
C. Inhibits the conversion of prothrombin to
D. Interferes with vitamin k to maintain

A newly nursing director assigned to a hospital. That is the first should


he do?
A. Evaluates the staff
B. Change the head nurses
C. Change the roles
D. Nothing to do

patient in a long-term care facility is in a (coma) with a tight contracture


of the right arm Which of the following is the best goal over the patient
related to the nursing diagnosis of

A. Develop no further contracture


B. Wear an arm and hand splint
C. Have the contracture resolved
D. Have no pain related to the contracture
A 1-year-old girl admitted to paediatric medical unit significant weight
loss, diminished mid-arm circumference diarrhoea, and red hair. Which
of the following type of malnutrition do the nurse suspect
A. Marasmus
B. Spitting up
C. Kwashiorkor
D. Rickets

A client who is breastfeeding her newborn requests assistance from the


lactation nurse. Which reflex does the nurse explain in order to assist
with latching on?

A- Extrusion reflex
B- Rooting reflex
C- Swallowing reflex
D- Tonic neck reflex

A nurse is completing the preoperative procedure during the nurse


checking she noticed the patient is waring Jewellery. What is the most
appropriate action?

A. Give ring to security office

B. Lock ring with patient's valuables

C. Call patient's family to give them the ring

In the hospital digital dashboard, what types of data displayed?

A. Staffing
B. Financial
C. Performance
D. Knowledge-based
A primigravida client comes to the clinic for her 36-week checkup. The
nurse prepares for which test that is done at every prenatal visit?

A- Measure of capillary blood glucose


B- Fetal heart rate using Doppler
C- Fetal ultrasound testing
D- Laboratory draw to assess for anemia

Rationale

Prenatal visits are scheduled every four weeks from confirmation of pregnancy
until 28 weeks, every two weeks from 28 to 36 weeks, then weekly until delivery.
Prenatal visits include measurement of maternal blood pressure and weight and
urine screening for protein. Fetal heart tones and fetal growth, using fundal height,
are measured and then compared to previous measurements as well as the
estimated date of confinement.
Assessment of fetal activity by maternal observation is also recorded. Additional
screening occurs at specific points during the pregnancy, such as testing for
gestational diabetes, neural tube defects, and sexually transmitted infections.
A postpartum nurse provides discharge instructions to the parents of a
newborn after a Gomco circumcision. Which statement by the father
indicates the need for additional instruction?
A- “We will place gauze with petroleum jelly over the tip of the penis
after every diaper change until it is healed.”
B- “When a yellow scab forms over the penis after a few days, we
will remove it so that continued healing may occur.”
C- “We should only use warm water to cleanse the diaper area until the
penis is fully healed.”
D- “It may be normal to see a small amount of blood on the gauze when
changing the diaper.”

A nurse cares for a client with an epidural catheter for labor pain
management and requests additional medication. A student registered
nurse anesthetist (SRNA) responds and administers medication via the
epidural catheter. Which action performed by the SRNA requires the
nurse to stop administration of medication?

A- Aspirating bloody fluid into a syringe


B- Obtaining a negative aspiration into a syringe
C- Cleansing the the port with alcohol for 15 seconds
D- Assessing the client’s blood pressure
A nurse cares for a client who reports sudden mid-sternal chest pain.
What nursing intervention takes priority?

A- Ask the client to describe the pain.

B- Place the client in a position of comfort.

C- Obtain the client’s vital signs.

D- Review the client’s past medical history.

Rationale

When establishing care in a life-threatening situation, the nurse must


remember airway, breathing, and circulation. Obtaining the vital signs
takes priority over obtaining the client’s past medical history, assessing
the client’s pain, and positioning the client in a comfortable position.

An older adult client with renal failure comes to the emergency


department with a report of nausea and vomiting. The client's heart rate
is 45 beats/min. The nurse is most concerned about which medication
that the client takes?

A- Nitroglycerin
B- Digoxin
C- Doxorubicin
D- Furosemide
Rationale

Digoxin is used to treat heart failure and atrial fibrillation. It has a low therapeutic index and
requires regular blood level monitoring to ensure levels are appropriate. Because this client has
renal failure, the risk for toxicity is higher.
Toxicity can occur with drug noncompliance and in clients with renal dysfunction because the
primary route of excretion is through the kidneys.
The nurse educates the family of a client who is experiencing delirium.
Which statement accurately describes delirium?
A- "Delirium is a condition that develops over months."

B- "Delirium indicates the client is developing dementia."

C- "Delirium is a condition that is only seen in older adults."

D- "Delirium symptoms are often worse in the evening and night."

Rationale

Delirium is an acute mental disorder that can occur among hospitalized clients. It often presents
within the first 48 to 72 hours of admission. Delirium is characterized by confusion,
disorientation, and restlessness. The condition often reverses when the underlying physical
condition is treated

A nurse cares for a client who is postoperative for a spleen repair. What
is the nurse’s priority action?

A- Examine the overall trend of vital signs.


B- Review the client’s most recent lab results.
C- Perform a physical assessment of the client.
D- Update the client’s family regarding the client’s condition.

Rationale

The nurse’s priority is assessment of the client. In order to determine the


priority of care, the nurse must first assess. Notification of the client’s
family and review of labs and vital signs occur after the client has been
assessed.
A client who is active duty military develops raised, hard pustules on the
mouth, face, and extremities after a two-day history of high fever,
malaise, and body aches. The nurse suspects the client's symptoms may
indicate the presence of which disease?

A- Smallpox

B- Shingles

C- Chickenpox

D- Rubella

Rationale

The first symptoms of smallpox include high fever (e.g., 101–104 °F


[38.3–40 °C]), malaise, head and body aches, and sometimes vomiting.
Smallpox may be spread through prolonged face-to-face contact, contact
with infected bodily fluids, or contact with contaminated bedding. A
person becomes most contagious with the onset of a rash, which starts in
the mouth and face and spreads, first as bumps, then pustules and scabs.
The infected person is contagious until the last smallpox scab falls off.
Standard, contact, and respiratory precautions are required to prevent
spread of the disease.
The nurse assesses a client admitted to the hospital after sustaining
multiple burns. The client’s arm is white, leathery, and does not blanch
or cause pain. Which depth of injury does the nurse describe to the
health care provider?

A- Full thickness burn


B- Deep partial thickness burn
C- Superficial partial thickness burn
D- Superficial burn

Rationale

Superficial burns involve only the epidermal layer of skin and do not blister but are painful, dry,
red, and blanch with pressure. Partial thickness burns include two subcategories: those involving
the epidermis (superficial) and those involving portions of the dermis (deep). Superficial partial
thickness burns form blisters within 24 hours and are very painful, red, and weeping but can still
blanch with pressure. Deep partial thickness burns extend into the deeper dermis and can be very
painful or painful only to pressure. They appear red and waxy white with wet blisters, and they
blanch slowly, if at all. Full thickness burns destroy all layers of the dermis and may involve
subcutaneous tissue. These burns do not hurt. The skin appears white or gray or even blackened.
No blisters will develop. Pale full thickness burns may appear like normal skin, but there is no
blanching and the skin is no longer elastic due to the damage. Deep full thickness burns are
black, dry, and painless. They are potentially life-threatening, extending through the skin into
fascia, muscle, and or bone.
The nurse cares for a client with full-thickness burns and assesses a
blood pressure of 85/55 mmHg, pulse of 109 beats/min., and respiration
rate of 22 breaths/min. Which action does the nurse take first?

A- Increase the client's IV fluid volume rate.


B- Request a prescription for vasopressors.
C- Place the client in the reverse Trendelenburg position.
D- Assess the client's current urine output volume.

Rationale

The client with full thickness burns is at high risk for fluid imbalance
due to third spacing or capillary leak syndrome. This results from leak of
plasma from the vascular space into the interstitial space. The nurse
should gather all relevant data related to this complication to provide to
the health care provider to ensure the best prescription to treat the
hypotension. Other relevant data would include current sodium,
potassium, albumin, and haematocrit levels.

A client with chronic obstructive pulmonary disease (COPD) requires


high-flow oxygen supplementation, or precise oxygen concentration, but
does not require ventilatory support. Which device does the nurse
recognize as appropriate for this client?

A- Aerosol mask
B- Bilevel positive airway pressure mask
C- Venturi mask
D- Nasal cannula

Rationale

The Venturi mask is the most accurate, noninvasive oxygen delivery system.
Clients at risk for CO2 retention, such as clients with COPD, benefit from a
Venturi mask because the FiO2 can be controlled. A nasal cannula is a low-flow
delivery system that does not provide precise FiO2 concentrations. The aerosol
mask is most often used for high humidity or for the use of medication delivery. It
does not provide high-flow oxygen or allow for precise control of FiO2
concentration. A bilevel positive airway pressure (BiPAP) mask is used when
additional pressure support is needed with ventilation.
A client presents to the emergency department after hitting his head on
the floor during a basketball game. The nurse obtains a Glasgow Coma
Score (GCS) as follows:

Best Eye Opening = 3


Best Verbal Response = 4
Best Motor Response = 6
Total GCS = 13

How does the nurse interpret these findings?

A- Opens eyes to pain, no verbal response, and flexes arm to pain.


B- Opens eyes to verbal command, confused, and obeys commands.
C- Spontaneous eye opening, oriented, and obeys commands.
D- No eye opening, confused, and no motor response.

Rationale

The Glasgow Coma Scale (GCS) is a standardized tool used to assess


the degree of neurologic impairment. A score of 15 indicates that a client
is awake and oriented, while a score of 8 or below means a client is
considered comatose. It is composed of three subscales including Eye
Opening, Best Verbal Response, and Best Motor Response. When the
client opens their eyes to verbal command, they are given 3 points for
Eye Opening. When the client displays confusion, they are given 4
points for Best Verbal Response. When the client obeys verbal
commands, they are given 6 points for Best Motor Response. This GCS
example totals 13, indicating minor impairment
A multiparous woman is admitted to the postpartum ward after vaginal
delivery. Assessment showed lochia: steady trickle of bright red blood,
Fundus: firm; Temperature: 37.5C

Which of the following is the most likely diagnosis?


A-Endometriosis
B-Uterine atony
C-Vulvar hematoma
D-Laceration of the genital tract

A 33-year old man presents to the emergency department with high


grade fever, tachycardia, and tachypnea.
What is an appropriate nursing intervention for the patient’s fever?
A-provide dry clothing
B-keep limbs close to the body
C-cover the patient’s scalp with a cap
D-measure the patient’s fluid intake and output
The community health nurse is designing a primary prevention
program for school-age children. What should the nurse include in this
program?

A-Importance of having dental check up every 6 months


B-Need to wear protective helmets and padding when using bicycles
C-Seeing the paediatrician for routine physical examination
D-How to preform breast and testicular self-examination
Rational:
The use of protective helmets and padding when using bicycles is considered
primary prevention activities. Primary prevention focuses on general health
promotion and prevention of injuries.

An adolescent male is diagnosed with a sexually transmitted infection.


The client asks if the visit can be kept private or if the parents need to be
informed of the infection. What response by the nurse is best?

A-“As you are a minor your parents should be notified”


B-“We will notify your parents only if you want them to know”
C-“There is no other way for you to get the needed medication”
D-“The diagnosis is confidential however the infection will be
reported”
Rational:
To promote access to STI testing and treatment all states allow minors to consent to STI services
and no state requires that physicians notify parents about the provided services. In some cases,
states allow physicians to inform the parents if this is deemed to be in the best interest of the
minor, however notifying the parents never a requirement. All STIs should be reported to the
Centres for Disease Control and Prevention.
Which of the following statements indicate that a nursing staff need an
education session related to confidentiality and information security?
A- giving the password to patient’s family member.

B- leaving the computer in the nursing station secured.


C-leaving the personal electronic password safe.
D-allowing nursing students to read medical record to their patients

A 4-day- old boy is diagnosed with physiological jaundice. His mother


is distressed and wants to know why he is having this condition. What
should the nurse tell the mother about the most prominent cause of
physiological jaundice?
A-Immature hepatic function
B-Decreased milk intake
C-Rh incompatibility
D-Red cell enzyme defects

A mother brought her 6-month-old healthy infant for a well-child check-


up. Which immunization should the nurse anticipate to administer as per
immunization schedule recommendation?
A-Measles, Mumps, Rubella.
B-Varicella, Measles, Hepatitis b
C- Hepatitis b, Rotavirus, Diphtheria

D- Diphtheria, Tetanus, Pertussis


A nurse is preparing a sterile field prior the
Which of the following procedures illustrates the nurse to the principles
of aseptic technique?
A- Touch the outer 2-inch border of the sterile gloves
B- Place the sterile linen-wrapped package on waist level
C- Hold the bottle lip 1-2 inches above sterile pouring the solution
D- Open sterile item while holding outside hand then throw object onto
sterile field

The nurse is caring for a patient with Parkinson’s disease. Which of the
following is an expected outcome of constipation related to diminished
motor function, inactivity, and medications?

A- patient will use laxative every other day


B- patient will have a soft bowel movement daily
C- patient will report minimal pain with bowel movement
D- patient will limit the intake of complex carbohydrates

A nurse is caring for a client with head injury and monitoring him for
decerebrate posturing. Which of the following is the characteristics of
this posture?

A- flexion of the extremities after stimulus


B- extension of the extremities after a stimulus
C- upper extremity flexion with lower extremity flexion
D- upper extremity flexion with lower extremity extension
A 26 years old man is admitted to the male surgical ward with acute
appendicitis. Which of the following assessments is appropriate for this
case immediately after admission?

A- initial assessment
B- emergency assessment
C- time- lapsed reassessment
D- problem focused assessment

A 62 years old diabetic type 2 patient is in the post-operative care unit


following amputation of the right foot. Post-operatively the patient tells
the nurse tingling, coldness and cramping in the right foot. Which of the
following describes best the type of pain?
A- visceral
B- somatic
C-ischemic
D- neuropathic

A 35 years old man with pulmonary edema is admitted to the intensive


care unit. He is on mechanical ventilation (MV) without sedation. After
the patient is placed on the MV, the nurse diagnosed the case as
impaired oral communication related to intubation and MV. What is the
best method for the nurse to communicate with the patient?

A- use normal verbal communication


B- communicate with patient after MV removal
C- use non-verbal communication such as head-nodding and writing
D- communicating with the patient’s family will be much more
effective
A home health nurse visits a patient who is newly diagnosed with
diabetes. The patient is compliant with taking the prescribed
hypoglycemic medications and eats three meals a day followed by
desserts sweetened with granulated sugar. The patient also exercises 30
minutes a day, three times a week. (See lab results)

Test Result Normal


Glucose Range from 6.6-8.3 3.9-5.5mmol/L
HgbA1c 6.9% 4.7-5.6%

Which of the following educational intervention takes priority?


A- glucoses Monitoring
B- dietary requirements
C- exercise regimen
D- medications

A 48 years old man with pneumonia is diaphoretic, tachycardic and


confused. The cardiac monitor indicates sinus tachycardia with frequent
premature multifocal ventricular beats (see lab results).

Test. Result. Normal


Abg HCO3 22 22-28mmol/L
Abg PCO2 8.7 4.7-6.0 kPa
Ph 7.26 7.35-7.45

Which intervention is the most important for the nurse to include in the
plan of care?

A-obtain a 12 lead electrocardiogram daily


B- assess lungs for increasing pulmonary secretions
C- continuously monitor respiratory rate and depth
D- maintain a patent IV catheter for antibiotic therapy
A 28 years old patient in the medical ward was admitted to rule out her
recurrent headaches for the last one year. On the second day of her
admission, she complained of severe generalized headache. After
identifying the intensity, duration and severity, the nurse informed the
physician and administered paracetamol and recorded the details in the
nursing notes. After half an hour when the nurse asked, the patient
replied that her pain was relieved and she felt better. What area of the
nursing process is exercised when the nurse inquired about the pain after
half an hour?

A- nursing problem
B- implementation
C- assessment
D- evaluation

A 29 years old woman patient was brought to the outpatient for the
removal of stitches on her left check, which was treated nine days back
after being involved in road traffic accident. She covers her face
completely and asked to be seen by a female doctor. The site of the
wound was red, swollen and some pussy points were visible. She states
that she did not wash her face since her accident and kept her face
covered all the time as she did not want anyone to see it. Which of the
following should the nurse do to elevate the patient’s self-esteem?

A- involve her husband in the care


B- explain the stitches marks will heal soon
C- ask her to wash her face and wear light makeup
D- discuss that it is normal to feel low temporarily
A patient comes to the emergency unit with cough and severe dyspnea.
The patient’s medical history revealed a diagnosis of chronic heart
failure and chronic obstructive pulmonary disease.

Blood pressure 110/70 mmhg


Heart rate 87/min
Respiratory rate 23/min
Temperature 37.3 C

Which of the following diagnostic tests will be most beneficial to a


nurse to figure out if there is an exacerbation of heart failure?

A- B-type natriuretic peptide (BNP)


B- arterial blood gas (ABG)
C- cardiac enzymes (CK-MB)
D- chest x-ray

A nurse is preparing to administer oxygen 4L/min via nasal cannula to a


patient complaining of shortness of breath. Which of the following
actions will protect the mucosal layer of the respiratory tract?

A- check for bubbling in the humidifier


B- change the position of the patient frequently
C- stop oxygen after oxygen saturation reaches 98%
D- give oxygen as long as needed as there is no side effects
A 29 years old patient in the gynecological ward had delivered a baby
boy through caesarian section 24 hours ago. Her vital signs are stable
and she is on routine pain medication and anti-coagulant therapy. She is
unwilling to get out of bed as she has a history of giddiness and fall in
her previous delivery. The attending nurse explained the importance of
mobility and agreed to wait until the patient is ready. How should the
patient be encouraged to get out of bed?

A- two nurses hold her from sides while she stands


B- help her set at the edge of bed with the legs down
C- deny providing bed pan and ask to use the toilet
D- explain the harmful effects of immobility

An 18 years old woman college student was rushed to the hospital after
she had fainted at the college. She complained of severe pain in the right
lower abdominal quadrant. Palpations shows guarding and
hypersensitivity with slight touch. A blood sample was obtained and sent
to the laboratory for analysis (see lab results)

Test Result Normal


Hb 145 142-176g/L
WBC 19.8 4-10.5 x 10^9/L
RBC 4.0 3.8-5.1x 10^ 12/L

Which diagnosis is the most likely?


A- Appendicitis
B- liver cirrhosis
C- kidney stones
D- duodenal ulcer

In a post-operative patient, which of the following is most indicative of


developing complication?

A- increasing alertness
B- weak and rapid pulse
C- negative homans’ sign
D- minimal bowel sounds in four quadrants
A 45 years old with severe upper GI bleeding is admitted to the medical
ward . The doctor orders the nurse to give the patient blood transfusion.
Which peripheral IV catheter gauge is best for this procedure?

A- 14 gauge
B- 16 gauge
C- 20 gauge
D- 24 gauge

A patient comes to the emergency department with extreme dyspnea,


orthopnea, anxiety and complains of feeling panicky. The patient is
coughing up white frothy sputum and is cyanotic with profuse
perspiration. Inspiratory and expiratory wheezing and bubbling sounds
are auscultated. The patient is diagnosed with acute pulmonary edema.
Which of the following should the nurse do first?

A- assess oxygen saturation rate


B- identify precipitating factors and underlying conditions
C- administer morphine ( Duramorph) to reduce anxiety
D- administer digoxin (Lanoxin) to decrease fluid backing up into the
lungs

A 55 years old man is receiving total parenteral nutrition (TPN) . The


nurse documented a risk for infection (sepsis) in the nursing care plan.
What is the most likely indicator for monitoring sepsis?

A- central venous pressure


B- white blood cell count
C- intake and output
D- electrolyte level
A nurse is working in the medical unit and has been assigned to care for
a 65 years old man with cerebrovascular accident, which caused right-
sided hemiparesis. The nurse is worried about the patient developing
contractures due to immobility. Which of the following nursing actions
is most important to be implemented?

A- provide diet rich in protein


B- turning the patient every two hours
C- provide range of motion exercises
D- apply moisturizing cream on the affected side

A registered nurse in the surgical ward is assigned to care for five


patients who were scheduled for different abdominal surgeries. She
developed nursing care plans for them with the list of nursing diagnosis
that was prioritized according to each of their needs. She was able to
implement the plans and completed the required tasks by the end of the
shift. Which of the following is the best expression of the nursing
process that is used here?

A- implementation of pre-operative protocols


B- provision of individualized patient care
C- implementation of quality standards
D- promotion of patients’ autonomy

After receiving the initial treatment of tuberculosis, a 27 years old


woman in the chest diseases ward was given discharge instructions on
dietary management, medication regimen, hygiene care and follow up
visits. What discharge instructions need to be emphasized the most?

A- increase protein diet to promote healing


B- more rest and relaxation to restore energy
C- maintain daily exercise regimen to improve health
D- dispose sputum as guided to avoid spread of disease
A 45 years old woman patient in the outpatient department has come
with severe throbbing headache at the crown site since last night. She is
a known hypertensive for the past five years but takes her anti-
hypertensive medicines only when she has headaches and not on daily
basis. She took her medicine twice since last night. Her vital signs show
mild elevation in her blood pressure and she is admitted to rule out the
cause of her headache after initial investigations. Which of the following
nursing intervention is required at first?

A- document information that is gathered


B- arrange for investigations as ordered
C- prepare for physical examination
D- promote rest and relaxation

Which client should be assessed first after the nurse receives a change of
shift report?

A- one day post-operative client with serosanguinous drainage on an


abdominal surgical dressing and a body temperature of 38C

B- two days post-operative client receiving intermittent epidural bolus


analgesia who now reports incisional pain as a 4 on a 0-10 scale

C- one day post-operative client who is to ambulate for the first time this
morning with fine inspiratory crackles over the lungs upon auscultation

D- two days post-operative client receiving fluids at 125ml/hr with a


Foley catheter and urine output of 100ml over the previous 8 hours
A 55 years old man is transferred to the day care unit after the
colonoscopy procedure. He complains of abdominal pain and discomfort
due to gaseous feelings. He is diagnosed as having irritable bowel
syndrome and is instructed to adjust his lifestyle. What is the cause of
the abdominal discomfort after colonoscopy?

A- residual gasses from the procedure


B- irritation due to colonoscope insertion
C- food and fluid ingested before procedure
D- medications administered During procedure

A 55 years old underwent total knee replacement. Two hours post-


operatively, an orthopedic unit nurse notes the patient was lethargic and
dizzy (see lab results)

Test Result Normal


Hb 9.8 130-170g/l (men)
120-160g/l(women)

What nursing intervention is the most appropriate?

A- start an IV fluid
B- administer paracetamol 1 gm IV
C- call blood bank and request one unit of packed RBCs
D- encourage ambulation to reduce enhance recovery
A 52 years old man was working in tall grass when a snake bit him. An
ambulance arrived at the scene 20 minutes later. They found the man
lying on the ground with cold and clammy skin. He was having
difficulty breathing and the right ankle was swollen. He complained of
double vision, feeling weak and itching skin. He reported that this was
his second snakebite. The paramedics prepare to administer
intramuscular epinephrine and place a tourniquet.

Blood pressure 86/48 mmHg


Pulse rate 130/min
Respiratory rate 28/min

Which of the following nursing diagnosis has highest priority?

A- disturbed sensory perception related to effects of pruritus


B- ineffective breathing pattern related to bronchospasm
C- decrease cardiac output related to systemic vasoconstriction
D- risk for injury related to drug treatment and adverse effects

A patient visits the clinic for the first time. In order to perform an
accurate and complete assessment, which of the following will be the
nurse’s first step?

A- obtain a complete history


B- obtain a blood pressure
C- preform a review of systems
D- obtain a temperature, pulse, and respiration
A 13 years old patient is admitted for diarrhea and vomiting. He looks
pale and lethargic. The nurse is preparing to give IV hypotonic solution.

Blood pressure 110/70mmHg


Heart rate 76/min
Respiratory rate 18/min
Temperature 36.1C

Which IV solution is most appropriate?

A- 0.9% normal saline


B- lactated ringers
C- 10% dextrose in water
D- 0.45% normal saline

A 60 years old patient was admitted with hepatic coma in the intensive
care unit. The physician has ordered protein restriction diet for the
patient. Which of the following substances is most likely causes harmful
effects when the patient increases protein intake?

A- urea
B- creatinine
C- ammonia
D- amino acid

A nurse calls a physician regarding a change in clients’ condition. A


physician gives orders over the telephone for STAT arterial blood
gasses. Which intervention should the nurse implement?

A- write down the order for ABGs immediately


B- call respiratory therapist stat to draw ABGs
C- write down the order and read it back to the physician
D- give the order stat to the health unit coordinator to input in computer
A patient complains of pain when standing upright and has developed a
hump on the upper back. In the past year, the patient has become slightly
shorter. The doctor has suggested tests to rule out osteoporosis. What is
the best appropriate intervention?

A- instruct the patient in the use of prescribed calcium and magnesium


supplements
B- have the patient sleep propped on two pillows at night to decrease
pain
C- prepare the patient for CT scan of both hips
D- instruct the patient in the use of vitamins A, C and E

An adolescent with juvenile diabetes mellitus develops chronic renal


failure. Which of the following diets is suitable?

A- low fat
B- low mineral
C- low protein
D- low carbohydrate

A discharge order was written for a 49 years old patient diagnosed with
hepatitis C after his condition got stable and he was given the advice to
take his medications regularly. Which of the following should be the
nurse’s primary focus while teaching the patient?

A- family support
B- dietary counseling
C- activity and exercise
D- understanding of the disease
A 66 years old woman is admitted in the orthopedic ward for the partial
hip replacement surgery. She has osteoarthritis for the last 25 years and
she is experiencing pain progressively restrictive mobility despite having
some physiotherapy sessions. What needs to be ensured as the most
important part of pre-operative nursing care?

A- stability of vital signs and hydration status


B- pain management and psychological support
C- readiness and understanding of this procedure
D- review chart and verification of the surgical site.

A nurse notes that the drug ordered for a patient is higher than the
recommended dose. The nurse calls the physician to clarify the order,
but the physician instructs the nurse to administer the dose as ordered.
Which of the following actions should the nurse take?

A- call medical director


B- call nursing supervisor
C- call pharmacy department
D- administer drug as ordered

A patient is admitted to the emergency department with a T3 spinal cord


injury. The skin of the patient is warm, dry and pink. Neurogenic shock
is suspected.

Blood pressure 70/50 mmHg


Heart rate 58/min
Respiratory rate 17/min

Which of the following is the most appropriate initial action?

A- cranial nerves assessment


B- isotonic fluids intravenously
C- perform Glasgow coma scale monitoring
D- place in modified Trendelenburg’s position
A 25 years old woman was admitted to the medical ward for anorexia
after chemotherapy. The nursing diagnosis was “imbalanced nutrition:
less than body requirements related to dysfunctional eating patterns”
which of the following is the best way to evaluate the outcome of
nursing care of this patient?

A- record daily weight


B- monitor vital signs accurately
C- schedule meals with family members
D- offer small portions of favorite foods

One month after starting new medications for hypertension, a patient


returns to the clinic with blood pressure in the acceptable range. The
patient admits to taking the medications when “feeling bad”. Which of
the following actions should the nurse take?

A-add a nursing diagnosis of non-compliance


B- assess further to determine the reason for the patient’s actions
C- re-evaluate the need for daily medications since the blood pressure is
acceptable
D- re-educate the patient about the importance of following the
medication plan

A patient receives a blood transfusion for severe anemia after surgery.


While evaluating the patient, the nurse finds that the patient’s oral
temperature has begun to rise from 36.8C to 38.3 C. Which of the
following should the nurse do?

A- discontinue the intravenous line and restart at another site


B- use a blood cooling device to cool the blood as it infuses
C- give the patient antipyretic medication and continue the transfusion
as ordered
D- stop the transfusion, keep the vein open with normal saline, and
notify the doctor immediately
A 3 years old child is admitted to the postoperative care unit following a
heart transplant. The nurse administers cyclosporine by intravenous
infusion. Fifteen minutes later the child has difficulty breathing, his skin
feels cool and clammy and he appears restless. Which of the following is
the most appropriate initial nursing action?

A- ensure airway patency


B- administer oxygen therapy
C- discontinue intravenous infusion
D- administer intramuscular epinephrine

A 5 years old child was brought to the emergency room with a fractured
right forearm. He had several bruises on his body but showed no signs of
pain while palpating them. He seemed scared and did not answer any
questions asked. Which of the following protocols should be
implemented here?

A- check vitals
B- develop care plan
C- physical assessment
D- immobilize right arm

A nurse is providing a health teaching to a client about the proper use of


a cane during ambulation. The nurse could include which of the
following instructions about the correct use of cane?

A- patient moves the stronger leg forward to the cane


B- patient holds the cane of the strong side of the body
C- patient leans on the cane to provide support to the weaker side
D- patient moves the cane forward about 4 inches to the stronger side
A 45 years old obese woman with cholelithiasis present to the
emergency room complaining of nausea and vomiting for two days,
along with sever continuous upper abdominal pain that radiates into the
back mid abdominal pain which is aggravated by eating. The abdomen is
swollen and tender. She has a low-grade fever and the ER physician
finds that she has a slightly elevated WBC (12000) and an elevated
serum amylase. Which of the following is the most likely diagnosis?

A- acute pancreatitis
B- peptic ulcer disease
C- early phase of acute appendicitis
D- ruptured abdominal aortic aneurysm

What is the most appropriate blood product for a patient with massive
hemorrhage to upper GI bleeding?

A- plasma
B- whole blood
C- packed red cells
D- serum albumin

A 5 years old child brought to the emergency room with fractured right
forearm. He had several bruises on his body but showed no signs of pain
while palpating them. He seemed scared and did not answer any
questions asked. Which of the following actions should the nurse do to
gain the child’s trust?

A- arrange meal
B- comfort measures
C- show the playing area
D- therapeutic communication
The nursing assistant with 20 years of experience approaches a recently
graduated nurse who recently passed the licensing examination. The
nursing assistant states “the only difference between you and me is the
size of our pay checks”. Which of the following is the most appropriate
response for the newly graduated nurse?

A- assert a hierarchical position


B- emphasize the additional education received
C- explain the legal difference in the scope of practice
D- focus on the need to work together for quality client care

A client is receiving mechanical ventilation when the alarm sounds and


displays an alert showing low tidal volumes. All connections and the ET
tube are checked and secured but the alarm persists while O2 saturation
continues to drop. Which of the following is the most appropriate initial
action?

A- increase oxygen delivery to 100%


B- manually ventilate through the endotracheal tube
C- call the respiratory therapist to assess the patient
D- elevate the head of the bed and apply non-rebreather mask
A 47 years old newly been diagnosed as having gastroesophageal reflux
disease (GERD) comes to the clinic for follow up appointment. The
nurse prepares health education on the cause and care of GERD. Which
of the following stomach area is associated with the diagnosis? (See
image)

A- 1
B- 2
C- 3
D- 4

A 45 years old woman presents to the emergency department with her


mother. She complains of headache in the morning and forceful,
projectile vomiting. She reports that she feels off balance lately when
walking. the symptoms have developed slowly, but are increasingly
worsening. Ophthalmoscopic examination of the fundi shows bilateral
venous engorgement. The mother says her daughter doesn’t seem
herself. She has become messy in the home, uses obscene language and
doesn’t seem to care about anything. Which clinical finding should the
nurse observe for most carefully?

A- decreasing muscle tone and strength


B- delayed pupillary response
C- decreasing alertness and orientation
D- headache in the afternoon hours
A 45 years old woman was discharged from the outpatient surgical
center following a procedure in which flexible tubing was inserted into
the urethra to visualize the urethra and bladder walls. The patient
telephones the surgical center two hours after discharge and reports that
her urine is pink colored. Which of the following is the most appropriate
nursing advice?

A- maintain a fluid intake of 1000ml/day


B- return to the surgical center for evaluation
C- elevate the legs above the level of the heart
D- explain that this is normal and continue to observe

A clinical nurse educator identified that patients are not washing their
hands. She planned and conducted sessions for patients about hand
washing techniques and put up posters in the ward. Which of the
following step of nursing process is exercised here?

A- assessment
B- implementation
C- nursing problem
D- evaluation

A35 years old patient was admitted to the medical ward through
emergency department accompanied by her mother. Her chief
complaints include severe epigastric pain, abdominal tenderness and
distention for that last 24 hours. She was anorexic and had passed six
watery diarrheas since the last few hours. She was feeling lethargic due
to frequent elimination and body fluid loss. Which of the following
should the nurse do to reduce the patient’s feeling of exertion?

A- explain that she will feel better soon


B- show support and understanding
C- encourage her to drink fluids
D- help her rest undisturbed
A Diabetic mellitus patient has had left low-knee amputation. The nurse
in the surgical unit has been doing wound care for three days. The nurse
evaluated the wound’s condition to review the care plan. Which sign
shows poor glucose control in wound healing?

A- swelling of the stump


B- bleeding of the stump
C- mild redness of the wound site
D- separation of wound edges

A 32 years old man develops chronic productive cough. He has not been
feeling hungry and has lost three kilograms body weight in the past three
weeks. On examination of his lung fields, there was diminished breath
sound and widespread crackles. An early morning sputum culture was
sent to the lab. (See image)

What is the most likely route of transmission?

A- airborne
B- body fluids
C- fecal-oral
D- nosocomial
During a patient’s blood transfusion, the nurse should recognize that
which of the following are associated with an allergic reaction?

A- diarrhea and rapid onset of chills


B- itching and asthmatic wheezing
C- low back pain and acute renal failure
D- distended neck veins and a rise in venous pressure

A nurse is assessing a patient who is two weeks postoperative a


kyphoplasty of L2 and L3. The patient has been participating in physical
therapy and has been doing daily stretching and strengthening. Which of
the following would indicate that the patient has met discharge goals?

A- reports pain the legs when sitting


B- urinating every two hours while awake
C- fatigue after performing activities of daily living
D- ambulates outdoors without assistive devices

A 57 years old man is admitted to the cardiac unit with palpitations,


headache and chest tightness. On auscultation S3 gallop and diastolic
murmur can be heard. A doctor orders 2.5 mg of verapamil slow IV
push. However, the ventricular rate does not slow down (see image).

Blood pressure 95/62mmHg


Heart rate 170/min
Respiratory rate 25/min
Temperature 36.9C

What next action should the nurse expect?

A- vagal maneuver
B- sedation and intubation
C- another dose of verapamil
D- synchronized cardio-version
A nurse is assigned to care for a 32 years old man with acute
glomerulonephritis. The nurse is transcribing the doctor’s order to the
patient file. Which of the following orders should the nurse clarify?

A- bed rest
B- daily weights
C- strict intake and output
D- frequent blood pressure checks

While preparing paperwork for a patient scheduled for neurosurgery, the


nurse asks about the patient’s use of medications. The patient reports of
taking an aspirin tablet every day, but has not taken it today. The patient
has had nothing by mouth since midnight of the day before. What should
the nurse do?

A- inform the anesthesiologist immediately


B- tell the patient the surgery must be rescheduled
C- record the information on the form in red ink
D- obtain a blood sample and notify the attending physician

A 73 years old bed ridden man is admitted in the geriatric ward in a


semi-conscious state accompanied by one of his neighbors. He has loose
motions on and off for the past few months. He is wearing diaper and
has nasogastric tube inserted. He has bruises on his forearms and is
developing bedsores at the sacral area. In physical examination, mild
crackles sounds are auscultated bilaterally in the upper part of his chest
and at sternum.

Blood pressure 106/86mmHg


Heart rate 96/min
Respiratory rate 28/min
Temperature 39.6 C
Which of the following nursing problems should be prioritized?

A- self-care deficit due to dependency on others


B- altered conscious level related to disease process
C- impaired physical mobility related to weakness
D- ineffective airway clearance due to chest congestion
A patient is being followed in the clinic for hypertension, adult onset
diabetes, and obesity. The patient is apathetic about learning about
nutritional guidelines to reach the goals of weight loss and consumption
of a healthy diet. The patient admitted to eating “whatever is put in front
of me”. Which of the following actions would the nurse take?

A- collaborate with the patient to set goals


B- add a nursing diagnosis of non-compliance
C- refer for psychiatric screening for depression
D- discuss nutritional intervention with the spouse

A 58 years old man had a permanent night pacemaker inserted. The


device is in place and functioning well. The skin at the incision site is
intact and free of infection. What is the most important nursing care
outcome?

A- understand functioning and operating of a pacemaker


B- monitor pulse rate and identify need for reporting
C- able to identify the signs of detached pacemaker
D- modify lifestyle and wear pacemaker alert sign

A nurse is making an assessment for the manifestation of cyanosis in a


dark-skinned client. What would be the best site for inspection if
cyanosis?

A- lips
B- palms
C- nail bed
D- conjunctiva
While setting up a sterile field before surgery, the operating room nurse
finds that the saline solution in the supply cart is contaminated. Which of
the following is the best next step should the nurse do?

A- return it to the supply cart


B- return the saline solution to the linen
C- follow the hospital policy for disposal
D- stock the saline solution in the client room and dispose of it later

A 53 years old man patient in transferred to the recovery room after


coronary bypass surgery. He is on mechanical ventilation and chest tube
drainage is attached. Intravenous line, nasogastric tube and urinary
catheter are intact. What short term goal of care needs priority?

A- monitor heart sounds and report abnormality


B- monitor kidney functioning by urinary output
C- appropriate positioning and promote rest
D- prevent infection at the incision site

A nurse is teaching cord care to a group of new mothers during a baby


bath demonstration. One of the mothers ask the nurse on how to prevent
infection of the cord. Which of the following is the best response?

A- apply antibiotic cream to the cord after every diaper change


B- clean the cord with water when necessary and keep dry
C- clean the cord with surgical spirits twice a day
D- cover the cord with a bandage and change daily

A newborn with hyperbilirubinemia was started on phototherapy. What


will be the nurse’s instruction regarding feeding?

A- feed glucose drinks


B- breastfeed two hourly
C- bottle feed till the bilirubin level reduce
D- breastfeed alternatively with bottle feeds
The nurse manager found the majority of staff took extended breaks
recently. The nurse manager announced the award of best employee of
the month who demonstrates punctuality and is most appreciated by
patients. Which of the following best describes the act of the nurse
manager?

A- encouraging and leading by reward


B- trouble shooting and problem solving
C- showing respect and educating by example
D- managing by objective and enhancing quality care

A nurse manager assigns tasks according to clinical competencies of the


nurses. Which of the following is the management function?

A- delegating
B- evaluating
C- planning
D- controlling

A nurse is teaching a group of new mothers in the post-natal ward on


how to manage breast engorgement after they are discharged to home.
Which of the following statements by the mother will indicate to the
nurse that they understood how to prevent engorgement of the breasts?

A- breast feed every 4 hours


B- breast feeding during the day and bottle feeds at night
C- breast feed every 2-3 hours during the day and stop at night
D- breastfeed the baby every 2-3 hours during the day and night
A 2 months old infant with cleft lip is seen in the primary health care
center to get the regular vaccine of 2 months. The mother asked about
the proper time for the corrective cleft lip surgery of her infant. Which
of the following is the best nurse response?

A- no specific age for repair of cleft lip


B- it is too late, the repair should be done immediate after delivery
C- the age of 2 months is the time of repair
D- the proper time for repair after the age of one year

A 32 years old gravida 2, para 1, at 38 weeks’ gestation presents with a


rigid and hypertonic abdomen; she rates her pain at a level 8 on a scale
of 1-10, and says it is constant. She has vaginal bleeding of dark red
blood with clots. She reports that her pain came on suddenly.

Blood pressure 106/60mmHg


Heart rate 90/min
Respiratory rate 22/min shallow
Temperature 37.0C tympanic

Which of the following is the most likely health problem?

A- vasa previa
B- placenta previa
C- placenta abruption
D- rupture of membranes
Less than 24 hours after being discharged, a mother who is newly
postpartum calls a nurse in the mother/infant unit. She reports that she
has very heavy bright red bleeding. What would be the advice to give
her?

A- “dont worry about it; it is normal”


B- “call your doctor and ask what to do”
C- “lie down, massage lower abdomen, if it does not work then come
to the hospital immediately”
D- “lie down for about an hour. Then check your bleeding, if it is still as
heavy, call me back”

Which of the following circumstances is most likely to cause uterine


atony and lead to postpartum hemorrhage?

A- endometriosis
B- urine retention
C- cervical and vaginal tears
D- hypertension
The nurse is caring for a 5 years old boy in the postoperative unit after
he underwent tonsillectomy. The child’s gag reflex is intact and he lies
in the side lying supine position. The nurse notices that he is repeatedly
swallowing. He rates his pain at a level 8 using the smiling face pain
scale for kids.

Blood pressure 90/50 mmHg


Heart rate 150/min
Respiratory rate 26/min
Temperature 37.1C
What is the most appropriate initial nursing response?

A- notify the doctor


B- elevate the head of bed
C- administer analgesic
D- administer infusion bolus

A 9 years old child has a fractured femur and full leg cast has been
applied. Which of the following is physiologic effect of immobilization?

A- venous stasis
B- increase metabolic rate
C- positive nitrogen balance
D- increased need for oxygen

Clomiphene citrate (Clomid) is prescribed for a 32 years old woman for


infertility treatment. The nurse should understand that this medication is
used for which of the following actions?

A- induce ovulation
B- decrease prolactin level
C- reduce endometriosis
D- stimulate the release of Follicle-stimulating hormone (FSH)
A 17 years old mother presented to the primary health center ten days
after delivery. She is suffering from fatigue, anemia, fever and extensive
vaginal discharge. (See lab results)

Test Result Normal


RBC 4.6 4.7-6.1 x10^12/L male
4.2-5.4 x10^12/L female

Hb 88 130-170 g/L male


120-160 g/L female

Which of the following is considered as the main maternal complication


of postpartum hemorrhage?

A- death
B- candidacies
C- cervical cancer
D- uterine prolapse

A 3 years old child is seen to the emergency department experiencing a


seizure at home. There is no previous history of seizures. The mother
informs the nurse that she does not believe the child has epilepsy. Which
of the following is the nurse’s best response?

A- no need to worry, because epilepsy is easily treated


B- very few children have actual epilepsy
C- the seizure may or may not be epilepsy
D- your child has had only one seizure
A discharge planner is reviewing the file of a patient in preparation to
move him from acute care to home care. What activity should the
discharge planner do to ensure the continuity of care?

A- make referral to the appropriate department


B- visit patient’s house to make sure is safe
C- call pharmacy to prepare medications for patient
D- organize transportation for patient to be moved to the house

A pregnant client with organ dysfunction presents with visual changes,


3+ deep tendon reflexes and a severe headache. At a subsequent follow-
up visit the client reports that the symptoms have resolved. What is the
most likely reason for the improvement?

A- sleeping in lateral position


B- consumption of 8 ounces of water per day
C- exposure to bright light
D- ambulation every 30 minutes

During a physical examination of a patient with mental illness, he


panicked, pointing at the nurse’s stethoscope, and said “there is a snake
around your neck, save yourself”. Which of the following symptoms
reflects the patient’s response?

A- illusion
B- obsession
C- hallucination
D- depersonalization
A child with ventriculoperitoneal shunt discharge is anticipated after 2
days. The nurse is teaching the parents about signs for shunt
malfunction?

A- depressed fontanel
B- vomiting, lethargy
C- increased heart rate
D- hematuria

A 5 years old child admitted with bacterial meningitis and is having


seizure. Which of the following intervention the nurse should initiate?

A- restrain the child’s limbs to prevent injury


B- slowly put the child on his side on the floor
C- clear the area of objects and administer oxygen
D- roll the child to prone position to protect the airway

A nurse is providing health education for a primigravida patient who has


gestational diabetes mellitus. Which of the following statements made
by the patient indicates the need for additional education?

A- “I should not do exercises”


B- “I should follow the prescribed diet”
C- “I should monitor my blood glucose”
D- “I should report any sign of infection”

A physically handicapped young woman uses a wheelchair to adapt with


her problem. She had a neighbor who repeatedly tell her that she might
did a fault and for that she has been punished from God. Which model of
disability does this attitude belongs to?

A- moral
B- medical
C- disability
D- rehabilitation
A 36 years old son is the primary caregiver to his 76 years old father
who has many chronic diseases and need full assistance. The sun
discussed with the community nurse the idea of referring his father to
one of the elderly day care centers. The nurse explains the eligibility of
this case to such services. Which of the following elderly groups this
patient belongs to and eligible to this service?

A- with busy caregivers who need an assistance


B- who are bored staying at home and need socialization
C- who have been diagnosed with Alzheimer
D- who want to engage in handcraft activities and art activities

What must be known on legal points of delegation when explaining the


process of new nurse?

A- evaluation of performance of delegatee by clients


B- actual time it takes to complete the task by delegatee
C- institution definition of the job description of delegatee
D- number of times that the delegatee has previously performed the task

A 34 weeks pregnant mother is in the antenatal clinic on her routine


visit. She is experiencing progressively decreased number of fetal
movements since past few days and had not felt any kicking since the
morning. She is having lower back discomfort and has on and off mild
lower abdominal pain for the past week. Which of the following is the
first most desired outcome?

A- send for abdominal ultrasound


B- identify signs of premature labor
C- check fundal height and size of the baby
D- auscultate and monitor fetal heart sounds
A 4 months old infant is admitted to the emergency department with a
fractured arm. The mother indicated that that while the child was
crawling, he fell down the stairs and broke his arm. Which of the
following observations would lead the nurse to suspect that this child is
a victim of abuse?

A- age inappropriate injury


B- pattern and shape of the injury
C- child is appearing malnourished
D- improper explanation of the cause of the injury

Which of the following patients is the most appropriate to delegate to a


new nurse?

A- patient on a strict bed rest


B- patient for discharge teaching
C- patient scheduled for an operation
D- patient who is hooked to mechanical ventilation

The midwife was assessing a 36 years old gravida 4, para 2 postnatal


mother. The patient was in labor for 10 hours and had vacuum
extraction. Two sanitary pads were fully soaked with blood within two
hours been admitted to the postnatal ward. Which of the following is the
appropriate nursing diagnosis?

A- anxiety related to blood loss


B- fatigue related to lack of oral intake
C- activity intolerance due to discomfort
D- fluid volume deficit due to uterine atony
In the antenatal clinic, a 9 months pregnant woman notifies the nurse
that she is concerned that she gained 15kg during this pregnancy and she
is fat and needs to start on a diet program. Which of the following is the
best nursing response?

A- this weight gain is normal during pregnancy


B- you can reduce the carbohydrates in your diet
C- wait till you have the baby and start a diet program
D- this weight is lower than you should gain during pregnancy

A nurse is conducting an awareness class for mothers about the common


health concerns primary school students and the requirements for
admission. Which of the following is a requirement prior admission to
primary school?

A- family history record


B- scoliosis screening results
C- immunization record
D- medical history record

A 7 years old child is admitted to pediatric medical diagnosed with


nephrotic syndrome. His urine analysis result revealed that the protein in
his urine is 2gm/24hrs. Which of the following diet should include to
this child?

A- high carbohydrates
B- high protein
C- salt restriction
D- low fat
During cardiopulmonary resuscitation for a patient, the team leader is
introduced to members as the decision maker. What is the leadership
style in this situation?

A- autocratic
B- democratic
C- bureaucratic
D- laissez-faire

A 4 months old infant is admitted with diarrhea and vomiting. To


prevent the reoccurrence of diarrhea the nurse should instruct the mother
to do which of the following?

A- frequent hand washing


B- increase milk intake
C- increase water intake
D- change diaper frequently

An 8 years old girl is seen in the emergency department because of a


fracture in the right fibula and she has a fiberglass cast on her leg instead
of a plaster cast. Which of the following is an advantage to use a
fiberglass cast?

A- cheaper
B- dries rapidly
C- smooth external
D- shape closely to body part
A new community nurse in the rural clinics has to have good
background about the health status of the community in order to assess
their needs. What is the appropriate way to assess the health status of the
community?

A- home visit
B- community assembly
C- mass information campaign
D- community health assessment

Which of the following patients the nurse should see first?

A- patient complaining of muscle aches and fever


B- patient scheduled for electrocardiography
C- patient newly diagnosed with hypertension
D- patient with complaints of chest pain

A nurse manager was not happy about low results of patient satisfaction
survey. Which of the following is the first step for the nurse manager?

A- schedule a meeting with the staff


B- start changes to improve
C- review evaluation details
D- report to director

A woman who is 32 weeks gestation. Her weight was 66kg last month
and today is 78kg. Which of the following is the best nursing action?
A- assess the size of the fetus
B- give health education on good nutrition
C- advise her to exercise and lose some weight
D- check her blood pressure and test her urine for protein
Which of the following is contraindicated for adenotonsillectomy among
children?

A- child with adenoid infection more than 4 times/ year


B- child with tonsillitis more than 4 times/ year
C- child’s age less than three years old
D- child with hypertrophied adenoids

When performing a postpartum assessment on a woman, the nurse notes


the presence of clots in the lochia. The nurse examines the clots and
notes that they are larger than 1 cm. Which of the following nursing
action is most appropriate?

A- document the findings


B- notify the physician
C- reassess the client in 2 hours
D- encourage increased intake of fluids

A woman is 5 weeks gestation is diagnosed with hyperemesis


gravidarum. What is the most important nursing action?

A- psychological support
B- folic acid supplementation
C- strict intake and output record
D- providing the woman with high protein diet
A 27 years old woman patient’s umbilical hernia is repaired under local
anesthesia. Her gold amulet which she kept under her pillow was
missing at first but now is given to her after it is found from the bag of
the housekeeping staff. The nurse in charge is to write an incident report
which includes the patient’s name, hospital identification number, date,
time, and place of incident and details of the item stolen. What short
term action is required by the in charge nurse?

A- explain patient her belongings are her responsibility


B- personal counseling of the housekeeping staff
C- report incident to the security department
D- dismiss the involved personnel

A nurse was planning for activity therapy for a patient with manic
episode. The nurse has a main concern that the patient is easily
distracted and has hyperactivity. What should be the characteristic of the
activity?

A- competitive
B- stimulating and colorful
C- required little concentration
D- build up positive self-esteem

A 6 years old pre-school returned from the operating room after


tonsillectomy. A nurse received the patient from the recovery room.
Which sign the nurse should assess first for bleeding?

A- frequent swallowing
B- decreased pulse rate
C- complaints of discomfort
D- elevation of blood pressure
A 32 years old multigravida woman presents to the outpatient clinic
complaining of dysmenorrhea and menorrhagia. She had been diagnosed
with uterine fibroids and blood studies has been ordered for her. Which
of the following results should the nurse report?

A- hematocrit, 37%
B- hemoglobin, 90 g/dl
C- white blood cell count, 10,000 cells/mm3
D- platelets count, 300,000 platelets/ microliter

A 7 months old infant seen in the emergency department suffering of


severe abdominal pain, and the infant’s stool became like red jelly.
Abdominal examination revealed sausage-shaped mass in the right upper
quadrant. Which of the following is the first line of therapeutic
management for this infant?

A- non-surgical hydrostatic reduction


B- surgical simple reduction
C- pyloromyotomy
D- endorectal pull-through

The nurse listens to patient’s complaints, and discussed how each of


complaints can be managed. Which of the following is the best
description of the nurse’s action?

A- promoting patient’s autonomy


B- demonstrating sympathy
C- ensuring patient support
D- showing concerns
Mother of nine children, three of them with congenital anomalies and
one down syndrome; she is primary school graduate, with low financial
status. She is not using any method of family planning. So, the primary
health care nurse has referred her for counseling. Which of the following
is the best health education method that can be used?

A- community organization
B- individual counseling
C- group discussion
D- health class

Which of the following is the most accurate description of successful


conflict resolution?

A- employees get along with one another


B- involved group decide on issues by majority rule
C- conflicting parties are encouraged to solve problems together
D- involve upper level management to decide the outcome

A mother in the midwife’s clinic with a complaint of redness and


swelling of her right breast. She is breastfeeding her nine months old
baby. What should be immediate intervention?

A- start on antibiotics
B- detailed investigation
C- stop the breastfeeding
D- wound cleaning and dressing
Which of the following statements indicate that a nursing staff need an
education session related confidentiality and information security?

A- “giving the password of a patient’s family member”


B- “leaving the computer in the nursing station secured”
C- “keeping their personal electronic password secured”
D- “allowing nursing students to read medical record for their patients”

A child is admitted to the pediatric ward with acute lymphocytic


leukemia, and has had joint pain for several weeks. Physical findings
revealed widespread ecchymosis, generalized lymphadenopathy,
hepatosplenomegaly, and pallor. Lab work shows low hemoglobin level,
low RBC count, low WBC count, low hematocrit and low platelets.
Which high risk is expected?

A- infection
B- fractures
C- dehydration
D- hepatitis

The quality nurse manager teaches a new nurse graduate about the
comparison between total quality assurance (QA) and quality
improvement (QI). What is the most appropriate statement regarding the
common aspect between QA and QI?

A- aim to improve quality


B- focus on preventing errors
C- focus on staff development
D- view nursing activities as major tasks
A newborn is diagnosed with hypospadias. When teaching the parents of
this child, the nurse should tell them to avoid which of the following
before the hypospadias repair?

A- circumcision
B- drinking acidic juices
C- urinary catheterization
D- riding a bicycle

A maternity nurse is performing a newborn assessment thirty minutes


after delivery of a baby who did not receive any prenatal care and has an
unknown gestational age. The skin is extensively leathery, cracked and
dry and there is an absence of lanugo and vernix. How many weeks’
gestation is this neonate?

A- <30
B- 30-35
C- 36-40
D- >40

A nurse is giving health education regarding the management of


premenstrual syndrome symptoms to a group of first year nursing
students. Which of the following information regarding diet will the
nurse include in the presentation?

A- take three healthy meals a day to prevent hypoglycemia and increase


feeling of well-being
B- include simple sugars in the diet to prevent low blood glucose levels
which cause mood swings
C- drink 2000ml of fluid (water, coffee, tea) per day to flush the kidneys
and improve fluid retention
D- decrease intake of caffeine such as coffee and chocolate to
minimize irritability, insomnia and anxiety
A newly assigned nurse to the home health care department was
conducting pre-entry phase of home visit. Which of the following
activities should the nurse do?

A- call physician for referral order


B- collect information and schedule a visit
C- establish mutually acceptable goals for care
D- conduct a health history and physical assessment

A 28 years old woman is accompanied by her mother to the psychiatric


ward. She is admitted for her affective disorders with anger, amnesia of
recent events and stammering spells. She has panic anxiety attacks
followed by apathy and listlessness. How should the nurse manage the
patient?

A- be supportive and use therapeutic communication


B- improvise sign language to control forgetfulness
C- maintain calm environment and avoid arguments
D- anger management and speech therapy

A woman was diagnosed with gestational trophoblastic disease. What is


the lab investigation was done to diagnose the disease condition?

A- cervical pap smear


B- serum HCG levels
C- serum estrogen levels
D- plasma thyroxine levels
A toddler is seen in the emergency room with a history of high
temperature for five days. He had watery eyes and cough for the last
three days, and then he developed a rash all over the body which started
in his face and spread down to his body. The mother did not remember
about the vaccination schedule of her son. For how long the child should
be isolated?

A- 5 days
B- 10 days
C- 15 days
D- 20 days

Which test should be performed to screen for cervical neoplasia during


antenatal assessment?

A- papanicolau (Pap) test


B- vaginal rectal culture
C- rapid plasma reagin test (RPR)
D- venereal disease research laboratory test (VDRL)

In the hospital digital dashboard, what type of data are typically


displayed?

A- staffing
B- financial
C- performance
D- knowledge-based
A 3 days old newborn is diagnosed with Hirschsprung disease. The
nurse is conducting a physical examination. Which of the following
findings will alert the nurse to suspect this disease in the newborn?

A- palpable sausage-shaped mass


B- cyanosis of fingers and toes
C- failure to pass meconium within 24-48 hours of life
D- weight less than expected for height and age

A 3 years old child is admitted to the hospital with seizures. He was


alert, oriented and has a rash in his extremities and is diagnosed with
meningitis. While doing physical examination of him, he starts to
develop seizures.

Blood pressure 100/57 mmHg


Heart rate 110/min
Respiratory rate 30/min
Temperature 39.5 C

Which of the following vaccine is used to prevent meningitis?


A- hib vaccine
B- varicella vaccine
C- BCG vaccine
D- Rubella vaccine

When a patient was first diagnosed with schizophrenia, one of his family
members asked the nurse about the possible causes. The nurse said that
one reason is that he may have had an excess secretion of a
neurotransmitter. Which of the following neurotransmitters?

A- serotonin
B- dopamine
C- glutamate
D- endorphins
A nurse accidentally dropped a medication ampule, informed the charge
nurse, completed an incident report form. The charge nurse arranges
medication replacement. Which of the following is the immediate
nursing action required?

A- revise protocol for medication related incidences


B- allocate senior nurses to medication assignments
C- provide missed medication dosage to patient first
D- incident reporting must be given the priority

A nurse documented assessment on a newborn as listed. (See table)

Indicator Points
Heart rate less than 98/min
Respiratory rate 28/min irregular
Muscle tone minimal flexion of the extremities
Reflex irritability grimace
Color body pink, extremities blue

What is the total apgar score?


A- 0
B- 3
C- 5
D- 7

A 32 years old gravida 1 and para 0, is now planning to become


pregnant within the next year. The patient herself had been born with
neural tube defects, which were surgically repaired in early childhood.
The nurse recommends that the patient begin taking daily tablets of folic
acid. How many micrograms would be most appropriate for this patient?

A- 400
B- 600
C- 1800
D- 4000
While a school nurse was teaching a group of 14 years old children, one
of them remarked “you are too young to be our teacher! You’re not
much older than we are!” How should the nurse respond?

A- “how old do you think i am”


B- “we need to stay focused on the topic”
C- “do you think you can teach it any better”
D- “i think i am qualified to teach this group”

Vital statistics and control of communicable disease serve many


functions. Which of the following is the basic function for health?

A- public
B- mental
C- education
D- maternal & child

A nurse is showing stress and anxiety due to long duty hours. Which of
the following should the nurse manager do?

A- give day off


B- give light tasks
C- offer vacation
D- talk about coping
A mother brought her 6 months old infant for a well-child checkup.
Which immunization should the nurse anticipates to administer as per
immunization schedule recommendation?

A- measles, mumps, rubella


B- varicella, measles, hepatitis b
C- diphtheria, tetanus, pertussis
D- hepatitis b, rotavirus, diphtheria

Which of the following would be the proper interpretation by the nurse


for a chromosomal analysis showing 46, xy?

A- normal male
B- abnormal male
C- normal female
D- abnormal female

During meningitis outbreak in one of the hospitals, the nun-immune staff


were given an immunoglobulin in order to prevent them from the
infection. What is the type of immunity that will be developed by these
staff?
A- active
B- long-term
C- natural
D- passive
A 68 years old woman is receiving parental nutrition at home. The
district nurse visits the woman and notes that has gained one kilogram of
weight since a health provider had visited one week ago. There is pitting
edema of 2+ of lower extremities. The patient is alert, active and
oriented. Which nursing diagnosis is most appropriate?

A- non-compliance
B- impaired gas exchange
C- imbalanced nutrition
D- fluid volume overload

A 54 years old woman presents to the emergency department with sharp


upper right abdominal pain that radiates to the right scapula. While
performing the admission assessment, the patient becomes nauseous and
begins vomiting. She states that she has had pain in the upper right
quadrant previously but that time it was far worse. There is positive
Murphy’s sign and an ultrasound confirms gallbladder wall thickening
and pericholecystic fluid collection. Which of the following would most
likely be associated with her clinical findings?

A- relief by drinking milk


B- alleviation with exercise
C- triggered by fatty meals
D- worsening on empty stomach

A nurse is documenting in patient records several events that occurred


during home health visits. Which of the following is an example of the
correct way to document patient information?

A- patient fell walking to bathroom. Busy preparing for sterile dressing


change when patient left the room
B- patient got out of bed without assistance. Denies any symptoms
when ambulating alone
C- patient sitting in chair. Strict bedrest orders ignored
D- patient showering. Electronic epidural infusion pump on floor by
shower stall
An 82 years old patient has Parkinson’s disease. During the assessment,
the nurse would expect which of the following actions to produce the
most tremor activity of the hands?

A- eating with a fork


B- resting hands in lab
C- standing with hands loose at sides
D- rolling a small pill between the fingers

A 67 years old man was admitted to the hospital following a closed bone
fracture. An intramedullary nail is inserted and the patient is placed in
balanced skeletal traction. The following day, the patient becomes
restless, drowsy and confused. He has difficulty breathing appears very
tired. Which additional sign or symptom would require immediate
intervention?

A- anxiety
B- cold skin
C- constipation
D- petechiae on chest

The nurse is assisting a patient to ambulate in the hall. The patient has a
history of coronary artery disease (CAD) and had coronary artery bypass
graft surgery (CABG) 3 days ago. The patient reports chest pain rated 3
on a scale of 0 (no pain) to 10 (severe pain). The nurse should first:

A- determine how long it has been since the patient’s last dose of
aspirin
B- obtain a chair for the patient to sit down
C- assess the patient radial pulse
D- ask the patient to take several slow, deep breaths
A nurse is caring for an 8 years old male with cystic fibrosis. Based on
the nurse’s understanding of the disease, what nursing intervention
should the nurse expect to perform?

A- restrict sodium and fluid intake


B- give antidiarrheal medication
C- discourage coughing after postural drainage
D- administer pancreatic enzymes with each meal

An infection control nurse notices purulent exudate, redness and


tenderness on the surgical wound site for a few postoperative patients in
a surgical unit. She discussed with the ward nurses and emphasized that
wound infection after surgery can be prevented. Which of the following
is the best possible action to minimize the incidence of wound infection?

A- perform assessment of pain on the wound site


B- wash hands before and after each patient activity
C- encourage adequate intake and early ambulation
D- support incision appropriately and avoidance of straining

A three years old has returned to the clinic after 4 days of being
diagnosed with gastroenteritis and dehydration. A parent reports that the
vomiting has stopped, and the child is tolerating liquids, rice,
applesauce, and bananas. The diarrhea persists, but seem to be
decreasing in volume. When evaluating for signs of dehydration, the
nurse will assess the patient’s skin turgor by:

A- grasping the skin over the abdomen with two fingers and raising
the skin with two fingers
B- grasping the skin over the forehead with two fingers and raising the
skin with two fingers
C- holding the patient’s mouth open and assessing the tongue for deep
creases or furrows
D- drawing two tubes of blood and running blood urea nitrogen (BUN)
and creatinine (Cr)
A 45 years old man who is hospitalized feels the constant need to keeps
things in order, particularly whilst eating. The nurse observes him
arranging the food on his plate into symmetrical and equal bite-sized
pieces. He constantly worries that the food served could be outdated and
potentially cause illness. Which nursing diagnosis is most important?

A- ineffective verbal communication


B- self-esteem disturbance
C- impaired social interaction
D- anxiety

A30 years old married man presents to the clinic with complaints of
feeling sad for the past three months. He is unable to maintain regular
sleep routine; he lost his appetite and has difficulty concentrating. He is
prescribed a medication which prevent the reuptake of specific
neurotransmitters that could contribute to his mental health problem.
Which side effects would be most important for the nurse to advise the
patient of?

A- polyuria
B- photophobia
C- fluid retention
D- sexual dysfunction

A 70 years old man is brought to the emergency department on a


wheelchair. He complains of drowsiness, fatigue and lack of appetite for
the past three days. With history taking, the patient stated that he did not
eliminate since five days and he is not ambulating a lot because he has
arthritis in both knees. Why is constipation one of the common problems
for immobilized patients?

A- decreased peristalsis
B- increased colon motility
C- an increased defecation reflex
D- decreased tightening of the anal sphincter
A 46 years old patient is admitted in the female medical ward for severe
back pain, which is graded 6 on the scale of 10. Her vital signs are stable
with the slight elevation in her blood pressure. The patient has refused to
eat lunch which is a low sodium diet. The attending nurse has
documented the patient is uncooperative and has refused to eat the
lunch. What can be assessed by the nursing documentation regarding the
patient?
A- subjective judgement of patient’s statement
B- misunderstanding of patient’s attitude
C- understatement of communication
D- unethical evaluation of reality

A 62 years old man is admitted in the surgical ward is scheduled for the
surgical removal of polyps from his descending colon under general
anesthesia. He is experiencing fatigue, abdominal pain and blood
streaked stool for a couple of months. He is worried whether the
bleeding in his stool is going to stop after the surgery. Which of the
following nurse’s responses is the most appropriate?

A- surgery often relieves the symptoms


B- let us have a detailed discussion with your physician
C- your condition may or may not resolve, it depends
D- in fact surgery is the only treatment for the problem

A 78 years old alert and oriented patient is returning from the radiology
department and the nurse is preparing to transfer the patient from the
wheelchair back into the bed. The nurse places the gait belt on the
patient and prepares to lift the patient from the chair. Which body
mechanics would be most appropriate?

A- widen leg stance


B- bend over to left
C- rotate from the waist
D- maintain base of gravity in the feet
A 33 years old man presents to the emergency department with high
grade fever, tachycardia, and tachypnea. What is an appropriate nursing
intervention for the patient’s fever?

A- provide dry clothing


B- keep limbs close to the body
C- cover the patient’s scalp with a cap
D- measure the patient’s fluid intake and output

A home care patient with chronic Obstructive Pulmonary disease reports


an upset stomach. The patient is taking Theo-dur and triamcinolone
acetonide (azmacort).
Which of the following counselling should be given?
A- Theo-dur on an empty stomach
B- Theo-dur and azmacort at the same time
C- Theo-dur and azmacort 12 hours apart

D- Theo-dur with milk or crackers

A cardiac monitor for a patient in a Coronary Care unit shows abnormal


ECG rhythm with heart rate of 159 beats, QRS complex (0.18 second),
and absent P wave.What could be the type of possible dysrhythmia?

A- Sinus tachycardia
B- Ventricular tachycardia
C- Ventricular fibrillation
D- Supraventricular tachycardia
A nurse is caring for a 72-year-old man patient who is unsteady. The
patient requests the nurse to help. nurse asked the patient to wait few
minutes to get a device to transfer him. Which of the following transfer
devices is the nurse to use?
A- Belt
B- Board
C- Handle
D- Mechanical lift
A 40-year-old man. smoker, presents to the clinic. On examination, the
toes are cold to the touch. Extremities are pale to blue. The pedal pulse
examination of the fingers shows small ulceration the skin. Blood
glucose testing is normal and then history of diabetes. On admission
Blood pressure 140/90 mmHg
Heart rate 86 /min
Respiratory rate 22 /min
Oxygen saturation 98% room air
Which of the following would be most effective?
A. Antibiotic administration
B. Reduced fat intake
C. Smoking cessation
D. Regular exercise

An outpatient nurse is drawing blood for laboratory analysis. Which of


the following steps is appropriate for this procedure?
A. Use a highly visible vein on ventral side of
B. Immediately draw blood while it is still wet after alcohol prep
C. Vigorously shake specimen tube to mix anticoagulant solution
D. Do not leave a tourniquet on more than 1 vein
A 28-year-old man and his wife involved in an accident, which his wife
was killed. The client is being admitted in the Care Unit for multiple rib
fractures and a broken. He asks which room his wife is located.

Which of the following is the most appropriate response?


A- "Your wife is not in the hospital"
B- " I'm sorry, but your wife did not survive the
C- "I need to get your family so that you can wife"
D- " The doctor will be talking to you about located"

A client with type 1 diabetes mellitus is admitted. Which medication if


found by the nurse in the should be clarified with the physician?
o A. Humalog (lispro) sliding scale before meals
o B. Glargine (Lantus) 10 units subcutaneously
o C. Metformin (Glucophage) 500 mg per orem
o D. Dextrose 50% ampule intravenous push for 50 mg/dL

During surgery requiring general anesthesia, and a carotid pulse is not


palpated. How many compressions per minute should be given?
A. 50
B. 60
C. 80

D. 100
A nurse is planning to discharge a known HIV, at the Isolation Unit after
the recovery from upper. Which of the following nursing problem
requires
A. Risk of infection due to altered immune
B. Fluid volume deficit due to frequent diarrhea
C. Anxiety due to disease, fear and social
D. Weight loss due to higher metabolism rate

A nurse is working in a medical unit. She is 65-year-old women with


cerebrovascular accident home care visits. The nurse teaches the patients
wheelchair safely. Which of the following statements by the
teaching was effective?
A. "The breaks should be used when storing
B. "I will push the wheelchair in from of the
C. "I will remove the feet rests from the

D. "When entering the elevator I will pull the that the big wheels enter
first"
After teaching the deep breathing and coughing who is undergoing a
surgery, the nurse asked demonstration and then helped him in
correcting. What part of therapeutic communication is used

o A. Evaluation
o B. Intervention
o C. Identification
o D. Demonstration

A nurse is caring for patient who is being admitted tract infection. The
patient feels cold and shivering
Blood pressure 110/70 mmHg
Heart rate 110 /min
Respiratory rate 22 /min
Temperature 39.7C
Which of the following is the best nursing action?

o A. Provide a hot drink


o B. Cover the patient with light blanket
o C. Start the air conditioning system
o D. Turn off lights and close curtains
A patient who has pain localized in the chest, now down the left arm.
What is this type of pain called?
o A. Phantom

o B. Regimented
o C. Shooting
o D. Radiating

A 75 year-old bedridden patient is hosp Medical ward for the treatment


of her in Area. Her wound is infected with multiple Longer than normal
time to treat. She Crying spells and is talking about death
Which of the following nursing diagnosis must be
o A. Health and wellness
o B. Coping mechanism
o C. Self-perception
o D. Belief system
A patient returned to the Surgical Unit from the thyroidectomy. The
nurse observed that the arousable.
Blood pressure 90/60 mmHg

Heart rate 108 /min


What immediate action should the nurse take?
o A. Recheck pulse and blood pressure
o B. Administer intravenous fluids as ordered
o C. Place client in modified Trendelenburg's
o D. Assess the back of surgical dressing for

A 42-year-old women is brought to the Emergency with severe chest


pain radiating to the left neck physician instructs nurses to start the
Morphine
Aspirin (MONA) protocol. What is the top priority of nursing care?
o A. Administer oxygen according to the physician
o B. Administer aspirin according to the physician
o C. Administer morphine according to the physician

o D. Administer nitroglycerin according to the physician


A nurse is assigned to care for a Muslim female emergency cesarean
section. The patient asks the to wear a head scarf during the surgery.
Which of the following is the most appropriate

o A. Ask the patient to remove the scarf


o B. Tell the patient that it is ok to wear the scarf
o C. Ask the patient to wear a surgical cap
o D. Tell the patient that a request will be during the surgery

A nurse is assigned to care for a 32-year-old glomerulonephritis. The


nurse is transcribing patient file.
Which of the following orders should the nurse
o A. Bed rest
o B. Daily weights
o C. Strict intake and output check
o D. Frequent blood pressure check

The nurse is assigned to care for several patients Who should be


assessed first by the nurse?
o A. A diabetic patient who will be discharged
o B. A patient with rheumatoid arthritis who has
o C. An asthmatic patient who is due for
o D. A patient with surgical incision who required
A 22-year-old patient is admitted in the male Medical diagnosis of
tonsillar abscess. He has high fever and sever along with dysphagia,
difficulty in talking and in opening patient is planned for needle
aspiration of the abscess intravenous antibiotics including penicillin.
What expected outcome of nursing care should be prioritized?
A. Patient is comfortable and has minimum pain
B. Patient is able to communicate appropriately
C. Able to swallow fluids and soft diet more easily
D. Normal body temperature and stable vital signs

A 40-year-old man with Hepatitis A is admitted in Medical Ward. He is


feeling exhausted, anorexic, and diarrhea. He needs hygiene assistance.

Heart rate 80 /min


Blood pressure 125/96 mmHg
Respiratory rate 24 /min
Temperature 39.4C
What nursing problem should be focused first?
A. High fever

B. Low energy level


C. Reduced food intake
D. Stomach discomfort
A nurse is caring for a patient diagnosed with abdominal x-ray reveals
evidence of pancreatic sign of biliary disease.
What would the nurse most likely instruct the patient

A. Consume a high-protein diet


B. Gradually increase dietary fibre
C. Avoid ingestion of alcoholic beverages
D. Limit activities when the patient is fatigued

A 57-year-old patient post laparotomy is complain increases with


movement. A family member ... The unit nurse is preparing to admini
for pain control after checking the medication
Blood pressure 110/63 mmHg
Heart rate 68 /min
Respiratory rate 11 /min
Temperature 36.8C

Oxygen saturation 93 %
Which of the following assessments should the administering Morphine
to the patient?
A. Monitor heart rate and rhythm
B. Inspect surgical site for bleeding
C. Perform a respiratory assessment

D. Send a blood sample to assess renal


A nurse is transferring a patient with chest tube the X-ray department for
chest X-ray. Which location should the nurse place chest tube
A. Directly on the stretcher in an upright

B. On the side of stretcher next to patient


C. Hanged on IV pole that is attached to the
D. Attached to the stretcher and hanged

The relative of a 25-year-old post-partum patient stopped her from


taking bath until the 40 days provided with the food by the family that
was of saturated fat and a liter of reduced full asking, she said the diet
was good for the milk
Which of the following should be the prioritized patient's understanding
about?
A. A healthy and balanced diet is beneficial
B. Breast feed is not affected by mother's diet

C. Maintaining personal hygiene is most


D. Rest and relaxation is necessary to regain
A 53-year-old patient who was referred to the primary healthcare center
for hyperglycemia him and finds out that he has been recently
says that he does not understand why he needs

Test Result Normal Values


Glucose, fasting 12.2 3.5-6.5 mm
Which of the following learning needs is indicated
A. Insulin Alternatives
B. Complications of diabetes
C. Disease process of diabetes

D. Lifestyle and dietary changes

At the beginning of the afternoon shift in the receiving handover from a


morning shift nurse post coronary angioplasty. The nurse waveform on
the monitor, upon assessment the carotid pulse (see image).

What should be the most appropriate next action


A. Give a DC shock

B. Refuse to receive patient


C. Check ventilators settings
D. Shout for help and start chest compression
A 33-year-old man with meningitis present Department. Healthcare
providers plan to precautions.
what is the best nursing practice for this case?

o A. Use N95 mask all the time


o B. Monitor the negative air pressure
o C. Use respiratory protective equipment all the
o D. Keep the patients with the same diagnosis

A 30-year-old construction worker was stung by roof structure . He then


began to complain of feel An ambulance was called and on arrival, they
four the ground and having breathing difficulties. His the radial pulse is
weak and thread.
Blood pressure 92/52 mmHg

Heart rate 118 /min


Respiratory rate 26 /min
Which white blood cell is most likely be elevated?
A. Eosinophils
B. Basophils
C. Monocytes

D. Neutrophils
A nurse working in medical unit is preparing to with droplet precaution
measures in place. The following personal protective equipment;

eyewear. what is the correct sequence foe putting the equipment on?
A. Face Mask, Gown, Eyewear, and Gloves
B. Gown, Face Mask, Eyewear, and Gloves
C. Eyewear, Cloves, Face Mask, and Gown
D. Gloves, Gown, Face Mask, and Eyewear

A 22-year-old patient is admitted in the male diagnosis of tonsillar


abscess. He has high fever along with dysphagia, difficulty in talking
and in patient is planned for needle aspiration of the intravenous
antibiotics including penicillin. What nursing problem needs attention
first?
A. Imbalance nutrition due to inadequate
B. Acute pain related to throat inflammation
C. Impaired swallowing related to dysphagia

D. Hyperthermia related to acute infection


An elderly patient who has an aortic aneurysm Intensive Care Unit to a
Medical Surgical Unit on day. While assessing the client, a nurse notes
extremities and is unable to palpate the pedal pulse

Which intervention should the nurse implement?


A. Wrap the lower extremities with warm
B. Use a Doppler ultrasound to reassess the
C. Elevate the extremities above heart level
D. Place a bed cradle over the bed to levitate

A nurse is assigned to care for a patient with extremities and the nurse
wants to assess the extremities.

Which of the following techniques should the nurse


A. Fingertips
B. Base of the fingers
C. Dorsal surface of hand
D. Grasping between fingertips
A patient who underwent a left foot amputation post-operative Care Unit
and the following assess (see lab results).
Blood pressure 110/80 mmHg

Heart rate 65 /min


Respiratory rate 13 /min
Temperature 37.2C
Oxygen Saturation 98 % on room air
Test Result Normal Value
Hb 120 120-158 g/L

WBC 10.2 4-10.5 ×10


RBC 3.8 3.8-5.1 ×10
Fasting blood sugar 9.3 3.5-6.5 mm
LDL 6.5 <4.0 mmoi
Triglycerides 3.8 <2.16 mm
Which long-term complication is most likely?
A. Pain

B. Bleeding
C. Infection
D. Immobility
A post-surgery client has a normal assessment with 37.6C at 0800 hours.
Later in the day, client is he continues to be stable. Based on client's
current temperature reading, actions should be taken by a nurse?

A. Inform surgeon since discharge should be


B. Record temperature twice daily, at morning
C. Aspirin 81 mg should be administered
D. Teach client to use a spirometer and drink recheck temperature in two
hours

A 60-year-old women was transferred to the of urinary calculus.


What is the most important nursing diagnosis for
A. Obstruction of urinary tract
B. Acute pain related to obstruction
C. Infection related to urinary tract infection
D. Deficient knowledge on prevention of

A nurse, in a cardiology ward conducted a management with angina


pectoris. After she activities and exercise and health seeking behavior
concerns that they will never be able to resume. What is the most
appropriate response by the
A. Teach about different coping behaviours
B. Discuss the disease and its prognosis in

C. Use reassurance technique to calm them


D. Listen carefully to their concerns before
Which of the following vitamins is necessary for wound healing
process?
A. Vitamin A

B. Vitamin C
C. Vitamin D
D. Vitamin K

A 15-year-old girl is admitted to the hospital with diarrhea. She has been
repeatedly vomiting for now weak and lethargic. She is oriented to time
to questions appropriately. The nurse prepares temperature using an
electronic thermometer. Which measurement would be most
appropriate?
A. Oral
B. Rectal

C. Axillary
D. Tympanic
a Medical Ward manager was evaluating a blood transfusion to a patient
admitted with vehicle accident. The nurse knows that the
patient post blood transfusion. What is the time interval the student
should asse
hour of the blood transfusion process?
A. As ordered by the physician
B. As stated in the hospital policy
C. As instructed by the registered nurse in
D. As directed by the blood bank staff

A nurse educates a patient about the use of prevent atelectasis after a


surgery. Which of the following steps of the nursing process
A. Diagnosis
B. Assessment
C. Implementation
D. Evaluation

A nurse is preparing a medication order of 100 mg IV stat. The available


hydrocortisone is 5mg in 200 ml distilled water for injection.
Which of the following is the correct administered?
A. 4.5 ML
B. 3.5 ML

C. 2.5 ML
D. 1.5 ML
A 48-year-old man was admitted to the extensive anterior MI. During
the night consciousness. The cardiac monitor showed VF carotid and
found no pulse. Which is the best emergency management?

A. Cardiac thumb twice


B. Cardioversion, 50 joules
C. Electrical shock, 200 joules
D. Chest compression for five minutes

The head nurse of a Coronary Care Unit delegated the staff


a senior nurse in that unit. What initial step must the head nurse
implement before
A. Check the hospital policies for delegating tasks
B. Explain the task to the senior nurse
C. Negotiate with the senior nurse
D. Take the signature of the senior nurse
A nurse is assessing a 2 days old full-term male neon circumcision. She
observed that the circumcised area is re
a large amount of fresh blood.

Heart rate 110 /min


Respiratory rate 40 /min
Temperature 36.6C
Which of the following action should the nurse take?
A. Apply antibiotic ointment on the affected area
B. Give the infant another injection of vitamin K

C. Clean the area with betadine to prevent infection


D. Apply gentle pressure with a sterile gauze

Nurse manager prepares unit clinical operational plan. What is top


priority in the plan?
A. Infection control
B. Staff orientation
C. Quality projects
D. Safe patient care
A 22-year-old gravida 2 para 1 with gestational age 38 week admitted to
the hospital. The chief complaint is decreased the fetal non-stress test
revealed decreased variability and fetal movement. The next morning as
part of the antenatal the nurse checks the fetal heart rate by Doppler
Sonicaid decreased the fetal heart rate to less than 100 /min.
which of the following action the nurse should do first?
A. Reassure the mother that the FHR is Ok
B. Notify immediately the physician or midwife
C. Reposition the patient to left lateral position

D. Ask the mother about the pattern of fetal movement

A child was admitted to the hospital three hours ago with a injury. The
child responds appropriately, but sluggishly to drifts in and out of sleep.
Which of the following best describes this patient's level of
A. Lethargic
B. Obtunded
C. Comatose

D. Semi-comatose
A 69 year-old female patient in the Medical Word is in a semi-conscious
state . Her pancreatic cancer is metastasized to her liver and lungs and
she is admitted for supportive treatment . Her physicians discussed with
the family that she will not be given the CPR to save her life if she goes
into the cardiac arrest . Her two sons agreed but the daughter is
indecisive .Which of the following is the critical thinking behind not
providing cardiopulmonary resuscitation?
A. Unilateral judgment of health professionals
B. Refusal of patients right to treatment

C. Ethical dilemma and is indecisiveness


D. Mercy killing to ease suffering

which of the following statement described the latent phase


A. First 3 cm of cervical dilatation
B. From onset of labour to full dilatation
C. When the cervix is 100% effaced

D. Time of progress from 4 cm to 7 cm

A four-year-old child is seen in the Emergency Department fracture of


the left arm. The x-ray examination showed and healed bones.
What is your immediate action?
A. Call social services to immediately arrange foster care

B. Ask the child about the previous accidents and mana


C. Report the child abuse to the local authorities
D. Try to establish rapport and trust with the childs family
A patient was on a course of lithium carbonate drug. During the nurse
found that he complained from nystagmus, visual hallucination, and
oliguria. Which of the following drug related complications best
symptoms?
A. Overdose
B. Mild toxicity
C. Severe toxicity
D. Moderate toxicity

A 9-year-old child is postoperative after tonsillectomy. The nurse should


ask the parents to give the child which of food after discharge from the
hospital?
o A. Meat and rice
o B. Hot dog and potato chips
o C. Mashed potatoes and soup
o D. Cucumbers and tomato salad

Which of the following patients the nurse should see first?


A. Patient complaining of muscle aches and fever
B. Patient scheduled for electrocardiography
C. Patient newly diagnosed with hypertension
D. Patient with complaint of chest pain
While planning for discharge education for a mother or rickets, the nurse
knows to include the need for an adequate Which food should the
mother choose for her child?

o A. Potato and squash


o B. Orange and tomatoes
o C. Egg yolk and fish
o D. Milk and yogurt

A 65-year-old women visited the gynecological outpatient history


reveals that she had 3 pregnancies, one abortion gestational age, had 2
normal deliveries. She smokes 20 Her complaint is that she wets herself
when she cough embarrassing for her. Which of the following can be
considered as risk factors pelvic floor muscles?
A. Chronic coughing
B. Diabetes mellitus

C. Excessive spot
D. Sedentary life style
A 7-year-old child is admitted to the Emergency Department injury. The
child is oriented to the place, person and time, spontaneously, obeys
commands. The nurse is doing a ped Coma Scale (PGCS).

Which of the following score the nurse should record?


A. 3
B. 8
C. 12
D. 15

After accessing patients' medical records, which behavior nurse shows


that patients confidentiality has been breached?
A. Reviews patients medical record
B. Read patients care plan
C. Disclosing patients information

D. Documents medication administered


There is a new community nurse assigned to work in a Makkah, one of
the most multicultural cities in Saudi Arab has to interact with
multicultural population every day. What is the most important step the
nurse should do befor with the clients?
A. Asking the client about his/her cultural background
B. Conducting an appropriate culturological assessment
C. Ensuring that the client has his/her a privacy
D. Looking at the client's file to take history

An 11-year-old child has been diagnosed with Diabetes insipidus


which of the following is disorder?
A. Posterior pituitary
B. Adrenal medulla
C. Anterior pituitary
D. Adrenal cortex

A newborn is delivered by the midwife. The umbilical cord cut safely by


following necessary aseptic techniques. Furth newborn is to be taken.
which of the following intervention is the most desired?
A. Assess sucking response
B. Increase mother child bonding
C. Assess and record APGAR score

D. Keep dry and maintain thermoregulation


A primigravida mother is assisted out of the bed a few normal vaginal
delivery. She is taken to the bathroom cleaning herself and to pass urine.
She has difficulty in commode seat and is having no urge to urinate.

Which of the following intervention is the most desired?


A. Teach kegel's exercises
B. Give warm sits bath first
C. Pour warm water over vulva
D. Identify possible perineal tears

Which of the following is the appropriate nursing advice to mother


complaining of epigastric burning sensation and do use of drugs?
A. Chewing gums
B. Increase fluids at bedtime
C. Drink orange juice on getting up
D. Eat small meals every 2 to 3 hours

A nurse is assessing a 6-month-old infant that has retar reduced


responsiveness and interaction with the environm to smile or make eye
contact. The nurse notices that the attempt to hold or comfort the crying
infant. What diagnosis should the nurse anticipate?
A. Celiac disease
B. Failure to thrive

C. Cystic fibrosis
D. Growth hormone deficiency
The nurse is assessing a 2 -years-old child with Wilms surgery.
Which of the following should the nurse avoid?
A. Putting the child in lateral position

B. Palpating the child's abdomen


C. Putting the child in a private room
D. Provide mouth hygiene 30 minutes after meal

while a nurse is assessing an infant born 11 hours ago section, she


auscultated moist lung sounds. Which of the following is the most likely
interpretation?
A. Abnormal finding
B. Normal finding
C. Pneumothorax
D. Surfactant aspiration

The nurse manager received complaints from some nursing to improper


work distribution and discrimination. How should the nurse manager
handle the situation?
A. Investigate the complaints
B. Individual counselling
C. Identify list of problems

D. Continue to observe
A women was discharged from gynecological ward after gestational
trophoblastic disease (molar pregnancy). Which of the following is the
best advice to give her?

A. Never to fall pregnant again


B. To request the doctor to sterilize her
C. To consider having her uterus removed
D. To avoid falling pregnant for at least one year

the nurse is preparing to discharge an 8-month-old recovering from


gastroenteritis. Which of the following topics the nurse should include in
session?
A. Weaning
B. Toilet training
C. Accident prevention
D. Dietary requirements

A nurse is educating a diabetic mother on symptoms of. Which of the


following symptoms stated by the mother indicate education was
effective?
A. Polyuria
B. Flushing

C. Blurred vision
D. Dry mouth
An autistic child makes no eye contact, unresponsive to continuously
spins, twist and head bang. Which of the following is the priority
nursing diagnosis?

A. Risk of injury related to head banging


B. Impaired verbal communication related to physical
C. Personal identity disorder related to poor ego develop
D. Social isolation related to unresponsiveness towards

A 40-year-old women is a gravida 2,para 2 and is current conceive. Her


previous pregnancy resulted in the birth of a cleft lip and palate. The
patient is anxious and concerned pregnancies and the nurse provides
genetic counselling and
Which foods would most effectively prevent recurrence a
palate?
A. Green vegetables and citrus fruit

B. Eggs, milk and dairy products


C. Wheat, corn, rice, oats and rye
D. Beef, chicken and yellow vegetables
A mother in the postnatal Ward is being provided with the her family
and she has been instructed to be on the bed members believe that the
diet with the specific home increase the milk production and will
promote her healing.
Which of the following intervention is the most desired?
A. Respect the cultural believes and encourage activities
B. Teach and monitor for correct breast feeding practices
C. Discuss dietary plan and monitor weight gain
D. Explain post-partum care and follow ups

A nurse is caring for a newborn with cleft lip. At which age would the
nurse expect the doctor to perform?
A. 3-6 months
B. -10 months
C. 11-14 months

D. more than 14 months

A nurse is performing an assessment of a women who is delivery. Which


assessment finding would indicates a need to physician?
A. Hemoglobin of 11.0 g/dl
o B. White blood cell count of 12,000

C. Fetal heart rate of 180 beats per minute


D. Maternal pulse rate of 85 beats per minute
A 17-years-old mother presented to the primary health after delivery.
She is suffering from fatigue, anemia, fever vaginal discharge (see lab
results).

Blood pressure 81/50 mmHg


Heart rate 98 /min
Respiratory rate 26 /min
Temperature 39.6 C
Test Result Normal Values
RBC 4.6 4.7-6.1 × 1012/L (Male)

4.2-5.4 × 1012 /L (Female)


Hb 88 130-170 g/L (Male)
120-160 g/L (Female)
Which of the following is considered as the main maternal
postpartum hemorrhage?
o A. Death
o B. Candidacies

o C. Cervical cancer
o D. Uterine prolapse
A gravid 8 para 8 women has just delivered a 4.5Kg infant a
pregnancy. Which of the following is a possible complication?
A. Postpartum depression

B. Maternal hypoglycemia
C. Postpartum hemorrhage
D. Pregnancy-induced hypertension

A nurse manager includes staff in decision-making process. what unit


structure is used?
A. Divisional
B. Centralized
C. Functional
D. Decentralized

A 20-year-old primigravida, who is pregnant at 40 weeks admitted to


labor and delivery unit in active labor. Vaginal revealed that, the fetal
occiput is close to the maternal. Which of the following would the nurse
expect to be increa this fetal position?
A. Leg cramps
B. Back discomfort
C. Vaginal bleeding

D. Nausea and vomiting


A primiparous mother who has mastitis is asking the nurse feeding.
Which of the following would be the proper nursing response the
organism of mastitis?

A. It will not affect the newborn


B. It will be deactivated by salivation
C. It will not be expressed in breast milk
D. It will be killed by immunoglobulins in breast milk

Mother of nine children, three of them with congenital an down


syndrome; she is a primary school graduate, with status. She is not using
any method of family planning. So Health care nurse has referred her for
counselling. Which of the following must be focused on by the
community nurse to provide an effective health education?
A. Educate regardless realistic objectives
B. Use clear and concise language
C. Use scientific terms during explanation

D. Explain the negative consequences in the family


Home Health Care Department is newly established at a hospital. At
launch, nurses asked the charge nurse about work at this department.
Which of the following is the first step of home health care
A. Referral
B. Planning

C. Home visit
D. Assessment
A 5-year-old child is seen in the primary care clinic with headache,
malaise for about 2 days and today he has a fluids. For how long the
child should be isolated after formation of

A. 2 days
B. 6 days
C. 10 days
D. 15 days

A 1-year-old child was seen to the Emergency Department abdominal


pain, palpable sausage-shaped mass, and Intussusception is suspected.
Which of the following is the best diagnostic evaluation to
A. X-ray
B. endoscopy
C. Rectal biopsy
D. Ultrasonography

a 5-week-old newborn was admitted to pediatric Ward stenosis, the


newborn has weight loss, and projectile feeding. They scheduled
surgical repair of pyloric stenosis for Which of the following
postoperative intervention for this
A. IV fluid infant is retaining adequate amount by
B. Administration of proper analgesia until infant dis

C. Start feeding immediately after postoperative


D. Vomiting is uncommon in the first24-48 hrs
A nurse in the Neonatal Intensive Care Unit is caring for premature
newborn, is diagnosed with Respiratory Distress (RDS) and the doctor
ordered administrating surfactant, should be given by which of the
following routes?
A. Intravenous
B. Subcutaneous
C. Intramuscular
D. Endotracheal

A nurse was assigned to a patient with major depression. he isolates


himself in his room and keeps staring outside the. Which of the
following interventions takes priority?
A. Keep him until he asks for help
B. Notify him with the routine activities
C. Sit with him and offer self frequently
D. Help him to make relationship with other patient
A 23-year-old vaginal delivery primigravida mother was discomfort due
to breast engorgement on the second post The mother complained of
pain on the breast site and the able to suck the milk. Which of the
following will relief the mother's discomfort?
A. Breast binder
B. Well-fitting brassiere
C. Encourage breast feeding

D. Lactation suppressing medication

The nurse is assessing a child who has Tetralogy of Fallot observed that
the child is having clubbing in his fingernails. Which of the flowing
might be the reason for this clubbing?
A. Prolonged tissue hypoxia
B. Delayed physical growth
C. Inactive bone marrow
D. Pulmonary fibrosis
A 56-year-old man with a history of COPD was rushed to the
Emergency Department with chest pain, shortness of breath, fever and a
cough. Upon assessment, crackles can be heard over the low lobes. The
patient looks pale and lethargic (see image).

Blood pressure 110/70 mmHg


Heart rate 130 /min
Respiratory rate 9 /min
Temperature 38.1C

Oxygen saturation 85 %
What is the most likely condition?
A. Bronchial asthma
B. Respiratory failure
C. Pulmonary embolism
D. Myocardial infraction
A 12- year- old boy was brought to the Emergency Department
respiratory arrest due to drowning. Cardiac resuscitation was what is the
major complication that might happen if the patient treated after
drowning quickly?
A. Sepsis
B. Alkalosis
C. Acidosis
D. Hypothermia

A 56-year-old man was admitted with complaint working for three days.
The nurse is preparing to administer infusion of saline 1000 ml over six
hours. What is the hourly infusion rate that the nurse needs to infusion
therapy in mL/hour?
A. 155
B. 167

C. 190
D. 217
While administering medications, the nurse gives an anti-hypertension
medication to the wrong patient (see table)
(A) Document the entire scenario

(B) Document vital signs


(C) Notify doctor
(D) Notify doctor if blood pressure is below 90/50 mmH
(E) Induce vomiting
(F) Administer antidote
(G) Monitor the patient

What sequence should the nurse follow?


A. A, D, F, G
B. B, C, G, A
C. B, D, E, F
D. B, E, F, G

A 47-year-old man is diagnosed with tuberculosis (TB) and negative


pressure room. Which of the following should wear a facemask?
A. Patient's health care providers
B. All people who enter the patient's room
C. Person has close contact with the patient
D. Family members who are at risk for infection
An ICU nurse reviews the chart of a 47-year-old man patient mechanical
ventilator for a long time. Arterial blood gas results (see lab results).
Test Result Normal Values

ABG HCO3 24 22-28 mmd/L


ABG PCO2 10.66 4.7-6.0 KPa
PH 7.16 7.36-7.45
ABG PO2 6.13 10.6-14.2 KPa
SA O2 81 95-100 %
What condition the patient is experiencing presently?

A. Metabolic acidosis
B. Metabolic alkalosis
C. Respiratory acidosis
D. Respiratory alkalosis

An 81-year-old bed ridden patient in the Geriatric Ward was pyretic


through his nasogastric tube. After an hour, the patie deep sleep, his
breathing pattern changed and he did not rep calling his name. Which of
the following should be the immediate nursing inte
A. Call the physician to examine the patient
B. Check for the correct dosage given
C. Check vitals and inform physician
D. Try to wake up the patient
A patient who is admitted denies any medication allergies. medical
records state, "No known drug allergies." A medic been sent to the
pharmacy for gentamicin. The pharmacist record in the pharmacy of an
allergy to the medication. Which of the following is the best way for the
nurse to pro
A. Administer the medication with Benadryl (Diphen
B. Notify the doctor of the allergy and request new orders
C. Investigate further then call the doctor for clar
D. Give the medication as ordered by the physician

A 65-year-old man is undergoing pre-operative preparation for


endoscope procedure in which the physician will visualize the lar and
distal part of the small bowel with a camera attached to the flexible tube.
Which of the following positions is the most appropriate?
A. Left lateral Sim's
B. Right lateral recumbent
C. Trendelenbreg
D. Prone
A 72-year-old women is admitted in the Neurosurgery Ward hairline
fracture on her right cranial region due to a fall 24 had a brief
unconsciousness at the time of the fall and has bruises on her face and
right arm. She is complaining of pain at her right hip.
Blood pressure 90/58 mmHg
Heart rate 102 /min
Respiratory rate 27 /min
Temperature 37.8C
What nursing diagnosis should be focused on first?

A. Risk of fall and injury due to senility


B. Self-care deficit due to inability and weakness
C. Risk of altered consciousness due to head injury
D. Hyperthermia due to injury response and infection

A registered nurse in the Intensive care Unit performed Glasgow coma


scale assessment on a 50-year-old man who sustained head injury 24
hours ago. The recorded Glasgow Coma score was 10. which one of the
following actions should be taken immediately by the nurse?
A. Inform the registered nurse in charge of the nursing t
B. Protect the patient by raising the two side rails

C. Inform operation theatre for emergency surgery


D. Perform neurologic assessment every 15 minutes
The nurses in the Male Medical Unit took a signature on the from a
patient who needs an abdominal CT with contrast. What is the best
nursing practice?

A. Consent should be taken after the procedure


B. Verbal consent is acceptable for this procedure
C. Consent should be taken from the patient's relatives
D. Consent should be taken according to the organization's

A 48-year-old patient in the male Surgical Ward had his gall bladder
removed through laparoscopic cholecystectomy 24 hours ago. While
evaluating his general condition, the patient appears lethargic and
complains of severe nauseated feeling along with discomfort in the
abdomen. What nursing problem needs to be prioritized?
A. Disturbed metabolism due to higher energy demand
B. Weak and lethargic due to low food and fluid intake
C. Nausea and vomiting due to slower gut movement

D. Impaired comfort related to post-surgical effects

The nurse in the Critical Care Unit is preparing a plan of care for a
postoperative cardiac transplant client. Which of the following maintain
highest priority intervention?
A. Apply sequential compression devices to prevent deep vein
thrombosis
B. Assist the client in changing positions slowly to prevent hypotension
C. Encourage coughing and deep breathing to prevent paranoia
D. Use careful handwashing and aseptic technique to prevent infection
a nurse is caring for a patient who had Coronary Artery bypass Graft
Surgery (CABG) four hours ago. The nurse notices that the patient has
increased confusion and is restless. The patient reports nausea,
weakness, and paresthesia in the extremities (see lab results).
Test Result Normal Values
Sodium 145 134-146 mmol/L
Potassium 6.8 3.5-5.2 mmol/L
Calcium 2.50 2.15-2.62 mmol/L
Which of the following is the best medication?

A. Naloxone (Narcan)
B. Hydralazine (Apresoline)
C. Potassium chloride (KCI)
D. Sodium polystyrene sulfonate (Kayexalate)

A nurse is assigned to care for a patient with physical immobility due to


right knee injury. the nurse is preparing to write nursing diagnosis.
Which of the following is a priority for nursing diagnosis?
A. Pain
B. Hygiene
C. Dehydration
D. Skin integrity
A postoperative patient has on order to discontinues the foley 11:30 AM,
the nurse removes the Foley intact. At 1:00 PM, the patient complains of
feeling the need to urinate and voids 30 cc. The nurse should assess the
patient for signs of which of the following?
A. Urinary retention
B. Urinary tract infection
C. Cystitis
D. Haematuria

An infection control nurse is responsible to conduct the surveillance on


how the hospital staff take precautionary measures against flu, based on
her findings, she is responsible to conduct the awareness raising for the
doctors and nurses in the in-patient areas. Which of the following is the
goal of the nursing practice here?
A. Infection control management
B. Research and survey
C. Primary prevention
D. Health education
A 17-year-old women went to the Emergency Department complaining
of fever, sore throat, and hoarseness of voice. Laboratory tests were
done (see lab results).

Test Result Normal values


Hb 120.5 120-158 g/L
HCT 0.39 0.36-0.47 g/L
RBC 4.0 3.8-5.1 × 1022/L
WBC 14.2 4-10.5 × 109/L
Throat culture positive Beta Haemolytic Non found Streptococcus

Which of the following would be the possible complication for the


patient condition?
A. Cellulitis
B. Bacterial cholangitis
C. Infective endocarditis
D. Rheumatic heart disease

A patient with a colostomy complains of itching of the peris tornal skin.


On assessment, the skin is covered in a red rash with white patches
visible. what is the most likely cause of this condition?
A. Not changing the pouch regularly
B. Candidiasis
C. Consuming acid-producing foods

D. Dehydration
A patient has a defect with the beta cells of the Islet of Langerhans.
Which of the following should the nurse most likely expect this patient
to exhibit?

A. Anemia
B. Appendicitis
C. Cholelithiasis
D. Hyperglycemia

A post-operative patient who underwent an abdominal procedure


requests a pain medication from the nurse and rates the pain at a level
nine. There a standing order for narcotic administration. When the nurse
opera the narcotic box and performs a count, the number of pills
remaining in the box is different than the number of pills recorded on the
sheet. What is the most appropriate initial nursing action?
A. Notify the nursing supervisor
B. Write the finding on the narcotic sheet

C. Administer the patient's requested medication


D. Identify the last nurse who used the narcotic box
A 60-year-old man is being discharged from the post-operative Care
Unit following a transurethral resection of the prostate. The nurse
provides discharge information regarding the care of the bladder
catheter. Which method would be most effective in bladder retraining
for this patient?
A. Scheduled urination every 2-3 hours
B. Limit fluid intake before sleeping time
C. Perform pelvic floor exercises daily
D. Increase fluid intake during the day time
The unit nurse conducts initial assessment and observes the dressing is
wet and requires change. The patient looks drowsy with warm skin
(see lab results)

Blood pressure 110/70 mmHg


Heart rate 99 /min
Respiratory rate 24 /min
Temperature 38.8C
Test Result Normal Values
RBC 5.5 4.7-6.1 × 1012 /L (male)

4.2-5.4 × 1012 /L (female)


Hb 133 130-170 g/L (male)
120-160 g/L (female)
WBC 25.5 4.5-10.5 × 109

What is the most likely underlying diagnosis?


A. Sepsis

B. Gangrenous tissue
C. Acid base imbalance
D. Hyperthermia secondary to infection
A 16-year-old girl was found unconscious in her home by her mother.
She brought to the hospital by ambulance. The patient has no previously
known medical history. On arrival, the nurse performs an assessment for
level of consciousness and notes a Glasgow Coma Score of five. The
nurse notes the patient's breathe smells like acetone or fingernail paint
remover. What is the priority therapeutic goal?
A. Increase blood glucose
B. Increase serum osmolarity
C. Increase circulatory volume

D. Decrease intracranial pressure

Before preparing a client for the first surgical case of the day, a part-time
scrub nurse asks the circulating nurse if a three-minute surgical hand
scrub is adequate preparation for this client. Which response should the
circulating nurse provide?
A. Inform the nurse that hand scrubs should be three minutes between
cases
B. Direct the nurse to continue the surgical hand scrub for a five
minute duration
C. Validate that the nurse is implementing the OR policy for a surgical
hand scrub
D. Ask the experienced nurse to perform this scrub since it is the first
one of the day
A nurse noticed that the digit probe of the pulse oximeter that is attached
to the right forefinger of a patient is not reading. Which of the following
is the most appropriate action that the nurse must take at this time?

A. Change the location of the pulse oximeter


B. Get a new pulse oximeter sensor probe other than a digit probe
C. Maintain position and check it after two hours
D. Assume oxygen saturation is at normal level

During skin assessment of a patient the nurse observed a skin described


as round solid mass beneath the skin (see image). Which of the
following terms is best to describe the lesion?

A. Cyst
B. Papule
C. Pastule

D. Blister
The nurse is assessing a 65-year-old patient, who reports the fatigue,
weight loss, night sweats, and a productive cough with thick sputum.
The nurse should immediately initiate isolation precautions for which of
the following?
A. Influenza
B. Pertussis
C. Bacterial pneumonia
D. Pulmonary tuberculosis

The nurse is planning for a discharge teaching plan for a family of a 30-
year-old man with AIDS in relation to sanitation practices. Which of the
following statements should the nurse include in her instructions?
A. Wash used dishes in a hot soapy water
B. Boil the dishes the patient used for 30 minutes and them wash with
soap

C. Have the patient use disposable eating tools so it can be discarded


after
D. Soak the kitchen tools in hot water for 24 hours before washing with
soap
A nurse is preparing to administer an intravenous infusion of 2000 ml of
Ringer's lactate over 12-hours. The administration set has a chamber that
delivers 15 drops per ml. How many drops per minute should be
administered?
A. 14
B. 24
C. 42
D. 56

a nurse is caring for a patient who had a cerebrovascular accident 30


minutes ago with residual right-side hemiparesis. The nurse places a
trochanter roll extending from the patient's iliac crest to the mid. What is
the most likely complication?
A. Adduction of the leg

B. External rotation of the hip


C. Muscle spasms in the thigh
D. Flexion contractures of the knee
A 72-year-old patient presents to the medical clinic drooling with a
blank, mask like facial expression and a high pitched, monotone, weak
while When walking, the patient does not swing the arms normally, but
holds them stiffly rigid.
What is the most likely diagnosis?
A. Parkinson's disease
B. Substance abuse
C. Traumatic brain injury
D. Transient ischemic attack

Which of the following tests measures the total quantity of prothrombin


in the blood and monitors the effectiveness of warfarin sodium
(Coumarin) therapy and prolonged deficiencies in the extrinsic factor?
A. Thrombin time

B. Prothrombin time
C. Partial prothrombin time
D. Activated partial prothrombin time
A 32-year-old women is brought to the Emergency Room severe
abdominal pain at the naval. She appears restless, agitated due to
swollen, bluish and painful protrusion from the umbilicus. Which
problem should the nurse address first?
A. Ineffective tissue perfusion
B. Risk of spread of infection
C. Anxiety and restlessness
D. Pain and discomfort

A nurse is preparing scheduled medications due at 6 pm. If a doctor


orders paracetamol tab 1g QID, and it was supplied from the pharmacy
in 250 mg tablets. Which of the following is the most appropriate
nursing actions?
A. Ask the pharmacy to provide 1g tablets
B. Call the doctor to recheck the dosage

C. Give the patient four 250 mg tablets


D. Hold the medication and document in nursing notes
A patient diagnosed with septic shock had an upward-trending glucose
level (180-225 mg/dL) requiring control with insulin. the patient's
spouse asks why is insulin needed as the patient does not have diabetes.
Which of the following is the most appropriate nursing response to
educate the patient's spouse?
A. "It is common for critically clients to develop type diabetes. We give
insulin to keep glucose level under control (less than 140 mg/dL)"
B. " Patient had diabetes before, you just didn't know it. we give insulin
to keep glucose level in the normal range (70-110 mg/dL)

C. " Increase in glucose is a normal response to stress by the body.


We give insulin to keep the level 1t 140-180-mg/dL"
D. " Increase is common in critically ill clients and effects their ability to
fight off infection. We give insulin to keep the glucose level in the
normal range (70-110 mg/dL)"
A 70-year-old man presents to the clinic with difficulty sleeping at night.
He has not had a good night's rest for several months and feels
exhausted. He needs to place three pillows behind his back in order to
sleep. Examination of the lungs reveals crackles and when Auscultation
of the heart confirms an S1 gallop. Which of the following is the most
likely underlying health problem?
A. Asthma
B. Pulmonary stenosis
C. Right-sided heart failure

D. Left-sided heart failure

A 69-year-old obese women with a pelvic fracture has been immobile


for four days. The patient is now anxious, dyspneic, and domaining of
substernal pain. The patient's capillary refill is greater than seconds.
Heart rate 122 /min
Respiratory rate 26 /min

Temperature 38.2C
Which of the following the nurse suspects for this patient?
A. Panic attack
B. Pulmonary embolism
C. Aspiration pneumonia
D. Pneumothorax
A nurse is trying to secure a peripheral IV access in a traumatic patient
who is deteriorating. The nurse has two failed attempts. What should the
nurse do next?

A. Activate code
B. Use Interosseous needle
C. Consider arterial line
D. Call another nurse who is more experienced

A 70-year-old women with a history of hypertension companies of lack


of appetite due to bloating and constipation. The abdomen is distended
and examination shows a positive fluid wave. palpation of the abdomen
confirms guarding and tenderness over the right upper quadrant, lower
leg edema 3+ and pitting bilaterally are also present. She has 4.5
kilograms since her last appointment three weeks before.
Blood pressure 164/92 mmHg

Heart rate 90 /min


Respiratory rate 26 /min
Temperature 37.1C
Oxygen saturation 92% on room air
What is the most likely underlying health problem?
A. Hepatic congestion
B. Pulmonary hypertension
C. Left-sided ventricle failure
D. Splenomegaly
Which of the following statements best describes for patients with
myopia?
A. They must hold a paper at arm's length or greater to read it
B. They can see close-up objects clearly, but distant objects are out
of focus
C. They can see distant objects clearly, but close-up objects are out of
focus
D. They are unable to focus horizontal and vertical rays of light on the
retina simultaneously

A 45-year-old of man was admitted to the Surgical Ward after removed


of pancreatic cyst. The registered nurse checked the post-operative
orders Ondansetron PRN was prescribed for the patient. Which of the
following complaints from the patient will require the registered nurse to
serve the Ondansetron after 12 hours of pos-operative period?
A. Wound pain
B. Nausea and vomiting
C. Congestion of flame

D. difficulty in passing urine


a nurse was newly appointed to work in the Surgical Ward. She was
assigned to take care of an obese patient who just had an open
laparotomy. Her mentor told her to pay attention to the surgical
associated with obesity. Which of the following postoperative
complications the nurse should consider as a high-risk complication?
A. lunger
B. Gas pain
C. Hemorrhage
D. Impaired wound healing
A 75-year-old patient was bed ridden for the last one year. She was
incontinent, restless, delirious and was trying to pull her nasogastric tube
out. Her arms were restrained, her forearms had bruises and her lips
were dry and cracked.
Blood pressure 100/64 mmHg
Heart rate 102 /min
Respiratory rate 28 /min
Temperature 39.2C
Which of the following nursing intervention should be done before
giving each nasogastric feed?
A. Pull to check the tube is in place
B. Check the tube for patency
C. Aspirate the content
D. Insert water
A 50-year-old man presents to the Emergency Department after having
suffered a fall while practicing mountain biking. He appears confused
and disoriented. His friend reports that he had been thrown over the
handle bars and hit his head against a rock.
Blood pressure 108/66 mmHg
Heart rate 102 /min
Respiratory rate 22 /min
Temperature 37.2C
Oxygen saturation 99% room air

Which initial intervention is the most important?


A. Intravenous infusion
B. Oxygen administration
C. Cervical immobilization
D. Trendelenburg position

A 46-year-old women is admitted in the Medical Ward for systemic


lupus erythematous, complaining of joint pain with dull aching pain in
the number region. She has butterfly rashes on her face. Which of the
following should be the first nursing intervention for this patient?
A. Skin care and cortisone ointment as ordered
B. Pain relieving measures and physiotherapy
C. Assess back pain and monitor urine output

D. Analgesics as ordered warm environment


The Surgical unit nursing staff were instructed to follow the hospital
protocol by reporting cases of any patient's illnesses that are developed
during their hospitalization especially the post-surgical wound infectors.
Why must the nurses implement this protocol?
A. Control unnecessary hospitalization
B. Assess hospital infection rate
C. Report nosocomial infection
D. Improve reporting system

A post-operative nurse assesses the newly formed stoma and skin of a


patient who is one day post-operative following a proctocolectomy with
the formation of a permanent ileostomy. Which of the following clinical
findings would necessities immediate care?
A. Bright red and moist stoma
B. Dark red and purple skin
C. Bloody liquid in pouch

D. Ulcerations with a rash

Which of the following is the best action to effectively increase surge


capacity?
A. Adapt outpatient departments for inpatients
B. Bring the injured people into the green zone

C. Provide care for the injured people with the available staff
D. Use the available supplies as it is difficult to arrange for additional
A 46-year-old patient is admitted in the female Medical Ward for seven
back pain, which is graded 6 on the scale of 10. Her vital signs are take
with the slight elevation in her blood pressive. The patient has refused
eat lunch which is a low sodium diet. The attending nurse has
documented that patient is uncooperative and has refused to eat the
lunch. What nursing intervention needs attention first?
A. Arrange for alternative diet
B. Teach importance of the low salt diet
C. Take appropriate measures to relieve pain

D. Inform the physician for elevated blood pressure

a 30-year-old man was brought to the hospital by ambulance after falling


from a height of 10 meters. He was mechanically ventilated on after a
Glasgow Coma Score showed a level of six. The nurse is to observe for
any changes in perceptual, sensory or cognitive. Which of the following
is an expected patient's response at this time?
A. Slowly obeys commands
B. Exhibits no motor response
C. Reacts towards painful stimuli

D. Uses incomprehensible words


After a code blue, a doctor announced the death of a patient. A signature
other begins to show signs of decreased level of consciousness. What is
the priority intervention of the nurse at this time?

A. Offer a meal
B. Ensure safety
C. Set up an IV line
D. Offer a glass of water

A male client has received a prescription for orlistat for weight and
nutrition management. In addition to the medication, client states plans
to take a multivitamin. what teaching should a nurse provide?
A. Be sure to take the multivitamin and the medication at leasr two
hours apart
B. As a nutritional supplement, orlistat contains all the recommended
daily vitamins and minerals

C. Multivitamins are contraindicated during treatment with weight


control medications such as orlistat
D. Following a well-balanced diet is a much healthier approach to good
nutrition than depending on a multivitamin
A 70-year-old man underwent a colectomy two days prior. In the past 24
hours, his blood pressure has decreased and heart rate risen. He now
weights two kilograms more than he did prior to admission. Urine output
from the indwelling catheter is 30 ml per hour and he has pitting +
bilaterally in the lower extremities. The abdomen appears distended and
is firm to the touch.
Blood pressure 112/62 mmHg
Heart rate 110 /min
Respiratory rate 20 /min

Which of the following is the best nursing intervention?


A. High flow oxygen by mask
B. Position in Trendelenburg
C. Hemoglobin and hematocrit
D. Intravenous hypotonic solution

A newborn has a diagnosed Developmental Dysplasia of the Hip (DDH)


and is using a Pavlik Harness as treatment. Which of the following
mechanical factors is associated with DDH?
A. Intrauterine breech position
B. Caesarean section
C. Small infant size
D. Single fetus
A 10-month-old infant is admitted with intestinal obstruction.
Colostomy was performed. Which of the following instruction should be
given to the parents about colostomy care?

A. Use baby powder after stoma cleaning


B. Empty the pouch when it is completely full
C. Avoid tight diapers around the infant abdomen
D. Use baby wipes to clean the skin around the stoma

A nurse who works in the surgical unit at one of the hospitals was asked
by the home health care nurse to make a home visit to a patient with
colostomy, who had been discharged the previous day in order to give
him a follow-up care and education. which of the following nurses
should do the assigned task?
A. Critical care nurse
B. Psychiatric nurse
C. Surgical nurse
D. Community nurse
A multipara mother complained of small vulva with swelling following
vaginal delivery of a baby weight 3.8 Kg. What is the initial nursing
action should the nurse advise the mother to perform?

A. Apply ice pack


B. Maintain bed rest
C. Administer analgesics
D. Encourage fluid intake

A 13-weeks-pregnant, multi gravida women is anxious and apprehensive


she has five children and is not willing to continue with this pregnancy.
She is requesting the midwife to abort the fetus. She is underweight
malnourished and is over worked. BMI 17 Kg/m2. What intervention is
desired immediately?
A. Admission and intravenous line management for induction

B. Family planning and birth control measures


C. Dietary management and supplements
D. Support, reassurance and counseling
A clinical picture of an antisocial personality disorder is a "pervasive
pattern of disregard for and violation of rights of others, deceit, and
manipulation". What will be the priority of care?

A. Safety
B. Set limitations
C. Behaviors therapy
D. Reduction of environmental stimuli

A man was restless and observed to have an increased physical activity


after a heated argument with another person. After the assessment, the
following data was gathered: agitated, diaphoretic, distorted. What is the
patient's level of anxiety?
A. Mild
B. Panic
C. Severe
D. Moderate

A 2-day-old newborn is admitted to the nursery. While the nurse is


administrating oral feeding, the milk returns through the child's nose and
mouth and the infant become cyanotic. Which of the following condition
the newborn should have?
o A. Anorectal malformation

o B. Tracheoesophageal fistula
o C. Cleft lip and palate
o D. Cardiac condition
A nurse is providing postpartum care for a GSP4 mother who had a
rapid labor of an infant weighing 4000 gm. Assessment revealed a
boggy uterus, heavy lochia and stable vital signs. After fundal massage
and bladder evacuation. the fundus remains soft. which of the following
is the most appropriate next nursing action?
A. Inform the physician
B. Reassess the vital signs
C. Continue fundal massage
D. Take venous blood sample

The nurse is assessing a child (an infant) with pyloric stenosis.


which of the following is likely to note?
A. Diarrhea
B. Projectile vomiting
C. Swallowing difficulties
D. Currant jelly like stool

When is the ideal time to administer analgesia to a woman in labor?


o A. A soon as she requests analgesia
o B. When labor is well established
o C. When the women enter into transition phase

o D. when the women progress from latent to active phase


A term baby boy has diagnosed with Down syndrome. Physical
examination revealed flattened nose, low set ears, upward slanting eyes,
single palmer crease. Which of the following is the most common
congenital anomaly associated with this disease?
A. Developmental dysplasia of hip (DDH)
B. Congenital heart disease
C. Hypospadias
D. Pyloric stenosis

which of the following is the most appropriate action for a head nurse
starting at a new hospital?
A. Make immediate change at the unit
B. Plan and coordinate new strategies
C. Assess unit activities for at least three months
D. Ask about the previous head nurse managerial style

A nurse is teaching a 26-year-old primigravida who are 33 weeks


pregnant on the how to use a kick chart. Which of the following
statements will indicate that she understands the nurse's teaching?
A. Fetal movements must be counted three times per day
B. Fetal movements are felt best when the woman is on her right side
C. Fetal movements is a reassuring sign which indicates that the
fetus is healthy
D. The kick chart is used to record fetal movement for the first time
during pregnancy
The nurse have been teaching a new mother how to feed her infant who
was born with a cleft lip and palate before surgical repair of the defect.
Which of the following action from the mother indicate that the nurse
teaching has been successful?
A. Burping the baby frequently
B. Prevent the infant from crying
C. Placing the baby flat during feeding
D. Keep the infant prone following feedings

A 5-year-old child is seen in the primary care clinic with mild fever,
headache, malaise for about 2 days and today he has a rash filled with
fluids. which of the following is the best suggested diagnosis?
A. Chicken pox
B. German measles
C. Measles
D. Scarlet fever

which of the following is the most appropriate action for a nurse


Manager starting at a new hospital?
A. Observe staff while keeping presence and role unknown
B. Plan and coordinate new strategies with top level management

C. Introduce self and observe unit activities for at least three months
D. Meet with each staff member separately about needed unit charges
The nurse was planning care for a 25-year-old primigravida post-partum
mother who had engorgement due to poor feeding technique. the left
breast appeared red and swollen and was diagnosed as. Which of the
following is the best education for the mother?
A. Avoid wearing brassiere
B. Begin suckling on the right breast
C. Stop pumping milk from the left breast
D. Take antibiotics till the soreness subsides

Mother of nine children, three of them with congenital anomalies and


one down syndrome; she is a primary school graduate, with low
financial status. She is not using any method of family planning.
Accordingly, the primary health care nurse has referred her for
counseling. Which of the following is a barrier facing the team
responsible for providing health services to handicapped individuals?
A. Sensory limitations

B. Rigid rules
C. Developmental disabilities
D. Deafness & hearing limitations

An infant is born with spina bifida. Which of the following complication


is always found in these infants?

A. Hydrocephalus
B. Craniosynostosis
C. Meningitis
D. Cerebral policy
A 71-year-old male was diagnosed with subdural underwent burr hole
craniotomy for subdural hernatoma days ago. In order to detect the sign
of meningitis as one of which of the following indicates the patient has
meninge?
A. Negative Kernig's signs
B. Positive Brudzinski's sign
C. Absence of nuchal rigidity
D. Glasgow comma scale of 14 points

Azithromycin is prescribed for an adolescent female who has pneumonia


and recurrent chlamydia. What information is most important for the
nurse to provide?
A. Use two forms of contraception while taking this drug
B. Have partners screened for human immunodeficiency
C. Decrease intake of high-fat foods, caffeine, and alcohol
D. Report a sudden onset arthralgia to the healthcare pro

A nurse is about to perform venipuncture to initiate intravenous therapy


with 0.9% normal saline solution. Before venipuncture, the nurse
collects the require supplies a Perform certain action. Which of the
following actions is the most important?
A. Apply a tourniquet below the site of venipuncture

B. Place an arm board at the joint above the venipuncture


C. Apply cool compress over the vein to be used for venipuncture
D. Inspect 0.9% normal saline solution date of expiry
A nurse evaluates dietary practices of a patient who had been as having
acute glomerulonephritis. The patient verbalizes following the
appropriate diet for acute glomerulonephritis. Which of the following
diet verbalized by the patient is approve diagnosis?
A. Restrict fluid intake
B. Restrict dietary protein
C. Increase intake of low-fiber foods
D. Increase intake of sodium-rich foods A

70 years-old woman is admitted to the cardiac care unit with atrial


fibrillation and is receiving intravenous diltiazem and heparin. What is
the most likely nursing diagnosis?
A. High risk for infection
B. High risk for impaired gas exchange
C. High risk for decreased cardiac output
D. High risk for disturbed sensory perception
A 28-year-old female in the Critical Care Ward has fluid her pleural
space due to tuberculosis. She is planned for (see lab results).
Test Result Normal Values

Hb 90 120-160 g/L
HCT 0.25 0.37-0.48
Platelets count 90 150-400 × 109
Which problem the client is highly at risk to develop?
A. Fever and chill
B. Fear and anxiety

C. Bleeding tendency
D. Coughing and chest infection

Following lumbar surgery, a patient has a 4-millimeter incision. The


incision is clean and the edges are well approximate. This type of tissue
healing is classified as which of the following
A. Primary intention
B. Secondary intention
C. Tertiary intention
D. Superficial epidermal
A 45 years-old patient was to be transferred to the operating room for
cholecystectomy procedure under general anesthesia. pre-an aesthesia
consultation was done and vital signs were stable. Which of the
following is the most important for the nurse to transferring the patient
to operating room?
A. Safety of patient’s belonging
B. Patient’s identification
C. Signed consent form
D. Skin preparation

A nurse received the serum digoxin level result for the patient the day,
and notes that the result is 2.6 ng/mL (see lab result)
Test Result Normal Value

Digoxin (men) 2.6 0.8-2 ng/mL


Which of the following nursing actions is the most important?
A. Notify the physician
B. Check previous vital signs of patient
C. Record normal value on nursing notes
D. Administer scheduled dose of medication
The nurse is assigned to care for a patient with Paget’s disease
complaining of bone pain. Which of the following conditions is this
patient high risk for?

o A. Hypomagnesaemia
o B. Hypernatremia
o C. Hypercalcemia
o D. Hyperkalemia

Upon respiratory assessment of a 42 years old man, crackles could be


heard on auscultation and dull note on chest per patient is febrile with
excessive production of sputum.
Blood pressure 130/75 mmHg
Heart rate 110 /min
Respiratory rate 24 /min

Temperature 38.3
Oxygen saturation 98
Which of the following is the most likely underlying problem?
A. Asthma
B. Pneumonia
C. Pneumothorax

D. Tuberculosis
A hospitalized 72-years-old man who uses a walker is received
medication and must use the bathroom several times each night. To
promote the safety of the patient, which of the following appropriate
nursing action?
A. Keep the side rails up
B. Leave the bathroom light on
C. Provide a bedside commode
D. Withhold the patient’s diuretic medication

A 44-year-old woman presented to the Emergency Department pain and


redness. She was diagnosed with uveitis related infection. The doctor
prescribed dexamethasone eye drops. Where is the correct place to instill
eye drops?
A. Above the eyelid
B. Directly into the cornea
C. Into the conjunctiva sac

D. Outside the conjunctiva sac

The nurse is preparing to transfer a 55-year-old patient on a the CT-scan


unit. The patient is too heavy for the nurse to transport, nurse went to get
an assistive device to transfer the patient. Which of the following
transfer device is the most appropriate for the nurse to use?

A. Board
B. Handle
C. Trapeze
D. Mechanical lift
A 29-year-old man is in the Surgical Ward on his first postoperative
thyroidectomy. He appears drowsy but he is able to respond to question
by nodding head. He is developing mild dy restlessness. What is the
initial recommended goal of care?
A. Monitor vital signs of thyrotoxic storm
B. Assess for bilateral vocal fold mobility
C. Monitor for swelling on the neck
D. Monitor for vocal cord paralysis
A man patient admitted to the Medical Unit was diagnosed with
thrombosis complaining of pain on both legs. Which of the following
nursing diagnosis most likely describes problem?
A. Risk for injury
B. Fluid volume excess
C. Electrolyte imbalance
D. Impaired tissue perfusion

A nurse is conducting health teaching to a group of members about the


ways to lessen the dietary risk factors Which instruction should the nurse
include in the health teach
A. Limiting amount of salt in food
B. Lessening intake of high-fiber foods
C. Avoiding consuming too much fatty foods

D. Avoiding the use of bran or bran-containing products


An insulin dependent diabetic patient is admitted for ketoacidosis (see
table).
(A) Knowledge deficit related to Diabetes Mellitus

(B) Anxiety related to new diagnosis of Diabetes Mellitus


(C) Altered nutrition, more than body requirement
(D) Altered health maintenance related to elevated blood glucose
(E) Alternation in fluid and electrolyte balance related to ketoacidosis
(F) Risk for infection related to chronic disease
Which of the following has the highest priority?

A. A, C
B. E, F
C. D, E
D. B, E

A nurse is assigned to care for a group of patients in the medical


expected to review the medical records of these patient. What patient is
at risk for excess fluid volume?
A. Patient with ileostomy
B. Patient taking a loop diuretic
C. Patient with chronic renal failure
D. Patient hooked to gastrointestinal
A client was admitted in the Emergency Room due to mild metabolic
acidosis associated with dehydration and potassium. the doctor
administered Hartmann’s (lactated Ringer’s) intravenous fluid and
electrolyte replacement. Which of the following elements of the lactated
Ringer’s sol highest value?
A. Calcium
B. Sodium
C. Potassium
D. Magnesium

A client with a blunt trauma underwent an exploratory lap repair an


intra-abdominal injury. A nasogastric tube attached suction and two
closed-wound Hemovac abdominal drains. He is receiving an
intravenous infusion of Ringer’s lactate epidural with continuous
morphine. Twenty-four-hour post develops hypotension, tachycardia,
oliguria and sever nausea. Which of the following is the priority nursing
diagnosis?
A. Nausea
B. Risk for infection

C. Deficient fluid volume


D. Impaired urinary elimination
At 6:00 PM while admitting a woman for a scheduled repeal section (C-
section), a client tells a nurse that she drank a cup 4:00 PM because she
wanted to avoid getting a headache. Which action should the nurse take
first?
A. Ensure preoperative lab results are available
B. Start prescribe IV with lactated Ringers
C. Inform an aesthesia care provider
D. Contact client’s obstetrician

A nurse reviews a plan of care for an elderly client diagnosed with


Chronic Obstructive Pulmonary Disease (COPD) and limited mobility.
Notes that a physical therapist has indicated a change in the to progress
ambulation from 1:00 to 2:00 feet twice a day. Which action is necessary
to ensure that the client’s needs are?

A. Cancelling physical therapy referral


B. Informing physician about the physical therapist’s plan
C. Instructing physical therapist not to ambulate the client
D. Informing physical therapist of client’s respiratory stat progressing
ambulation
A nurse is preparing to administer 25 mg Iron Dextran inject patient
with iron deficiency anemia. The nurse knows this d to subcutaneous
tissue and wants to administer the drug safely. Which of the following is
the best administration techniques?
A. Z-track
B. Deep IM
C. Use large gauge needle
D. Insert needle at 45 angles

The physician has written an order for the nurse’s assignee have a 24-
hour urine collection sent to the laboratory for special. Which of the
following should the nurse realize prior to urine?
A. Start the urine collection at either 12:01 a.m. or 12:01pm
B. Provide enough sterile receptacles for the urine collection
C. At the start of the collection period, have the patient discard this
urine

D. Inform the patient that they must save all urine for 24 beginning at
12:01 a.m.
A 68-year-old diabetic women undergoes a below knee amputation to
vascular insufficiency and infection. On admission to care unit, the nurse
makes the priority diagnosis: Risk for ineffective peripheral tissue
perfusion. Which intervention is most appropriate?

A. Ensure adequate pain relief


B. Elevate residual limb on a pillow
C. Administer oxygen by facemask
D. Apply ice to the stump
A 26-year-old married woman is admitted in the plastic surgery for the
correction of burn strictures and skin grafting on her neck and face under
general anesthesia. While discussing the treatment with her, the plastic
surgeon explained that she will have a series of surgeries but she needs
to be on family planning until the treatment is completed. the patient
asked the nurse whether she will be normal again. What initial
assessment is required?
A. Detailed history and physical examination
B. Patient’s acceptance for the treatment plan

C. Need for psychological support to reduce anxiety


D. Family’s involvement and consent for her treatment

A 40-year-old client underwent an exploratory laparotomy an aesthesia.


An assessment of the abdomen 36-hours showed abdominal distension
and an absence of bowel quadrants. Which complication is most likely?

A. Paralytic ileus
B. Hemorrhage
C. Ruptured colon
D. Intussusception
A 2-year-old diabetes type II patient is admitted to amputation of the
right foot due to gangrene and pre-operatively, the patient rates the pain
as 9 (using the 1-10). How can this patient’s phantom (spirit) pain best
be controlled?
A. Post-operative elevation of limb
B. Apply pressure bandage to stump
C. Control pain pre-operatively
oD. Apply ice to the site for twenty minutes

A patient is scheduled for a total hip arthroplasty. The nurse reviews the
chart and notes the following: serum potassium 2.8 mEq/L, AB positive
blood type, and elevated ST segment electrocardiogram. Which of the
following would be the most appropriate action to do next?
A. Report abnormal diagnostic results to the surgeon
B. Review the patient consent for the surgical procedure
C. Educated the patient on the risk factors and side effects surgery

D. Ensure that the patient has a post-surgery physical therapy

A patient hospitalized with chest pain is diagnosed with angina. Which


of the following discharge instruction takes priority?
A. recognize signs and symptoms that require immediate
B. Maintain low cholesterol, low sodium, and low potassium

C. Carry heavy objects away from the chest area


D. Maintain physical exercise daily
A 45-year-old patient admitted with pulmonary tuberculosis nurse
placed the patient in an isolation room with negative and prepared all the
personal protective equipment at the entrance room. What type of
precaution measure has the nurse activated?
A. Contact
B. Droplet
C. Airborne
D. Standard

A 55-year-old patient was admitted in the Medical ward investigation of


his right sided severe headache. He had seizures and occasional
vomiting episodes for the past 12 months. After tests and computerized
topography scan, he was informed by physician of the presence of a
small brain tumor on the right region which would possibly be
malignant. The patient refused to eat his lunch and started crying. Then
he started the nurse who attentively listened to him until he calmed
down. Which one of the following will be the suggested long-term home
health care?
A. Identify community support system
B. Share his grief with significant others
C. Develop acceptance of his medical situation
D. Renew activities and relationship as before
A physician ordered an ECG for a 35-year-old woman with chest pain.
What is the main finding that can be detected by the ECG?
A. Abnormality with the valves in the heart

B. Abnormality in the ejection function


C. Myocardial ischemia or infarction
D. Cardiomegaly

A 25-year-old patient who delivered a baby through caesarean was


planned for her to be up after 24 hours and to be discharged fifth day of
her surgery. Which of the following intervention should the nurse do
regains her consciousness?
A. Help her urinate in the bed pan
B. Clean her with wet sponge
C. Elevate the head side
D. Check her vital signs

A patient with a spinal cord injury states, "I have no constitution; I can’t
do anything for myself." Which of the following best describes this
patient condition?
A. Powerlessness
B. Delusions

C. Suicidal
D. Resignation
A 17-year-old patient is in the outpatient Department with he the follow
up after the lithotripsy of her right kidney. She was bringing the early
morning urine sample, which she is going to clinic. While preparing her
for the physical examination hesitant and asked the nurse if it was really
necessary. Which of the following is the best intervention?
A. Leave her alone with the physician
B. Explain importance of physical
C. Cover her appropriately and maintain privacy
D. Send mother away while the nurse stays with her

Which components most likely be utilized instruct methodologies?


A. Specific formal documents that provide a framework on
procedure or intervention should be conducted
B. Systematic procedures that follow a logical progression added
information or client responses to treatment
C. Current research strategies that are developed by profe decrease
variations in practice.
D. Documents that orient the nurse easily to the client’s the day by
framing the expected course of treatment.

What is the best course of action by a nurse to examine nerve?


A. Check pupil size

B. Test hearing acuity


C. Inspect the tongue
D. Examine trapezius muscle
patient is being weaned off from the mechanical ventilator is about to
hook the endotracheal tube to oxygen at FiO2 of 40 Which of the
following oxygen administration device is the best the nurse in this
situation?
A. Ambo bag
B. Ventura mask
C. Tracheostomy collar
D. T-piece/ Briggs

A 22-year-old man presents to the Emergency Department complaints of


breathing difficulties. He appears restless and reports that he has had a
cough with thick green sputum for days. The nurse performs
auscultation over the lung fields.
Blood pressure 130/80 mmHg
Heart rate 100 /min, irregular
Respiratory rate 24 /min, irregular

Temperature 39.0C
Which clinical finding is most likely heard over the right lower?
A. Decreased vocal resonance
B. Decreased fremitus
C. Tympani
D. Bronchial
A37 year-old man presents to Emergency Department chest pain. An
ECG shows significant elevation in the ST segment II, III, and aVf,
indicating MI related to occlusion in the artery. What is the location of
the MI?
A. Posterior MI
B. Anterior MI
C. Inferior MI
D. Lateral MI

A medical-surgical unit implemented a quality improvement addressing


patient pain relief and management. Which of the following set of
criteria is the most important the goal has been achieved?
A. Chart audits found patient self-reported pain scores 10
B. Number of narcotics used on the unit increased by 20
C. Positive comments on returned patient satisfaction by 30
D. In recent survey, ninety percent(90 ) of the nurses patients had
better pain control
A 60-year-old man client had a permanent pacemaker complains of
chest pain and dyspnea with rapid shallow breathing, feels suffocated
and appears restless.

Blood Pressure 100/70 mmHg


Heart Rate 96 /min
Respiratory Rate 32 /min
Temperature 37.2
What is the immediate nursing intervention?
A. Monitor and report findings of chest auscultation

B. Chest X-ray to identify dislocation of pacemaker


C. Manage pain with medication as ordered
D. Administer oxygen as ordered

An Indian patient, who is vegetarian, is being discharged from after an


elective surgical procedure. The unit nurse is teaching using visual aids
and pictures about food combinations complete protein. Which of the
following food items should the nurse recommended dietary list of the
patient?
A. Lentils
B. Potatoes
C. Macaroni
D. Green salad
A medication was ordered by physician for a patient. The nurse noticed
that the dose was inaccurate based on patient’s body weight. Which of
the following is the most appropriate nursing action?

A. Seek another nurse’s opinion


B. Administer the medication as prescribed
C. Hold the medication and contact the physician immediately
D. Contact the nurse manager

The nurse is mentoring a newly graduated nurse who is right-handed 39


year-old woman who underwent a left-sided and is now in the medical-
surgical unit. The intravenous fluid to infiltrate into the infusion site and
the student nurse equipment to insert a new intravenous catheter. Which
catheter insertion site would indicate that the new nurse needs to be
corrected?
A. A site that has soft and elastic skin
B. A site next to a joint

C. The right arm


D. A distal site
A patient is admitted to the hospital with a cerebrovascular hemiplegia,
and expressive aphasia. With a nursing diagnosis of Impaired verbal
communication, best short-term goal for this patient?

A. Learn to speak clearly within 30 days o


B. Communicate effectively within one week
C. Have all needs anticipated by staff daily
D. Make basic needs known daily

A 43-year-old man in the post-surgical area complains of abdominal


pain radiating to the naval which is increasing with examination his
abdomen is guarded with marked tender lower quadrant.
What is the immediate goal of care to do?
A. Teach abdominal splinting during coughing
B. Administer pain medication as ordered
C. Assess pain and report immediately
D. Position on the left lateral side
A Cardiac Care unit nurse decide to gather relevant data of myocardia
infraction patient who has breathing difficulty, bradycardia. What is the
best kind of databases relevant to his situation for the nurse?

A. Focused database
B. Complete database
C. Emergency database
D. Follow-up database

A 68-year-old woman is admitted to the Medical ward for Hep has


abdominal distention with rapid shallow breathing. exhausted and dull.
Which of the following short-term goal needs to be addressed?
A. Focus on breathing and proper positioning
B. Promote rest to reduce demand on the liver
C. Skin care and optimum nutrition and hydration
D. Isometric exercises to prevent thromboembolism

27-year-old man is brought in Emergency Room after a lower back and


abdomen in the road traffic accident. He has passing urine and
complains of dull pain and feeling of full back, abdomen and thighs. An
emergency laparoscopy is order Which problem needs immediate
attention?
A. Anxiety and restlessness

B. Disturbed body function


C. Immobility and dysuria
D. Pain and discomfort
A patient with chronic kidney disease receives hemodialysis week. The
nurse notices that the arteriovenous fistula is red (see lab results).
Test Result Normal Values

WBC 12.0 4-10.5x10^9/L


Glucose 5.3 3.5-6.5 mmol/L
HCT 0.38 0.41-0.50
Iron 10.7 11.7-31.8 mol/L
Phosphorus 1.4 2.5 mg/dL
Which of the following nursing diagnoses takes priority?

A. Activity intolerance related to generalized weakness se


B. Impaired skin integrity related to dry skin secondary
hypophosphatemia
C. Constipation related to changes in iron level secondary hemodialysis
D. Risk for infection related to invasive exposure secondary
A 39-yar-old woman is admitted to the Endocrinology pancreatitis. She
appeared exhausted and restless due to abdominal pain radiating to her
back. The pain is graded 6 out of 10 (see lab result).

Blood Pressure 100/60 mmHg


Heart Rate 110 /min
Respiratory 26 /min
Temperature 37.8
Test Result Normal Value
Fasting blood glucose 110 77-99 mg/dl

Which of the following will be the initial short-term goal?


A. Maintain blood glucose level within normal range
B. Restore energy with more rest and mild activities
C. Alleviate exhaustion and minimize inflammation
D. Manage pain and reduce body temperature

To reduce the risk of transmitting methicillin-resistant staphylococcus


aurous (MRSA) from an infectious wound, which of the following
precautions should be implemented?
A. Airborne
B. Contact
C. Droplet

D. Reverse isolation
A75 year-old man is scheduled to undergo phacoemulsification surgery.
The nurse meets with the family for a pre-operative. The patient has a
diagnosis of Alzheimer’s disease with memory decline. The family says
that he has become increasingly for and that his emotions are unstable.
Which of the following actions the patient is expected to do operation?
A. Lay supine for 20 minutes
B. Sit without moving under laser
C. Consistently speak clearly
D. Tolerate general anesthesia

A client in the second stage of labor is unable to push and lacks to bear
down. What is the most appropriate next step?
A. Assess fetal descent
B. Infuse intravenous fluids
C. Empty the client’s bladder

D. Administer oxygen to the mother

The community midwife is discussing the importance of antenatal with a


group of primigravida mothers who are in their first trimester Which of
the following long-term goal of care needs more emphasis?
A. Minimize health hazards and risk factors

B. Prepare psychologically for parenthood


C. Discuss birth plan and spacing
D. Identify mode of delivery
A multiparous woman in labor, cervical dilatation was 7 cm 2 who states
that she feels the urge to move her bowels. Which of the following in the
best nursing action?

A. Let the women go to the bathroom.


B. Allow the woman to use bedpan.
C. Preform vaginal exam.
D. Assess fetal heart rate.

The nurse is receiving a child postoperative tonsillectomy. Which of the


following nursing assessment is suitable for the postop care?
A. Encourage the child to cough spontaneously
B. Observe for subtle signs of hemorrhage
C. Place the child in the prone position
D. Suction the mouth to clear the airway Because of violent behavior,

a woman was brought to the psychiatric ward because she attempting to


harm her brother. A doctor diagnosed her with schizophrenia. Which of
the following nursing intervention will be prioritized in of this client?
A. Promote reality testing
B. Make frequent and brief contacts
C. Start with one-to-one interactions

D. Establish trust through therapeutic nurse-patient relationship


Which of the following patient is the most appropriate to delegate new
nurse?
A. Patient on a strict bed rest

B. Patient for discharge teaching


C. Patient scheduled for an operation
D. Patient who is hooked to mechanical ventilation

Which of the following condition is a contraindication for a woman oral


contraceptive?

A. Dysmenorrhea
B. Menorrhagia
C. Thrombophlebitis
D. Toxic shock syndrome

Mother of nine children, three of them with congenital anomalies Down


syndrome; she is a primary school graduate, with low status. She is not
using any method of family planning. So, the health care nurse has
referred her for counseling. Which of the following application the
counselor can help Mrs. regarding family planning?
A. Prevention level
B. Gather model

C. Group teaching
D. Rejection of Mrs. M expression
Mother of nine children, three of them with congenital anomalies Down
syndrome; she is a primary school graduate, with low status. She is not
using any method of family planning. According primary health care
nurse referred her for counseling Which of the following types of home
visits that the community her nurse should conduct for this client?
A. Systematic routine
B. Selective
C. Follow up
D. Field trip

A nurse was educating a group of women on prevention of infection.


The nurse asked each woman to state one preventive of vaginal
infection. Which woman needs more education?
A. First woman "keep vaginal area clean & dry"
B. Second woman "wear cotton under wear"
C. Third woman "wipe from front to back after urination or

D. Fourth woman "Do vaginal douche twice a day "

A 3-year-old child is admitted to the Medical Ward for vomiting, and


dehydration. The nurse sat with the parents to comply admission
interview and wanted to get as much information as Which of the
following communication techniques should the nurse

A. Use of question containing the word "how"


B. Use of question with direct comments to clarify
C. Use of statements that indicate patient will be fine
D. Use of leading question and those involving yes or no
A 27 years old woman patient’s umbilical hernia is repaired under local
anesthesia. Her gold amulet which she kept under her pillow was
missing at first but now is given to her after it is found from the bag of
the housekeeping staff. The nurse in charge is to write an incident report
which includes the patient’s name, hospital identification number, date,
time, and place of incident and details of the item stolen. What
professional issue is evaluated from the scenario?
A. Fulfilment of ethical obligation
B. Honesty in dealing with the patient

C. Necessity of following hospital protocol


D. High legal value of the incident reporting

A 15-year-old girl is admitted after a Motor Vehicle Accident has a


fracture in her left leg and a wound in her face. When dealing with this
child the nurse should be aware that important task to achieve during
this age is which of the following?
A. Initiative versus guilt
B. Industry versus inferiority
C. Trust versus mistrust
D. Identity vs. Role confusion
A 20-yar-old primigravida, who was admitted to the labor ward ago
assessed by a nurse. Vaginal examination reveals the following 3 cm
dilated, 100% effaced, station at-2. Contractions: every 3 minutes lasting
40-50 seconds. Fetal heart rate: 130-140 bpm. Complaining about
annoying back ache. Which from of pain management is suitable for this
patient at this labor?
A. Nitrous Oxide
B. Intra-theca opioid analgesics
C. Non-pharmacological pain management such as walking

D. Regional pain management such as an epidural block

To decrease the anxiety of a 10-year-old girl who is undergoing surgery.


Which of the following should the nurse do?
A. Use a heart model to show her how the surgery will go
B. Provide her with verbal explanation of the upcoming surgery
C. Give her a book to read about the surgery 2 weeks prior

D. Let her parents talk to her about the importance of having surgery
A 3-year-old child is admitted to the hospital with seizures. He oriented
and has a rash in his extremities and is diagnosed meningitis. While
doing physical examination of him, he starts to seizures.

Blood pressure 100/57 mmHg


Heart rate 110 /min
Respiratory rate 30 /min
Temperature 39.5
Which of the following is the priority of care during the seizure?
A. Put the child on right side

B. Protect the child from injury


C. Call the physician immediately
D. Administer oxygen 100%

A nursing student was learning about Maslow’s hierarchy of basic the


university. He then expressed his concern about the relevance topic to
nursing practice. The lecturer told him that it is planning and
implementing nursing care. What Maslow’s hierarchy of basic needs
provide to nursing care?
A. Establishing priority of care
B. Making accurate nursing diagnosis
C. Integrating science into nursing care
D. Communicating concerns more concisely
The nurse begins a morning shift with the following see the doctor in the
waiting room (see table).
1. A first time mother who delivered two days ago and is having
Difficulties breastfeeding
2. A60 year-old man who has a history of deep vein Thrombosis and is
taking daily heparin
3. A76 year-old women who was hospitalized three days Previously with
pneumonia
4. A 56-year-old man who has used all his diuretic pills Coughing up
sputum
Which patient should be seen first?
A. One
B. Two
C. Three
D. Four

which of the following best describes the role of a case management?


A - promotes healthy lifestyle.
B - provides coordination of care to ensure continuity.
C - possesses highly skilled communication methods.
D - promotes the comfort of the patient.
A 25-year-old-primipare visited her primary health care clinic with
husband. He says that his wife delivered 5 days ago. For the past days
she is very irritable and crying for nothing, she is tired and sleep and he
is very concerned about her condition. Which of the following diagnosis
by the attending physician will her symptoms?
A. Post-partum blues
B. Postpartum psychosis
C. Postpartum depression
D. Postpartum anxiety

A new nurse threw a needle into the waste basket. When asked action,
the new nurse admits the mistake. Which professional act best describes
the nurse’s response?
A. Responsibility
B. Accountability

C. Assertiveness
D. Autonomy
A nurse prepares to administer a vitamin K injection to a full term the
mother wants to know the importance of the injection. Which of the
following is the best nurse response to the mother?

A. Needed for blood clotting to prevent hemorrhage


B. Accelerate the growth and development of infants
C. Help in maintain healthy gut and passage of meconium
D. Protect the infant from developing severe respiratory distress

The intensive care unit nurse manager plans to delegates a nurse.


What is indicated for a successful delegation?
A. Nurse manager supervises nurse.
B. Nurse has authority to change task.
C. Nurse manager checks task sometimes.
D. Nurse manager asks another nurse to check task.

A 13-year-old child with an elevated body temperature is administer


aspirin. The nurse records the body temperature of the child later and
finds that it is still elevated. What is the most likely underlying
physiology for the delayed the action of the aspirin?
A. High gastric pH
B. Thin epidermis

C. Low muscle tone


D. Short intestines
An 11-month-old girl is brought to the Emergency Department frequent
diarrhea. Her mother reported that she passed loos 7 times since the
morning. She seems to be thirsty and irritable. Examination revealed dry
mucus membrane and poor skin turgor.
Heart rate 150 /min
Respiratory rate 35 /min
Temperature 37.1
Weight 8 kg
Which of the following degrees of dehydration does the infant

o A. Extensive dehydration
o B. Mild dehydration
o C. Moderate dehydration
o D. Severe dehydration

In a discharge plan of newly diagnosed patient with Hepatitis C,


encourages the patient and family to cope with the disease pro includes
taking medication regularly, good nutrition, and appointments. Which of
the following best explains the nurse’s role the family?
A. Help understand patient’s condition
B. Ask to respect patient’s privacy

C. Explain the disease process


D. Guide to be supportive
A 10-year-old girl presents to the Emergency Room (ER) with pain. On
assessment, you noticed that when you palpate the quadrant of the
child’s abdomen, the child feels pain in the right quadrant. Which of the
following is the name of this sign?
o A. Rebound tenderness
o B. McBurney sign
C. Roving’s sign
D. D. Obdurate sign

A patient was on a regular does of lithium carbonate. The nurse that he


has hand tremor, polyuria, diarrhea and vomiting. What immediate
action should be taken by the nurse?
A. Diuretics
B. Withholding lithium
C. Calling the psychiatrist

D. Monitoring serum lithium level


A 9-month-old child who has a repair cleft palate the nurse explaining
mother on how she will give feeds to her child. Which of the following
instruction can be expected to include feeding education?

A. Open cup
B. Tea spoon
C. Bottle feed
D. Special bottle feed

Upon reviewing the pregnant client’s blood test results, the nurse that
traces of mercury are present even after the nurse had healthy dietary
modifications. Which action of the client does the nurse discuss to
reduce risk client?
A. Client has five soaked walnuts every day
B. Client consumes king mackerel very often
C. Client eats one medium bowl of flax seeds daily
D. client has cooked soybean seeds as an evening snack

A 25year-old woman presents to the Emergency Room with problem.


Her main complaint is that she woke up that morning see anything. All
medical examination was normal. However, reported that two days ago
her husband had asked for a divorce. What is the most likely disorder?
A. Pain

B. Conversion
C. Somatization
D. Body Dimorphic
A 32-year-old prim pare attended the postnatal clinic 4 days post- she
says she is keen to breastfeed but the baby to the painful. The nurse
examined the breasts and found that the red and cracked. Which will be
nurse advice to her to help the woman situation?
A. Apply antibiotic nipple cream to prevent infection
B. Use correct positioning of the infant to latch on nipple
C. Use the same position when feeding not to confuse the
D. Use breast pads with plastic lining to prevent leaking of

A baby born at 35-week was admitted in the Neonatal Intensive 27 hours


ago. Physical examination revealed yellow discoloration sclera and
mucus membrane. The result of bilirubin level every 170mol. The infant
was diagnosed with neonatal jaundice physician order to start single
phototherapy. Which of the following should the nurse consider as a
priority during phototherapy of this newborn?

A. Ensure proper fitting of eye covering (patches)


B. Monitor bilirubin levels every 48 hours
C. Feed the infant formula every 4 to 5 hours
D. Avoid removing the infant from phototherapy
A group of mothers, in their third trimesters, are attending the education
session regarding the alert signs for which they are immediately contact
the hospital. The midwife is to assess the understanding of the session by
asking question. What response from the mothers helps assure the
midwife about right understanding?
o A. Gush of urinary outflow without any urge
o B. Low back pain at night and difficulty in sleep
o C. Activity intolerance and breathlessness on exertion
o D. Absence of contraction after 42 weeks of pregnancy

A nine-month-old child who has had four ear infection six months is
being discharged. Which statement by the parent indicates the need for
teaching?
A. I should never put my baby to bed with a bottle
B. My child should not use a pacifier after age six
C. My child should drink his bottle while lying flat

D. My child should not be around people who sm

A pregnant women of 15 weeks gestation and Rh-negative has an


abortion admitted in the ward. What should the nurse do for this
woman?
A. Administer Rheum within 72 hour

B. Do not dive Rheum since it is not used with abortion


C. Do not give Rheum since the pregnancy is more than 12
D. Make certain she receives Rheum on her first clinic visit
A nurse is performing physical examination on a newborn. She that the
baby has developed cephalohematoma. This baby is at risk of which of
the following?

A. Sudden death
B. Pathological jaundice
C. Infected umbilical cot
D. Increased intracranial pressure

A 9-year-old child is admitted to pediatric word diagnosed with Acute


Glomerulonephritis (AGN). The mother asks the nurse why you check
the blood pressure frequently Which of the following the nurse should
be respond based on knowledge?
A. Acute hypertension must be anticipated and identified
B. Hypotension leading to sudden shock can develop at any

C. Blood pressure fluctuations are a common side effect of therapy


D. Blood pressure fluctuation are a sign that the condition has become
chronic
A 20-weeks-pregnant, primary gravid woman visits the Antenatal she
has sickle cell anemia trait and worried that this disease transmitted to
her baby. Which of the following should be the initial intervention?

A. Plan for the fetal genetic screening


B. Educate mother that her disease is inactive .
C. Discuss the chances of genetic disease in the fetus
D. Gather data about other family members having the disease

A 25-year-old mother gravid 2 para q came for a routine check


Antenatal Clinic. The nurse assessed fetal heart rate for the 38 pregnant
mother. What is the expected normal fetal heart rate per minute?
A. 90
B. 100
C. 140
D.170

In determining the one minute APGAR score of a male in assesses a


heart rate of 120 beats per minute and respire 44 per minute. He has
flaccid muscle tone with slight flex resistance to straightening. He has a
loud cry with stimula colour is acrocyanotic. What is the APGAR score
for the infant?
A. 7

B. 8
C. 9
D. 10
A 30-gestation preterm is admitted to the Neonatal Care Unit 2 hours
ago. The neonate starts to have grunting, tac and nasal flaring. Which of
the following should the nurse recognize with regard above signs and
symptoms?
A. Neonate has respiratory distress syndrome
B. It normally within 24 hours after birth
C. This is not significant unless becomes cyanosis
D. Neonate has hypoglycemia

a nurse is caring for a child with traction of fractured bone. In the chart,
a doctor has placed a reminder to maintain even and constant traction.
What would be the most likely understanding of the nurse for this order?
A. Add or remove weights every other day
B. Allow weights to hang free continuously
C. Elevate head and foot of the bed alternatively when in pain

D. Allow weights to hang free every 12 hours to achieve good


circulation
A 9-year-old girl is seen in the emergency department because of a
fracture in the right fibula. Which of the following is the expected
response to parent question about bone healing period of the girl?

A. 2-4 weeks
B. 4-6 weeks
C. 6-8 weeks
D. 8-10 weeks

Which of the following represents secondary infertility?

A. Male infertility
B. A couple that does not conceive
C. Infertility that occurs after previous pregnancy
D. Infertility lasts for more than 3 years

A patient returns to a clinic for a follow-up visit and is diagnosed as


positive for Human Immunodeficiency Virus ( HIV). The patient
expresses fear related to lack of finances, fear of social avoidance, and
hopelessness. Which of the following nursing actions should provide the
most patient support?
A. Referral to a physician infectious disease specialist
B. Referral to a community-based HIV clinic

C. Referral to the local public health department


D. Recommendation to disclose the diagnosis to family
which type of study design provides the strongest evidence?
A. Qualitative study
B. Randomized control trial

C. Systematic review of descriptive studies


D. Systematic review of correlational studies

Which of the following is most appropriate to delegate assistant?


A. Insertion of an oral airway
B. Changing postoperative dressing

C. Initial interview on a newly admitted patient


D. Repositioning a patient to side-lying position

The nursing director wants to evaluate the quality of nursing care at the
in-patient areas. The management team will evaluate on quarterly basis
the documentation and the relationship between the patient's length of
stay and the quality of care. Which of the following is the most
important data source to identify the quality care?
A. Patient's complaints and time taken to resolve them
B. Patient's satisfaction level at the time of discharge
C. Details of nursing notes for patient's progress
D. Relation between incidents and cost of care
A mother, who is planned for the labor induction, is started on
intravenous medication. She is in the first stage of her labor and is
having regular and increasingly stronger uterine contractions. Her cervix
is 1 cm dilated for the past few hours; both the mother and the baby are
being monitored. Which of the following signs should alert the midwife?
A. Baby's head not engaged
B. Decreasing heart rate of the baby
C. Mother's blood pressure 110/60 mmHg
D. Mother's perspiration and increased thirst

When the nurse assessed the fundus of a multiparous mother who


delivered 2 hours ago, she found the following: Level: 2 cm above the
umbilicus. Position: deviated to the right. Consistency: Not well
contracted. what is the next nursing action after massaging the fundus
until it becomes firm?
A. Assess vital signs
B. Increase IV fluids
C. Evacuate the bladder

D. Ask the mother to rest


A nurse is leading an educational session on the correct use of oral
contraceptives. One of the attendees ask the nurse what to do if she
missed taking an oral contraceptive for one day? Which of the following
should be the nurse advice to her?
A. Continue as usual with no back up contraception
B. Take an active pill immediately and take the next pill at the usual
time
C. Take two pills as soon as possible and then one pill daily at the usual
time

D. Use back up contraception such as a condom for the next 7 days

A mother of a sickle cell anemia child is asking why her child can't go
hiking with his friends. Which of the following complications hiking can
leads to?
A. Enhance iron absorption

B. Decrease oxygen consumption


C. Inhibit hemoglobin production
D. Precipitate vaso-oclusive crisis
A man nurse is caring for a women diagnosed with major depression.
The client states " I never had a male nurse before". which of the
following is the most appropriate nursing response?

A. " Do you feel threatened"


B. "Most of the nurses here are males"
C. " How do you feel having a male nurse"
D. " Do you want me to find a female nurse"

The nurse is teaching a 32-weeks pregnant woman how to distinguish


between pre-labor (false) contractions and true labor contractions.
Which statement about pre-labor contraction is accurate?
A. They are regular and increase gradually
B. They are felt in the abdomen
C. They start at the back and radiate to the abdomen

D. They become more intense during walking


A mother brought her 6-month-old healthy infant to the well-baby clinic
Which immunization should the nurse anticipate to administer as per
World Health Organization's recommendation?

A. Varicella (Chicken pox)


B. Rotavirus and hepatitis
C. Measles, Mumps, Rubella
D. Diphtheria, Tetanus and pertussis

A postpartum mother is to be discharged on the second day of her


forceps delivery. She had sutures on her vaginal and perineum tears. She
is breastfeeding her baby and eating the special food provided by Her
family. what discharge teaching needs more emphasis?
A. Diet management and exercise plan
B. Newborn care and vaccination records
C. Hygiene practices and alert signs to report
D. Family planning and child growth monitoring
After receiving multiple mosquito bites and experiencing flulike
symptoms, a patient consults a nurse at a clinic and asks whether an
appointment to see a health-care provider is necessary. which of the
following statements should be the basis for the nurse's response?
A. antiviral medications can be prescribed to destroy virus
B. Clinical signs can be mild flu-like symptoms to fatal encephalitis
C. If patient has West Nile virus, symptoms will progressively worsen
D. If patient used insect repellent, virus would have been destroyed
when mosquito made skin contact

A 7-month-old infant seen in the Emergency Department suffering from


episodes of severe abdominal pain, and the infant's stool become like red
jelly. Abdominal examination revealed palpable sausage-shaped mass in
the right upper quadrant. which of the following is the best diagnosis?
A. Hirschsprung disease
B. Hypertrophic pyloric stenosis

C. Infant colic
D. Intussusception
A 33-year-old women has come to the outpatient clinic for treatment of
a vaginal infection. Physical assessment reveals yellowish, excessive,
thin, offensive and frothy discharge. Which of the following is the most
likely diagnosis?

A. Candidiasis
B. Trichomoniasis
C. Bacterial vaginosis
D. Chlamydia
A 24-weeks-pregnant mother is in the Antenatal Clinic. She is upset and
crying about having her third baby girl as her family wants a boy. She is
requesting an abortion immediately before her family finds out about the
sex of the baby. Which of the following is the most appropriate plan for
the mother?
A. Calm her down and reassure for an appropriate solution
B. Provide moral support and book her for procedure
C. Repeat ultrasound and wait for a few more weeks
D. Family counselling and follow religious guidance

While taking the history from a new patient, the nurse identified that he
had hypomanic episode which was alternating with a mirror depressive
episode for the last two years. what is the most likely diagnosis?
A. Bipolar I disorder
B. Bipolar II disorder
C. Dysthymic disorder

D. Cyclothymic disorder
A 30-year-old women was admitted with ectopic pregnancy on the sixth
gestational week. The patient was scheduled for resection of the
involved fallopian tube with end to end anastomosis. Which is the initial
nursing diagnosis for this patient?
A. Grieving

B. Acute pain
C. Hyperthermia
D. Knowledge deficit
A 17-year-old mother presented to the primary health center ten days
after delivery. She is suffering from fatigue, anemia, fever and vaginal
discharge (see lab results)

Blood pressure 80/50 mmHg


Heart rate 112 /min
Respiratory rate 35 /min
Temperature 39.6C
Test Result Normal Values
RBC 4 4.7-6.1 × 1012 /L (male)

4.2-5.4 × 1012 /L (female)


Hb 90 130-170 g/L (male)
120-160 g/L (female)
HCT 0.29 0.42-0.52 (male)
0.37-0.48 (female)
WBC 12.8 4.5-10.5 × 109/L
Which of the following is the best diagnosis of health problem in this
case?
A. Severe urinary tract infection
B. Vesico-vaginal fistula
C. Puerperal sepsis
D. Post-partum hemorrhage
While the nurse is performing postpartum assessment for a Primiparous
mother delivered 2 hours ago, she found the following: Fundus level:
Midway between symphysis pubis and umbilicus. Position: At the mid
line. Consistency: Contracted. Lochia: Constantly
Which of the following is the most likely diagnosis?
A. full bladder
B. Perineal hematoma
C. Birth canal laceration
D. Retained placental part

A 62-year-old women admitted to the emergency department for the


fourth time this year, each time the patient comes with severe injuries
and bruises in the body. What is your responsibility as a nurse to prevent
such incident to happen again?
A. Reports the assault to the local police and write a report

B. Provides information about safe shelter and support


C. Instructs the women to move away from her home
D. Discharge the patient to a safe shelter

You might also like